You are on page 1of 70

Amendments & Revisions to enact a regional, provincial, city, municipal, or

barangay law, resolution or ordinance


1. GARCIA V. COMELEC Under its Sec.16(a), it provided the limitations on
Sept. 30, 1994 local initiatives, which is “the power of local initiative
FACTS: shall not be exercised more than once a year.”
On May 24, 1993, petitioners filed a petition with the
Sangguniang Bayan of Morong to annul Pambansang 3. Lambino v. Comelec - 2006
Kapasyahan Blg. 10, Serye 1993 which includes the Fact:
Municipaloty of Morong as part of the Subic Special The Lambino Group commenced gathering
Economic Zone in accord with the RA No. 7227. signatures for an initiative petition to change the 1987
The municipality did not take any action on the Constitution and then filed a petition with Comelec to
petition within 30 days after its submission; so, they hold a plebiscite for ratification under Sec. 5(b) and
resorted to their power of initiative under the Local (c) and Sec. 7 of RA 6735.
Government Code of 1991. They solicited the The proposed changes under the petition will shift the
required number of signatures to repeal the said present Bicameral-Presidential system to a
resolution. Unicameral-Parliamentary form of government.
However, the Vice Mayor, Hon. Edilberto de Leon, Comelec did not give it due course for lack of an
and the Presiding Office of the Sangguniang Bayan enabling law governing initiative petitions to amend
ng Morong wrote a letter dated June 11, 1993 to deny the Constitution, pursuant to Santiago v. Comelec
the petition for local initiative and/or referendum. ruling.
On July 6, 1993, the Comelec denied the petition for Issue:
local initiative because its subject is “merely a w/n the proposed changes constitute an amendment
resolution and not an ordinance.” or revision
ISSUE: w/n the initiative petition is sufficient compliance
w/n the Pambansang Kapasyahan Blg. 10, Serye 1993 with the constitutional requirement on direct proposal
is the proper subject of an initiative? by the people
Sub-issue: w/n the decision of the Comelec to deny Held:
the petition be set aside? Initiative petition does not comply with Sec. 2, Art.
HELD: XVII on direct proposal by people
The petition is granted and the decision of the Sec. 2, Art. XVII…is the governing provision that
Comelec on July 6, 1993 is annulled and set aside. allows a people’s initiative to propose amendments to
RULING: the Constitution. While this provision does not
The 1987 Constitution installed back the power to the expressly state that the petition must set forth the full
people regarding legislation because of the event in text of the proposed amendments, the deliberations of
February 1986. The new Constitution became “less the framers of our Constitution clearly show that:
trusting of public officials.” a. the framers intended to adopt relevant American
Through initiative, the people were given the power jurisprudence on people’s initiative; and
to amend the Constitution under Sec. 2 Art. 17 which b. in particular, the people must first see the full text
provides “amendments to this Constitution may of the proposed amendments before they sign, and
likewise be directly proposed by the people through that the people must sign on a petition containing such
initiative upon a petition of at least 12% of the total full text
number of registered voters, of which every The essence of amendments “directly proposed by the
legislative district must be represented by at least 3% people through initiative upon a petition” is that the
of the registered voter therein.” entire proposal on its face is a petition by the people.
The Comelec was also empowered to enforce and This means two essential elements must be present:
administer all laws and regulations relative to the a. the people must author sign the entire proposal. No
conduct of an initiative and referendum. agent or representative can sign on their behalf
On Aug. 4, 1989, the Congress approved RA No. b. as an initiative upon a petition, the proposal must
6735 entitled “An Act Providing for a System of be embodied in a petition
Initiative and Referendum and Appropriating Funds These essential elements are present only if the full
Therefor.” text of the proposed amendments is first shown to the
YES. Sec. 32 of Art. 6 provides “ the Congress shall people who express their assent by signing such
provide for a system of initiative and referendum, and complete proposal in a petition. The full text of the
the exceptions therefrom, whereby the people can proposed amendments may be either written on the
directly propose face of the petition, or attached to it. If so attached,
and enact laws or approve or reject any act or law or the petition must state the fact of such attachment.
part thereof passed by the Congress or local This is an assurance that every one of the several
legislative body. millions of signatories to the petition had seen the full
Under Sec. 32(a) of RA No. 6735 it provided the 3 text of the proposed amendments before – not after –
systems of initiative, namely: signing.
1. Initiative on the Constitution – petition to amend Moreover, “an initiative signer must be informed at
the Constitution the time of signing of the nature and effect of that
2. Initiative on statutes – petition proposing to enact a which is proposed” and failure to do so is “deceptive
national legislation and misleading” which renders the initiative void.
3. Initiative on local legislation – petition proposing In the case of the Lambino Group’s petition, there’s
not a single word, phrase, or sentence of text of the
Page 1 of 70
proposed changes in the signature sheet. Neither does Executive – affecting a total of 105 provisions in the
the signature sheet state that the text of the proposed entire Constitution. Qualitatively, the proposed
changes is attached to it. The signature sheet merely changes alter substantially the basic plan of
asks a question whether the people approve a shift government, from presidential to parliamentary, and
from the Bicameral-Presidential to the Unicameral- from a bicameral to a unicameral legislature.
Parliamentary system of government. The signature A change in the structure of government is a revision
sheet does not show to the people the draft of the A change in the structure of government is a revision
proposed changes before they are asked to sign the of the Constitution, as when the three great co-equal
signature sheet. This omission is fatal. branches of government in the present Constitution
The initiative violates Section 2, Article XVII of the are reduced into two. This alters the separation of
Constitution disallowing revision through initiatives powers in the Constitution. A shift from the present
Article XVII of the Constitution speaks of three Bicameral-Presidential system to a Unicameral-
modes of amending the Constitution. The first mode Parliamentary system is a revision of the
is through Congress upon three-fourths vote of all its Constitution. Merging the legislative and executive
Members. The second mode is through a branches is a radical change in the structure of
constitutional convention. The third mode is through government. The abolition alone of the Office of the
a people’s initiative. President as the locus of Executive Power alters the
Section 1 of Article XVII, referring to the first and separation of powers and thus constitutes a revision
second modes, applies to “any amendment to, or of the Constitution. Likewise, the abolition alone of
revision of, this Constitution.” In contrast, Section 2 one chamber of Congress alters the system of checks-
of Article XVII, referring to the third mode, applies and-balances within the legislature and constitutes a
only to “amendments to this Constitution.” This revision of the Constitution.
distinction was intentional as shown by the The Lambino Group theorizes that the difference
deliberations of the Constitutional Commission. A between amendment and revision is only one of
people’s initiative to change the Constitution applies procedure, not of substance. The Lambino Group
only to an amendment of the Constitution and not to posits that when a deliberative body drafts and
its revision. In contrast, Congress or a constitutional proposes changes to the Constitution, substantive
convention can propose both amendments and changes are called revisions because members of the
revisions to the Constitution. deliberative body work full-time on the changes. The
Does the Lambino Group’s initiative constitute a same substantive changes, when proposed through an
revision of the Constitution? initiative, are called amendments because the changes
Yes. By any legal test and under any jurisdiction, a are made by ordinary people who do not make an
shift from a Bicameral-Presidential to a Unicameral- occupation, profession, or vocation out of such
Parliamentary system, involving the abolition of the endeavor. The SC, however, ruled that the express
Office of the President and the abolition of one intent of the framers and the plain language of the
chamber of Congress, is beyond doubt a revision, not Constitution contradict the Lambino Group’s theory.
a mere amendment. Where the intent of the framers and the language of
Tests to determine whether amendment or revision the Constitution are clear and plainly stated, courts do
In California where the initiative clause allows not deviate from such categorical intent and language.
amendments but not revisions to the constitution just
like in our Constitution, courts have developed a two- By Congress as Constituent Assembly
part test:
Quantitative test—asks whether the proposed change 4. Almario v. Alba, G.R. No. L-66088 January 25,
is so extensive in its provisions as to change directly 1984
the substantial entirety of the constitution by the CASE DIGEST
deletion or alteration of numerous existing
provisions. The court examines only the number of Facts: As provided for in Batas Pambansa Blg. 643,
provisions affected and does not consider the degree the Filipino electorate will go to the polls on January
of the change. 27, 1984 to either approve or reject amendments to
Qualitative test—inquires whether the change will the Constitution proposed by Resolution Nos. 104,
accomplish such far reaching changes in the nature of 105, 110, 111, 112, and 113 of the Batasang
our basic governmental plan as to amount to a Pambansa. The proposed amendments are embodied
revision. Whether there is an alteration in the in four (4) separate questions to be answered by
structure of government is a proper subject of inquiry. simple YES or NO answers. Petitioners herein seek
Thus, a change in the nature of the basic to enjoin the submission on January 27, 1984 of
governmental plan includes change in its fundamental Question Nos. 3 (“grant” as an additional mode of
framework or the fundamental powers of its acquiring lands belonging to the public domain) and
Branches. A change in the nature of the basic 4 (the undertaking by the government of a land reform
governmental plan also includes changes that program and a socialreform program), which cover
jeopardize the traditional form of government and the Resolution Nos. 105 and 113, to the people for
system of check and balances. ratification or rejection on the ground that there has
Under both the quantitative and qualitative tests, the been no fair and proper submission following the
Lambino Group’s initiative is a revision and not doctrine laid down in Tolentino v. COMELEC. The
merely an amendment. Quantitatively, the Lambino petitioners do not seek to prohibit the holding of the
Group’s proposed changes overhaul two articles – plebiscite but only ask for more time for the people to
Article VI on the Legislature and Article VII on the study the meaning and implications of Resolution
Page 2 of 70
Nos. 105 and 113 until the nature and effect of the proposed amendment would have been short of the
proposals are fairly and properly submitted to the necessary three-fourths vote in either branch of
electorate. Congress. The petition for prohibition sought to
prevent the enforcement of said congressional
ISSUE: Whether or not Questions 3 and 4 can be resolution, as it is allegedly contrary to the
presented to the people on a later date. Constitution. The members of the Commission on
Elections, the Treasurer of the Philippines, the
Auditor General, and the Director of the Bureau of
HELD: The necessity, expediency, and wisdom of the Printing are made defendants. Eight senators, 17
proposed amendments are beyond the power of the representatives, and the presidents of the Democratic
courts to adjudicate. Precisely, whether or not “grant” Alliance, the Popular Front and the Philippine Youth
of public land and “urban land reform” are unwise or Party.
improvident or whether or not theproposed
amendments are unnecessary is a matter which only Issue: Whether the Court may inquire upon the
the people can decide. The questions are presented for irregularities in the approval of the resolution
their determination. Assuming that a member or some proposing an amendment to the Constitution.
members of this Court may find undesirable any
additional mode of disposing of public land or an Held: It is a doctrine too well established to need
urban land reform program, the remedy is to vote citation of authorities that political questions are not
“NO” in the plebiscite but not to substitute his or their within the province of the judiciary, except to the
aversion to the proposed amendments by denying to extent that power to deal with such questions has been
the millions of voters an opportunity to express their conferred upon the courts by express constitutional or
own likes or dislikes. The issue before us has nothing statutory provision. This doctrine is predicated on the
to do with the wisdom of the proposed amendments, principle of the separation of powers, a principle also
their desirability, or the danger of the power being too well known to require elucidation or citation of
abused. The issue is whether or not the voters are authorities. The difficulty lies in determining what
aware of the wisdom, the desirability, or the dangers matters fall within the meaning of political question.
of abuse. The petitioners have failed to make out a The term is not susceptible of exact definition, and
case that the average voter does not know the precedents and authorities are not always in full
meaning of “grant” of public land or of “urban land harmony as to the scope of the restrictions, on this
reform. ground, on the courts to meddle with the actions of
the political departments of the government. If a
political question conclusively binds the judges out of
5. Mabanag vs. Vito Case Digest (Consti-1) respect to the political departments, a duly certified
law or resolution also binds the judges under the
Mabanag vs. Vito "enrolled bill rule" born of that respect. If ratification
[GR L-1123, 5 March 1947] of an amendment is a political question, a proposal
En Banc, Tuason (J): 3 concur, 1 concur in separate which leads to ratification has to be a political
opinion, 2 dissent in separate opinions, 1 filed question. The two steps complement each other in a
separate opinion scheme intended to achieve a single objective. It is to
Facts: Three senators and eight representatives had be noted that the amendatory process as provided in
been proclaimed by a majority vote of the section I of Article XV of the Philippine Constitution
Commission on Elections as having been elected "consists of (only) two distinct parts: proposal and
senators and representatives in the elections held on ratification." There is no logic in attaching political
23 April 1946. The three senators were suspended by character to one and withholding that character from
the Senate shortly after the opening of the first session the other. Proposal to amend the Constitution is a
of Congress following the elections, on account of highly political function performed by the Congress
alleged irregularities in their election. The eight in its sovereign legislative capacity and committed to
representatives since their election had not been its charge by the Constitution itself. The exercise of
allowed to sit in the lower House, except to take part this power is even in dependent of any intervention
in the election of the Speaker, for the same reason, by the Chief Executive. If on grounds of expediency
although they had not been formally suspended. A scrupulous attention of the judiciary be needed to
resolution for their suspension had been introduced in safeguard public interest, there is less reason for
the House of Representatives, but that resolution had judicial inquiry into the validity of a proposal then
not been acted upon definitely by the House when the into that of ratification.
petition for prohibition was filed. As a consequence
these three senators and eight representatives did not
take part in the passage of the congressional 7. OCCENA VS. COMELEC
resolution, designated "Resolution of both houses SAMUEL OCCENA VS. COMELEC
proposing an amendment to the Constitution of the G.R. NO. L-34150
Philippines to be appended as an ordinance thereto," APRIL 2, 1981
nor was their membership reckoned within the
computation of the necessary three-fourths vote FACTS: Petitioner Samuel Occena and Ramon A.
which is required in proposing an amendment to the Gozales instituted a prohibiting proceedings against
Constitution. If these members of Congress had been the validity of three batasang pambansa resolutions
counted, the affirmative votes in favor of the (Resolution No. 1 proposing an amendment allowing
Page 3 of 70
a natural-born citizen of the Philippines naturalized in constitution was complied as follows: "Any
a foreign country to own a limited area of land for amendment to, or revision of, this Constitution shall
residential purposes was approved by the vote of 122 be valid when ratified by a majority of the votes cast
to 5; Resolution No. 2 dealing with the Presidency, in a plebiscite which shall be held not later than three
the Prime Minister and the Cabinet, and the National months after the approval of such amendment or
Assembly by a vote of 147 to 5 with 1 abstention; and revision." 21 The three resolutions were approved by
Resolution No. 3 on the amendment to the Article on the Interim Batasang Pambansa sitting as a
the Commission on Elections by a vote of 148 to 2 constituent assembly on February 5 and 27, 1981. In
with 1 abstention.) The petitioners contends that such the Batasang Pambansa Blg. 22, the date of the
resolution is against the constitutions in proposing plebiscite is set for April 7, 1981. It is thus within the
amendments: 90-day period provided by the Constitution.

ISSUE: Whether the resolutions are


unconstitutional? 7. Gonzales vs COMELEC
21 SCRA 774 – Political Law – Amendment to the
HELD: In dismissing the petition for lack of merit, Constitution – Political Question vs Justiciable
the court ruled the following: Question
In June 1967, Republic Act 4913 was passed. This
1. The power of the Interim Batasang Pambansa to law provided for the COMELEC to hold a plebiscite
propose its amendments and how it may be exercised for the proposed amendments to the Constitution. It
was validly obtained. The 1973 Constitution in its was provided in the said law that the plebiscite shall
Transitory Provisions vested the Interim National be held on the same day that the general national
Assembly with the power to propose amendments elections shall be held (November 14, 1967). This
upon special call by the Prime Minister by a vote of was questioned by Ramon Gonzales and other
the majority of its members to be ratified in concerned groups as they argued that this was
accordance with the Article on Amendments similar unlawful as there would be no proper submission of
with the interim and regular national assembly. 15 the proposals to the people who would be more
When, therefore, the Interim Batasang Pambansa, interested in the issues involved in the general
upon the call of the President and Prime Minister election rather than in the issues involving the
Ferdinand E. Marcos, met as a constituent body it plebiscite.
acted by virtue of such impotence. Gonzales also questioned the validity of the
procedure adopted by Congress when they came up
2. Petitioners assailed that the resolutions where so with their proposals to amend the Constitution (RA
extensive in character as to amount to a revision 4913). In this regard, the COMELEC and other
rather than amendments. To dispose this contention, respondents interposed the defense that said act of
the court held that whether the Constitutional Congress cannot be reviewed by the courts because it
Convention will only propose amendments to the is a political question.
Constitution or entirely overhaul the present ISSUE:
Constitution and propose an entirely new I. Whether or not the act of Congress in proposing
Constitution based on an ideology foreign to the amendments is a political question.
democratic system, is of no moment, because the II. Whether or not a plebiscite may be held
same will be submitted to the people for ratification. simultaneously with a general election.
Once ratified by the sovereign people, there can be no HELD:
debate about the validity of the new Constitution. The I. No. The issue is a justiciable question. It must be
fact that the present Constitution may be revised and noted that the power to amend as well as the power to
replaced with a new one ... is no argument against the propose amendments to the Constitution is not
validity of the law because 'amendment' includes the included in the general grant of legislative powers to
'revision' or total overhaul of the entire Constitution. Congress. Such powers are not constitutionally
At any rate, whether the Constitution is merely granted to Congress. On the contrary, such powers are
amended in part or revised or totally changed would inherent to the people as repository of sovereignty in
become immaterial the moment the same is ratified a republican state. That being, when Congress makes
by the sovereign people." amendments or proposes amendments, it is not
actually doing so as Congress; but rather, it is sitting
3. That leaves only the questions of the vote necessary as a constituent assembly. Such act is not a
to propose amendments as well as the standard for legislative act. Since it is not a legislative act, it is
proper submission. The language of the Constitution reviewable by the Supreme Court. The Supreme
supplies the answer to the above questions. The Court has the final say whether or not such act of the
Interim Batasang Pambansa, sitting as a constituent constituent assembly is within constitutional
body, can propose amendments. In that capacity, only limitations.
a majority vote is needed. It would be an indefensible II. Yes. There is no prohibition to the effect that a
proposition to assert that the three-fourth votes plebiscite must only be held on a special election. SC
required when it sits as a legislative body applies as held that there is nothing in this provision of the
well when it has been convened as the agency through [1935] Constitution to indicate that the election
which amendments could be proposed. That is not a therein referred to is a special, not a general election.
requirement as far as a constitutional convention is The circumstance that the previous amendment to the
concerned. Further, the period required by the Constitution had been submitted to the people for
Page 4 of 70
ratification in special elections merely shows that action in the last analysis is still dependent on
Congress deemed it best to do so under the Congressional action.” Bluntly stated, the right of the
circumstances then obtaining. It does not negate its people to directly propose amendments to the
authority to submit proposed amendments for Constitution through the system of inititative would
ratification in general elections. remain entombed in the cold niche of the constitution
Note: **Justice Sanchez and Justice JBL Reyes until Congress provides for its implementation. The
dissented. “Plebiscite should be scheduled on a people cannot exercise such right, though
special date so as to facilitate “Fair submission, constitutionally guaranteed, if Congress for whatever
intelligent consent or rejection”. They should be reason does not provide for its implementation.
able to compare the original proposition with the
amended proposition. ***Note that this ruling has been “reversed” on
November 20, 2006 when ten justices of the SC ruled
b. By Constitutional Convention that RA 6735 is adequate enough to enable such
initiative. HOWEVER, this was a mere minute
Tan vs Macapagal resolution which reads in part:
Ten (10) Members of the Court reiterate their
C. By People’s Initiative position, as shown by their various opinions already
10. Santiago vs Comelec given when the Decision herein was promulgated,
Amendment to the Constitution that Republic Act No. 6735 is sufficient and adequate
On 6 Dec 1996, Atty. Jesus S. Delfin filed with to amend the Constitution thru a people’s initiative.
COMELEC a “Petition to Amend the Constitution to
Lift Term Limits of elective Officials by People’s
Initiative” The COMELEC then, upon its d. By President during emergency
approval, a.) set the time and dates for signature
gathering all over the country, b.) caused the 11. SANIDAD VS COMELEC
necessary publication of the said petition in papers of 73 SCRA 333 – Political Law – Constitutional Law –
general circulation, and c.) instructed local election Amendment to the Constitution
registrars to assist petitioners and volunteers in On 2 Sept 1976, Marcos issued PD No. 991 calling
establishing signing stations. On 18 Dec 1996, MD for a national referendum on 16 Oct 1976 for the
Santiago et al filed a special civil action for Citizens Assemblies (“barangays”) to resolve, among
prohibition against the Delfin Petition. Santiago other things, the issues of martial law, the interim
argues that 1.) the constitutional provision on assembly, its replacement, the powers of such
people’s initiative to amend the constitution can only replacement, the period of its existence, the length of
be implemented by law to be passed by Congress and the period for the exercise by the President of his
no such law has yet been passed by Congress, 2.) RA present powers. Twenty days after, the President
6735 indeed provides for three systems of initiative issued another related decree, PD No. 1031,
namely, initiative on the Constitution, on statues and amending the previous PD No. 991, by declaring the
on local legislation. The two latter forms of initiative provisions of PD No. 229 providing for the manner of
were specifically provided for in Subtitles II and III voting and canvass of votes in “barangays” applicable
thereof but no provisions were specifically made for to the national referendum-plebiscite of Oct 16, 1976.
initiatives on the Constitution. This omission Quite relevantly, PD No. 1031 repealed inter alia, Sec
indicates that the matter of people’s initiative to 4, of PD No. 991. On the same date of 22 Sept 1976,
amend the Constitution was left to some future law – Marcos issued PD No. 1033, stating the questions to
as pointed out by former Senator Arturo Tolentino. he submitted to the people in the referendum-
ISSUE: Whether or not RA 6735 was intended to plebiscite on October 16, 1976. The Decree recites in
include initiative on amendments to the constitution its “whereas” clauses that the people’s continued
and if so whether the act, as worded, adequately opposition to the convening of the interim National
covers such initiative. Assembly evinces their desire to have such body
HELD: RA 6735 is intended to include the system of abolished and replaced thru a constitutional
initiative on amendments to the constitution but is amendment, providing for a new interim legislative
unfortunately inadequate to cover that system. Sec 2 body, which will be submitted directly to the people
of Article 17 of the Constitution provides: in the referendum-plebiscite of October 16.
“Amendments to this constitution may likewise be On September 27, 1976, Sanidad filed a Prohibition
directly proposed by the people through initiative with Preliminary Injunction seeking to enjoin the
upon a petition of at least twelve per centum of the Commission on Elections from holding and
total number of registered voters, of which every conducting the Referendum Plebiscite on October 16;
legislative district must be represented by at least to declare without force and effect Presidential
there per centum of the registered voters therein. . . Decree Nos. 991 and 1033, insofar as they propose
The Congress shall provide for the implementation of amendments to the Constitution, as well as
the exercise of this right” This provision is obviously Presidential Decree No. 1031, insofar as it directs the
not self-executory as it needs an enabling law to be Commission on Elections to supervise, control, hold,
passed by Congress. Joaquin Bernas, a member of the and conduct the Referendum-Plebiscite scheduled on
1986 Con-Con stated “without implementing October 16, 1976.Petitioners contend that under the
legislation Section 2, Art 17 cannot operate. Thus, 1935 and 1973 Constitutions there is no grant to the
although this mode of amending the constitution is a incumbent President to exercise the constituent power
mode of amendment which bypasses Congressional to propose amendments to the new Constitution. As a
Page 5 of 70
consequence, the Referendum-Plebiscite on October
16 has no constitutional or legal basis. The Soc-Gen
contended that the question is political in nature hence 12. TOLENTINO VS COMELEC
the court cannot take cognizance of it. 41 SCRA 702 – Political Law – Amendment to the
ISSUE: Whether or not Marcos can validly propose Constitution – Doctrine of Proper Submission
amendments to the Constitution. The Constitutional Convention of 1971 scheduled an
HELD: Yes. The amending process both as to advance plebiscite concerning only the proposal to
proposal and ratification raises a judicial lower the voting age from 21 to 18. This was even
question. This is especially true in cases where the before the rest of the draft of the Constitution (then
power of the Presidency to initiate the amending under revision) had been approved. Arturo Tolentino
process by proposals of amendments, a function then filed a motion to prohibit such plebiscite.
normally exercised by the legislature, is seriously ISSUE: Whether or not the petition will prosper.
doubted. Under the terms of the 1973 Constitution, HELD: Yes. If the advance plebiscite will be
the power to propose amendments to the Constitution allowed, there will be an improper submission to the
resides in the interim National Assembly during the people. Such is not allowed.
period of transition (Sec. 15, Transitory Provisions). The proposed amendments shall be approved by a
After that period, and the regular National Assembly majority of the votes cast at an election at which the
in its active session, the power to propose amendments are submitted to the people for
amendments becomes ipso facto the prerogative of ratification. Election here is singular which meant
the regular National Assembly (Sec. 1, pars. 1 and 2 that the entire constitution must be submitted for
of Art. XVI, 1973 Constitution). The normal course ratification at one plebiscite only. Furthermore, the
has not been followed. Rather than calling the interim people were not given a proper “frame of reference”
National Assembly to constitute itself into a in arriving at their decision because they had at the
constituent assembly, the incumbent President time no idea yet of what the rest of the revised
undertook the proposal of amendments and submitted Constitution would ultimately be and therefore would
the proposed amendments thru Presidential Decree be unable to assess the proposed amendment in the
1033 to the people in a Referendum-Plebiscite on light of the entire document. This is the “Doctrine of
October 16. Unavoidably, the regularity of the Submission” which means that all the proposed
procedure for amendments, written in lambent words amendments to the Constitution shall be presented to
in the very Constitution sought to be amended, raises the people for the ratification or rejection at the same
a contestable issue. The implementing Presidential time, NOT piecemeal.
Decree Nos. 991, 1031, and 1033, which commonly
purport to have the force and effect of legislation are 13. Planas v Comelec
assailed as invalid, thus the issue of the validity of (49 SCRA 105)
said Decrees is plainly a justiciable one, within the Ponente: Concepcion, C.J.
competence of this Court to pass upon. Section 2 (2)
Article X of the new Constitution provides: “All cases Facts:
involving the constitutionality of a treaty, executive On 1 June 1971, a Constitutional Convention was
agreement, or law shall be heard and decided by the held to propose amendments to the Constitution of the
Supreme Court en banc and no treaty, executive Philippines. While the Convention is in session on 21
agreement, or law may be declared unconstitutional September 1972, the entire Philippines were placed
without the concurrence of at least ten Members. . . under martial law by virtue of Proclamation No. 1081.
..” The Supreme Court has the last word in the On 29 November 1972, the Convention approved its
construction not only of treaties and statutes, but also proposed Constitution. The next day the President
of the Constitution itself. The amending, like all issued PD No. 73 “submitting to the Filipino people
other powers organized in the Constitution, is in form for ratification or rejection the Constitution of the
a delegated and hence a limited power, so that the republic of the Philippines proposed by the 1971
Supreme Court is vested with that authority to Constitutional Convention, and appropriating funds
determine whether that power has been discharged therefor”, as well as setting the Plebiscite for said
within its limits. ratification. On 7 December 1972, Charito Planas
This petition is however dismissed. The President can petitioned the Court to enjoin respondents from
propose amendments to the Constitution and he was implementing PD No. 73 on the grounds that the
able to present those proposals to the people in calling of plebiscite and appropriation of public funds
sufficient time. The President at that time also sits as are lodged exclusively in the Congress and that there
the legislature. being no freedom of speech, press and assembly and
no sufficient time to inform the people of the contents
e. Submission of Proposals thereof, there is no proper submission to the people of
Doctrine of Proper Submission said proposed Constitution. On 17 December 1972,
Doctrine of Proper Submission the President temporarily suspended the effects of PD
1080 for purpose of free and open debate on the
proposed Constitution. On 23 December 1972, the
Amendments cannot be submitted to the people in a Plebiscite was postponed until further notice by the
piecemeal fashion wherein the other amendments are President. Thereby the Court refrained from ruling on
to follow. The people should have a frame of the cases. On 12 January 1973, petitioners filed an
reference from which to read the amendments being “urgent motion” praying for the resolution of the
proposed. (Tolentino vs COMELEC) cases “as soon as possible”.
Page 6 of 70
guidelines of national policy’. In instances where the
head of state is at the same time the president of the
political party that is in power, it does not necessarily
follow that he speaks with two voices when he
Issues: dialogues with the governed. The president is
WON the validity of PD 73 is justiciable on the accorded certain privileges that the opposition may
ground that the question at hand is political in nature. not have. Further, the SC cannot compel TV stations
WON PD 73 is valid. and radio stations to give UNIDO free air time as they
WON the Constitutional Convention have the are not party to this case. UNIDO must sought
authority to pass the proposed Constitution. contract with these TV stations and radio stations at
WON martial law affected the proper submission of their own expense.
the proposed Constitution to a plebiscite.

Held: II – RATIFICATION
Yes. The Court finds that the issue aforementioned is
a justiciable one since the assailed decree purports to 15. JAVELLANA VS. EXECUTIVE SECRETARY
have the force and effect of legislation, not only G.R. NO. 36142. March 31, 1973
because of the long list decided by the Court on the JOSUE JAVELLANA, petitioner,
acts of the Executive, but also of Subdivision (1) of vs.
Section 2, Article VIII of the 1935 Constitution. THE EXECUTIVE SECRETARY, THE
The validity of PD 73 was declared moot and SECRETARY OF NATIONAL DEFENSE, THE
academic by the Court because the plebiscite SECRETARY OF JUSTICE AND THE
ordained in said Decree has been postponed. SECRETARY OF FINANCE, respondents.
Yes. The Court held that the Constitutional
Convention was legally free to postulate any
amendment it may deem fit to propose for as long as Facts:
they adhere to Section 1 of Article XIV of the 1935 The Plebiscite Case
Constitution. On March 16, 1967, Congress of the Philippines
The issue involves question of fact which cannot be passed Resolution No. 2, which was amended by
predetermined, and that martial law per se does not Resolution No. 4 of said body, adopted on June 17,
necessarily bar the factual possibility of adequate 1969, calling a Convention to propose amendments to
freedom. the Constitution of the Philippines.
Said Resolution No. 2, as amended, was implemented
by Republic Act No. 6132, approved on August 24,
14. UNIDO vs COMELEC 1970, pursuant to the provisions of which the election
Equal Protection” – Access to Media of delegates to the said Convention was held on
In 1981, the BP proposed amendments to the 1973 November 10, 1970, and the 1971 Constitutional
Constitution. The amendments were to be placed to a Convention began to perform its functions on June 1,
plebiscite for the people’s approval. The YES vote 1971.
was being advanced by KBL – Marcos’ Party. While While the Convention was in session on September
the NO vote was being advanced by UNIDO. To 21, 1972, the President issued Proclamation No. 1081
ensure parity and equality, COMELEC issued placing the entire Philippines under Martial Law.
Resolutions 1467-1469 w/c basically provided that On November 29, 1972, the Convention approved its
there be equal opportunity, equal time and equal Proposed Constitution of the Republic of the
space on media use for campaigns for both sides. On Philippines. The next day, November 30, 1972, the
12 Mar 1981, Marcos campaigned for the YES vote President of the Philippines issued Presidential
via TV and radio from 9:30pm to 11:30pm. The same Decree No. 73, “submitting to the Filipino people for
was broadcasted live by 26 TV stations and 248 radio ratification or rejection the Constitution of the
stations nationwide. UNIDO petitioned before the Republic of the Philippines proposed by the 1971
COMELEC that they be granted the same opportunity Constitutional Convention, and appropriating funds
as Marcos has pursuant to Res’ns 1467-69. therefor,” as well as setting the plebiscite for said
COMELEC denied the demand. UNIDO assailed the ratification or rejection of the Proposed Constitution
denial as a denial of equal protection before the laws. on January 15, 1973.
ISSUE: Whether or not UNIDO was denied equal On December 7, 1972, Charito Planas filed a case
protection by virtue of COMELEC’s denial of their against the Commission on Elections, the Treasurer
request. of the Philippines and the Auditor General, to enjoin
HELD: The SC ruled that UNIDO was not denied said “respondents or their agents from implementing
due process nor were they not afforded equal Presidential Decree No. 73, in any manner, until
protection. It is the considered view of the SC that further orders of the Court,” upon the grounds, inter
when Marcos conducted his ‘pulong-pulong’ or alia, that said Presidential Decree “has no force and
consultation with the people on March 12, 1981, he effect as law because the calling … of such plebiscite,
did so in his capacity as President/Prime Minister of the setting of guidelines for the conduct of the same,
the Philippines and not as the head of any political the prescription of the ballots to be used and the
party. Under the Constitution, the ‘Prime Minister question to be answered by the voters, and the
and the Cabinet shall be responsible . . . for the appropriation of public funds for the purpose, are, by
program of government and shall determine the the Constitution, lodged exclusively in Congress …,”
Page 7 of 70
and “there is no proper submission to the people of met during the period comprised between January 10
said Proposed Constitution set for January 15, 1973, and January 15, 1973, on the two questions quoted in
there being no freedom of speech, press and paragraph 1 of this Supplemental Urgent Motion.”
assembly, and there being no sufficient time to inform On the same date January 15, 1973 the Court passed
the people of the contents thereof.” a resolution requiring the respondents in said case
On December 17, 1972, the President had issued an G.R. No. L-35948 to file “file an answer to the said
order temporarily suspending the effects of motion not later than 4 P.M., Tuesday, January 16,
Proclamation No. 1081, for the purpose of free and 1973,” and setting the motion for hearing “on January
open debate on the Proposed Constitution. 17, 1973, at 9:30 a.m.” While the case was being
On December 23, the President announced the heard, on the date last mentioned, at noontime, the
postponement of the plebiscite for the ratification or Secretary of Justice called on the writer of this
rejection of the Proposed Constitution. No formal opinion and said that, upon instructions of the
action to this effect was taken until January 7, 1973, President, he (the Secretary of Justice) was delivering
when General Order No. 20 was issued, directing to him (the writer) a copy of Proclamation No. 1102,
“that the plebiscite scheduled to be held on January which had just been signed by the President.
15, 1978, be postponed until further notice.” Said Thereupon, the writer returned to the Session Hall and
General Order No. 20, moreover, “suspended in the announced to the Court, the parties in G.R. No. L-
meantime” the “order of December 17, 1972, 35948 inasmuch as the hearing in connection
temporarily suspending the effects of Proclamation therewith was still going on and the public there
No. 1081 for purposes of free and open debate on the present that the President had, according to
proposed Constitution.” information conveyed by the Secretary of Justice,
Because of these events relative to the postponement signed said Proclamation No. 1102, earlier that
of the aforementioned plebiscite, the Court deemed it morning.
fit to refrain, for the time being, from deciding the The Ratification Case
aforementioned cases, for neither the date nor the On January 20, 1973, just two days before the
conditions under which said plebiscite would be held Supreme Court decided the sequel of plebiscite cases,
were known or announced officially. Then, again, Javellana filed this suit against the respondents to
Congress was, pursuant to the 1935 Constitution, restrain them from implementing any of the
scheduled to meet in regular session on January 22, provisions of the proposed Constitution not found in
1973, and since the main objection to Presidential the present 1935 Constitution. This is a petition filed
Decree No. 73 was that the President does not have by him as a Filipino citizen and a qualified and
the legislative authority to call a plebiscite and registered voter and as a class suit, for himself and in
appropriate funds therefor, which Congress behalf of all citizens and voters similarly situated.
unquestionably could do, particularly in view of the Javellana also alleged that the President had
formal postponement of the plebiscite by the announced the immediate implementation of the new
President reportedly after consultation with, among constitution, thru his Cabinet, respondents including.
others, the leaders of Congress and the Commission Respondents are acting without or in excess of
on Elections the Court deemed it more imperative to jurisdiction in implementing the said proposed
defer its final action on these cases. constitution upon ground that the President as
“In the afternoon of January 12, 1973, the petitioners Commander-in-Chief of the AFP is without authority
in Case G.R. No. 
L-35948 filed an “urgent to create the Citizens Assemblies; without power to
motion,” praying that said case be decided “as soon approve proposed constitution; without power to
as possible, preferably not later than January 15, proclaim the ratification by the Filipino people of the
1973.” proposed constitution; and the election held to ratify
The next day, January 13, 1973, which was a the proposed constitution was not a free election,
Saturday, the Court issued a resolution requiring the hence null and void.
respondents in said three (3) cases to comment on said Following that, petitioners prayed for the nullification
“urgent motion” and “manifestation,” “not later than of Proclamation No. 1102 and any order, decree, and
Tuesday noon, January 16, 1973.” Prior thereto, or on proclamation which have the same import and
January 15, 1973, shortly before noon, the petitioners objective.
in said Case G.R. No. L-35948 riled a “supplemental
motion for issuance of restraining order and inclusion Issues:
of additional respondents,” praying: “… that a Whether or not the issue of the validity of
restraining order be issued enjoining and restraining Proclamation No. 1102 is a justiciable question.
respondent Commission on Elections, as well as the Whether or not the constitution proposed by the 1971
Department of Local Governments and its head, Constitutional Convention has been ratified validly
Secretary Jose Roño; the Department of Agrarian conforming to the applicable constitutional and
Reforms and its head, Secretary Conrado Estrella; the statutory provisions.
National Ratification Coordinating Committee and its Whether or not the proposed Constitution has been
Chairman, Guillermo de Vega; their deputies, acquiesced in (with or without valid ratification) by
subordinates and substitutes, and all other officials the people.
and persons who may be assigned such task, from Whether or not the petitioners are entitled for relief.
collecting, certifying, and announcing and reporting Whether or not the proposed Constitution by the
to the President or other officials concerned, the so- 1971 Constitutional Convention in force.
called Citizens’ Assemblies referendum results
allegedly obtained when they were supposed to have
Page 8 of 70
Four (4) of its members, namely, Justices Barredo,
Rulings: Makasiar, Antonio and Esguerra hold that “the people
It is a justiciable and a non-political question. have already accepted the 1973 Constitution.”
To determine whether or not the new constitution is Two (2) members of the Court hold that there can be
in force depends upon whether or not the said new no free expression, and there has even been no
constitution has been ratified in accordance with the expression, by the people qualified to vote all over the
requirements of the 1935 Constitution. It is well Philippines, of their acceptance or repudiation of the
settled that the matter of ratification of an amendment proposed Constitution under Martial Law. Justice
to the constitution should be settled applying the Fernando states that “(I)f it is conceded that the
provisions of the constitution in force at the time of doctrine stated in some American decisions to the
the alleged ratification of the old constitution. effect that independently of the validity of the
The issue whether the new constitution proposed has ratification, a new Constitution once accepted
been ratified in accordance with the provisions of acquiesced in by the people must be accorded
Article XV of the 1935 Constitution is justiciable as recognition by the Court, I am not at this stage
jurisprudence here and in the US (from whom we prepared to state that such doctrine calls for
patterned our 1935 Constitution) shall show. application in view of the shortness of time that has
The Constitution was not validly ratified as held by elapsed and the difficulty of ascertaining what is the
six (6) members of the court. mind of the people in the absence of the freedom of
The Constitution does not allow Congress or anybody debate that is a concomitant feature of martial law.”
else to vest in those lacking the qualifications and Three (3) members of the Court express their lack of
having the disqualifications mentioned in the knowledge and/or competence to rule on the question.
Constitution the right of suffrage. Justices Makalintal and Castro are joined by Justice
The votes of persons less than 21 years of age render Teehankee in their statement that “Under a regime of
the proceedings in the Citizen’s assemblies void. martial law, with the free expression of opinions
Proceedings held in such Citizen’s Assemblies were through the usual media vehicle restricted, (they)
fundamentally irregular, in that persons lacking the have no means of knowing, to the point of judicial
qualifications prescribed in Article V Section 1 of the certainty, whether the people have accepted the
1935 Constitution were allowed to vote in said Constitution.”
Assemblies. And, since there is no means by which The Court is not prepared to concede that the acts the
the invalid votes of those less than 21 years of age can officers and offices of the Executive Department, in
be separated or segregated from those of the qualified line with Proclamation No. 1102, connote recognition
voters, the proceedings in the Citizen’s Assemblies of or acquiescence to the proposed Constitution.
must be considered null and void. A department of the Government cannot “recognize”
Viva voce voting for the ratification of the its own acts. Recognition normally connotes the
constitution is void. Article XV of the 1935 acknowledgment by a party of the acts of another.
Constitution envisages with the term “votes cast” Individual acts of recognition by members of
choices made on ballots – not orally or by raising Congress do not constitute congressional recognition,
hands – by the persons taking part in plebiscites. This unless the members have performed said acts in
is but natural and logical, for, since the early years of session duly assembled. This is a well-established
the American regime, we had adopted the Australian principle of Administrative Law and of the Law of
Ballot System, with its major characteristics, namely, Public Officers. The compliance by the people with
uniform official ballots prepared and furnished by the the orders of martial law government does not
Government and secrecy in the voting, with the constitute acquiescence to the proposed Constitution.
advantage of keeping records that permit judicial Neither does the Court prepared to declare that the
inquiry, when necessary, into the accuracy of the people’s inaction as regards Proclamation No. 1102,
election returns. and their compliance with a number of Presidential
The plebiscite on the constitution not having been orders, decrees and/or instructions, some or many of
conducted under the supervision of COMELEC is which have admittedly had salutary effects, issued
void. The point is that, such of the Barrio Assemblies subsequently thereto, amounts to a ratification,
as were held took place without the intervention of the adoption or approval of said Proclamation No. 1102.
COMELEC and without complying with the The intimidation is there, and inaction or obedience
provisions of the Election Code of 1971 or even of of the people, under these conditions, is not
those of Presidential Decree No. 73. The procedure necessarily an act of conformity or acquiescence.
therein mostly followed is such that there is no As regards the applicability to these cases of the
reasonable means of checking the accuracy of the “enrolled bill” rule, it is well to remember that the
returns filed by the officers who conducted said same refers to a document certified to the President
plebiscites. This is another patent violation of Article for his action under the Constitution by the Senate
X of the 1935 Constitution which form part of the President and the Speaker of the House of Reps, and
fundamental scheme set forth in the 1935 attested to by the respective Secretaries of both
Constitution, as amended, to insure the “free, orderly, Houses, concerning legislative measures approved by
and honest” expression of the people’s will. For this, said Houses. Whereas, Proclamation No. 1102 is an
the alleged plebiscite in the Citizen’s Assemblies is act of the President declaring the results of a
null and void, insofar as the same are claimed to have plebiscite on the proposed Constitution, an act which
ratified the revised Constitution Article X of the 1935 Constitution denies the
No majority vote has been reached by the Court. executive department of the Government.

Page 9 of 70
In all other respects and with regard to the other
respondent in said case, petitions therein should be I. THE FACTS
given due course, there being more than prima facie
showing that the proposed Constitution has not been This is a petition to nullify the sale of shares of stock
ratified in accordance with Article XV of the 1935 of Philippine Telecommunications Investment
Constitution, either strictly, substantially, or has been Corporation (PTIC) by the government of the
acquiesced in by the people or majority thereof; that Republic of the Philippines, acting through the Inter-
said proposed Constitution is not in force and effect; Agency Privatization Council (IPC), to Metro Pacific
and that the 1935 Constitution is still the Fundamental Assets Holdings, Inc. (MPAH), an affiliate of First
Law of the Land, without prejudice to the submission Pacific Company Limited (First Pacific), a Hong
of said proposed Constitution to the people at a Kong-based investment management and holding
plebiscite for its ratification or rejection in accordance company and a shareholder of the Philippine Long
with Articles V, X and XV of the 1935 Constitution Distance Telephone Company (PLDT).
and the provisions of the Revised Election Code in
force at the time of such plebiscite. The petitioner questioned the sale on the ground that
Being the vote of the majority, there is no further it also involved an indirect sale of 12 million shares
judicial obstacle to the new Constitution being (or about 6.3 percent of the outstanding common
considered in force and effect. shares) of PLDT owned by PTIC to First
Four (4) members of the Court, namely, Justices Pacific. With the this sale, First Pacific’s common
Barredo, Makasiar, Antonio and Esguerra hold that it shareholdings in PLDT increased from 30.7 percent
is in force by virtue of the people’s acceptance to 37 percent, thereby increasing the total common
thereof; 4 members of the Court, namely, Justices shareholdings of foreigners in PLDT to about
Makalintal, Castro, Fernando and Teehankee cast no 81.47%. This, according to the petitioner, violates
vote thereon on the premise stated in their votes on Section 11, Article XII of the 1987 Philippine
the third question that they could not state with Constitution which limits foreign ownership of the
judicial certainty whether the people have accepted or capital of a public utility to not more than 40%, thus:
not accepted the Constitution; and 2 members of the
Court, voted that the Constitution proposed by the Section 11. No franchise, certificate, or any other
1971 Constitutional Convention is not in force; with form of authorization for the operation of a public
the result, there are not enough votes to declare that utility shall be granted except to citizens of the
the new Constitution is not in force. Philippines or to corporations or associations
organized under the laws of the Philippines, at least
D. SELF – EXECUTING AND NON SELF sixty per centum of whose capital is owned by such
EXECUTING citizens; nor shall such franchise, certificate, or
SELF-EXECUTING and NON-SELF-EXECUTING authorization be exclusive in character or for a longer
provisions of the Constitution period than fifty years. Neither shall any such
- Are provisions of the Constitution self-executory? franchise or right be granted except under the
Or is there a need for legislation to implement these condition that it shall be subject to amendment,
provisions? alteration, or repeal by the Congress when the
A constitutional provision is self-executing when it common good so requires. The State shall encourage
can be given effect without the aid of legislation, and equity participation in public utilities by the general
there is nothing to indicate that legislation is intended public. The participation of foreign investors in the
to make it operative. For example, a constitutional governing body of any public utility enterprise shall
provision that any municipality by vote of four- be limited to their proportionate share in its capital,
sevenths of its qualified electors may issue and sell and all the executive and managing officers of such
revenue bonds in order to pay for the cost of corporation or association must be citizens of the
purchasing a municipally owned public utility is self- Philippines. (Emphasis supplied)
executing and effective without a legislative
enactment. II. THE ISSUE
Constitutional provisions are not self-executing if
they merely set forth a line of policy or principles Does the term “capital” in Section 11, Article XII of
without supplying the means by which they are to be the Constitution refer to the total common shares
effectuated, or if the language of the constitution is only, or to the total outstanding capital stock
directed to the legislature. As a result, a constitutional (combined total of common and non-voting preferred
provision that the legislature shall direct by law in shares) of PLDT, a public utility?
what manner and in what court suits may be brought
against the state is not self-executing. III. THE RULING
Just as with constitutional provisions, statutes and
court judgments can be self-executing. [The Court partly granted the petition and held that
the term “capital” in Section 11, Article XII of the
16. Wilson P. Gamboa v. Finance Secretary Constitution refers only to shares of stock entitled to
Margarito Teves, et al., G.R. No. 176579, June 28, vote in the election of directors of a public utility, i.e.,
2011 to the total common shares in PLDT.]
DECISION
Considering that common shares have voting rights
CARPIO, J.: which translate to control, as opposed to preferred
Page 10 of 70
shares which usually have no voting rights, the term common shares. Moreover, 99.44% of the preferred
“capital” in Section 11, Article XII of the Constitution shares are owned by Filipinos while foreigners own
refers only to common shares. However, if the only a minuscule 0.56% of the preferred
preferred shares also have the right to vote in the shares. Worse, preferred shares constitute 77.85% of
election of directors, then the term “capital” shall the authorized capital stock of PLDT while common
include such preferred shares because the right to shares constitute only 22.15%. This undeniably
participate in the control or management of the shows that beneficial interest in PLDT is not with the
corporation is exercised through the right to vote in non-voting preferred shares but with the common
the election of directors. In short, the term “capital” shares, blatantly violating the constitutional
in Section 11, Article XII of the Constitution refers requirement of 60 percent Filipino control and
only to shares of stock that can vote in the election of Filipino beneficial ownership in a public utility.
directors.
In short, Filipinos hold less than 60 percent of the
To construe broadly the term “capital” as the total voting stock, and earn less than 60 percent of the
outstanding capital stock, including both common dividends, of PLDT. This directly contravenes the
and non-voting preferred shares, grossly contravenes express command in Section 11, Article XII of the
the intent and letter of the Constitution that the “State Constitution that “[n]o franchise, certificate, or any
shall develop a self-reliant and independent national other form of authorization for the operation of a
economy effectively controlled by Filipinos.” A public utility shall be granted except to x x x
broad definition unjustifiably disregards who owns corporations x x x organized under the laws of the
the all-important voting stock, which necessarily Philippines, at least sixty per centum of whose capital
equates to control of the public utility. is owned by such citizens x x x.”

Holders of PLDT preferred shares are explicitly To repeat, (1) foreigners own 64.27% of the common
denied of the right to vote in the election of directors. shares of PLDT, which class of shares exercises
PLDT’s Articles of Incorporation expressly state that the sole right to vote in the election of directors, and
“the holders of Serial Preferred Stock shall not be thus exercise control over PLDT; (2) Filipinos own
entitled to vote at any meeting of the stockholders for only 35.73% of PLDT’s common shares, constituting
the election of directors or for any other purpose or a minority of the voting stock, and thus do not
otherwise participate in any action taken by the exercise control over PLDT; (3) preferred shares,
corporation or its stockholders, or to receive notice of 99.44% owned by Filipinos, have no voting rights; (4)
any meeting of stockholders.” On the other hand, preferred shares earn only 1/70 of the dividends that
holders of common shares are granted the exclusive common shares earn; (5) preferred shares have twice
right to vote in the election of directors. PLDT’s the par value of common shares; and (6) preferred
Articles of Incorporation state that “each holder of shares constitute 77.85% of the authorized capital
Common Capital Stock shall have one vote in respect stock of PLDT and common shares only 22.15%.
of each share of such stock held by him on all matters This kind of ownership and control of a public utility
voted upon by the stockholders, and the holders of is a mockery of the Constitution.
Common Capital Stock shall have the exclusive right
to vote for the election of directors and for all other [Thus, the Respondent Chairperson of the Securities
purposes.” and Exchange Commission was DIRECTED by the
Court to apply the foregoing definition of the term
It must be stressed, and respondents do not dispute, “capital” in determining the extent of allowable
that foreigners hold a majority of the common shares foreign ownership in respondent Philippine Long
of PLDT. In fact, based on PLDT’s 2010 General Distance Telephone Company, and if there is a
Information Sheet (GIS), which is a document violation of Section 11, Article XII of the
required to be submitted annually to the Securities Constitution, to impose the appropriate sanctions
and Exchange Commission, foreigners hold under the law.]
120,046,690 common shares of PLDT whereas
Filipinos hold only 66,750,622 common shares. In
other words, foreigners hold 64.27% of the total E. GEN. PROVISIONS
number of PLDT’s common shares, while Filipinos 17. ROQUE R. MARTINEZ, A.M. No. P-04-1795
hold only 35.73%. Since holding a majority of the vs LIM
common shares equates to control, it is clear that March 25, 2009
foreigners exercise control over PLDT. Such amount
of control unmistakably exceeds the allowable 40 RESOLUTION
percent limit on foreign ownership of public utilities CORONA, J.:
expressly mandated in Section 11, Article XII of the This complaint involves two interrelated
Constitution. administrative charges against respondent Norvell R.
Lim, Sheriff III of the Regional Trial Court of
As shown in PLDT’s 2010 GIS, as submitted to the Romblon, Romblon, Branch 81.
SEC, the par value of PLDT common shares is P5.00 On March 11, 2002, respondent sent a letter to
per share, whereas the par value of preferred shares Arsenio
is P10.00 per share. In other words, preferred shares R.M. Almaddin, officer-in-charge of the Office of the
have twice the par value of common shares but cannot Provincial Prosecutor (OPP) of Romblon stating:
elect directors and have only 1/70 of the dividends of
Page 11 of 70
I wish to inform you that today, Monday, March 11, lack of probable cause the complaint for violation of
2002, at 8 a.m., and for the month of March 2002, [it] PD 26.[5]
is the turn of the [OPP] to lead the flag ceremony.
But the Ombudsman referred the administrative
However, this morning, this was not done because aspect of the complaints against respondent to the
none of the personnel of your office was present. Office of the Court Administrator (OCA).[6]
With regard to the complaint for grave misconduct,
We hope that we would be able to look forward to the OCA found that respondent bore no malice when
seeing all the personnel of [the OPP] in the Hall of he sent the March 11, 2002 letter. It noted:
Justice, Romblon, Romblon, participate in [the flag There is nothing in the letter that is suggestive of
ceremony] every Monday morning and Friday complainants lack of patriotism as to impute bad faith
afternoon.[1] on the part of respondent. Respondent was merely
expressing his concern so that any similar incident
may not happen again mindful of everyones bounden
On May 16, 2002 complainants Roque R. Martinez, duty to express and manifest their patriotism and love
Maria Elena M. Felipe, Robert R. Miano, Rosalinda of country and respect for the flag.
G. Macasa and Ciriaco D. Mariveles, Jr., all
employees of the OPP, filed an administrative
complaint for grave misconduct against respondent in Thus, it recommended the dismissal of the complaint
the Office of the Ombudsman.[2] They asserted that for lack of merit.
respondents March 11, 2002 letter portrayed them as
unpatriotic Filipinos, tarnished their reputation as With regard to the complaint for violation of PD 26,
public officers and cast dishonor, disrepute and the OCA found that respondent mailed his counter-
contempt on their persons. affidavit in the previous complaint (for grave
misconduct) using envelopes intended for free
Respondent explained that, in the absence of the postage. Inasmuch as the mailed matter was not an
presiding judge, he was the administrative officer-in- official communication related to the conduct of
charge of the Hall of Justice. As such, it was his duty judicial proceedings, respondent was guilty of
to require complainants to attend the flag ceremony. violating the law. Hence, it recommended that
Thus, he wrote Almaddin to remind him that the OPP complainant be fined P1,000.
had been assigned to lead the flag ceremony for the
month of March 2002 and to inform him that no one We adopt the findings of the OCA with a
from his office attended the ceremony that morning. modification of the penalty.
Respondent denied ill-will against complainants.
Misconduct implies wrongful intention and not a
Subsequently, complainants filed another complaint mere error of judgment; an act that is corrupt or
against respondent charging him of violation of inspired by an intention to violate the law or a
PD[3] 26[4] which provides: persistent disregard of well-known legal rules.[7]

(1) Judges of the Courts of First Instance, Flag ceremonies inspire patriotism and evoke the
Circuit Criminal Courts, Juvenile and Domestic finest sentiments of love of country and
Relations Courts, Courts of Agrarian Relations, Court people.[8] Section 18 of RA[9] 8491 provides:
of Industrial Relations, Military Tribunals and City
and Municipal Courts, may transmit in the mail, free Section 18. All government offices and educational
of charge, all official communications and papers institutions shall henceforth observe the flag-raising
directly connected with the conduct of judicial ceremony every Monday morning and the flag
proceedings. lowering ceremony every Friday afternoon. The
ceremony shall be simple and dignified and shall
(2) The envelope or wrapper of the include the playing or singing of the Philippine
privileged mail matter shall bear on the left upper National Anthem.
corner the name, official designation and station of
the official sending such mail matter and on the right
upper corner, the words: "Private or unauthorized use Pursuant to this mandate, Supreme Court Circular No.
to avoid payment of postage is penalized by fine or 62-2001 (dated September 21, 2001) provides:
imprisonment or both." (emphasis supplied) All Executive Judges shall supervise the holding of
the flag raising and flag lowering ceremonies in their
respective Hall of Justice buildings or courthouses
and shall ensure the attendance of all judges and court
Complainants stated that respondent did not pay for personnel in the rites.
postage stamps when he mailed copies of his counter-
affidavit to them. Since the mailed matter neither
involved a court process nor was in any way In deference to these mandates, the Chief State
connected to the conduct of judicial proceedings, he Prosecutor directed the personnel of the OPP to attend
was guilty of violating the said decree. the flag ceremony.[10]
Respondent asserted that the allegations against him
were baseless. In fact, the Ombudsman dismissed for
Page 12 of 70
Consequently, as administrative officer-in-charge of
the Hall of Justice of Romblon, respondent was duty- MAGDALO filed a Motion for Reconsideration,
bound to remind the employees to attend the flag which was elevated to the COMELEC En Banc for
ceremony. Furthermore, the March 11, 2002 letter resolution. MAGDALO also filed a Manifestation of
(quoted above) was courteously written. Respondent Intent to Participate in the Party-List System of
neither used offensive language nor insinuated that Representation in the 10 May 2010 Elections, in
complainants were unpatriotic. Thus, there was no which it stated that its membership includes former
misconduct on the part of respondent. members of the AFP, Anti-Corruption Advocates,
Reform-minded citizens. They filed an Amended
Nonetheless, we agree that respondent violated PD Manifestation, and in which they manifest that the
26. In Bernadez v. Montejar,[11] we held that the instant MANIFESTATION is being filed ex abutanti
franking privilege granted by PD 26 extended only to (sic) cautelam (out of the abundance of caution) only
judges and referred to official communications and and subject to the outcome of the resolution of the
papers directly connected with the conduct of judicial Motion for Reconsideration that is still pending. It is
proceedings.[12] Respondent was not a judge nor was not in any way intended to preempt the ruling of the
the mailed matter related to the discharge of judicial Commission but merely to preserve the possibility of
functions. Thus, respondent violated PD 26 for which pursuing the Partys participation in the Party-List
a fine of P500 should be imposed on him. System of Representation in the eventuality that their
Considering that respondent compulsorily retired on Petition is approved.
September 7, 2003, the fine of P500 shall be deducted
from his retirement benefits. The COMELEC En Banc denied the Motion for
Reconsideration. In the instant Petition, MAGDALO
WHEREFORE, the complaint for grave misconduct argues that the findings of the assailed resolutions on
against Sheriff Norvell R. Lim is hereby dismissed for the basis of which the Petition was denied are based
lack of merit. But he is found guilty of violating on pure speculation. The assailed Resolutions
Presidential Decree No. 26 and is hereby fined P500 effectively preempted the court trying the case. The
which shall be deducted from his retirement benefits. subject Resolutions unfairly jumped to the conclusion
that the founders of the Magdalo committed mutiny,
SO ORDERED. held innocent civilian personnel as hostage, employed
violence and used unlawful means and in the process
defied the laws of organized society purportedly
during the Oakwood incident when even the court
18. Magdalo v. COMELEC, G.R. No. 190793, trying their case, (RTC Makati) has not yet decided
June 19, 2012 the case against them; and the Resolution violates the
TOPIC: Composition of Congress, Qualifications constitutional presumption of innocence in favor of
of Members, and Term of Office founders of the Magdalo and their basic right of to
due process of law.
DOCTRINE: The registration of political parties
does not involve administrative liability as it is On the other hand, the COMELEC asserts that it had
only limited to the evaluation of qualifications for the power to ascertain the eligibility of MAGDALO
registration. for registration and accreditation as a political party.
It contends that this determination, as well as that of
FACTS: assessing whether MAGDALO advocates the use of
On 2 July 2009, Petitioner Magdalo sa Pagbabago force, would entail the evaluation of evidence, which
(MAGDALO) filed its Petition for Registration with cannot be reviewed by this Court in a petition for
the COMELEC, seeking its registration and/or certiorari.
accreditation as a regional political party based in the
NCR for participation in the 10 May 2010 National However, MAGDALO maintains that although it
and Local Elections. In the Petition, MAGDALO was concedes that the COMELEC has the authority to
represented by its Chairperson, Senator Antonio F. assess whether parties applying for registration
Trillanes IV, and its Secretary General, Francisco possess all the qualifications and none of the
Ashley L. Acedillo (Acedillo). disqualifications under the applicable law, the latter
nevertheless committed grave abuse of discretion in
On 26 October 2009, the COMELEC denied the basing its determination on pure conjectures instead
Petition for Registration as it was not in accordance of on the evidence on record.
with Art. IX-C, Section 2(5) of the Constitution. It is
common knowledge that the party’s organizer and ISSUE:
Chairman, and some members participated in the Whether the COMELEC gravely abused its discretion
take-over of the Oakwood Premier Apartments in when it denied the Petition for Registration filed by
Ayala Center, Makati City on July 27, 2003, wherein MAGDALO on the ground that the latter seeks to
several innocent civilian personnel were held hostage. achieve its goals through violent or unlawful means.
This and the fact that they were in full battle gear at
the time of the mutiny clearly show their purpose in HELD: NO
employing violence and using unlawful means to This Court rules in the negative, but without prejudice
achieve their goals in the process defying the laws of to MAGDALOs filing anew of a Petition for
organized societies. Registration. The COMELEC has a constitutional and
Page 13 of 70
statutory mandate to ascertain the eligibility of parties that respondent did not commit grave abuse of
and organizations to participate in electoral contests. discretion.
The relevant portions of the 1987 Constitution read:
A. The COMELEC did not commit grave abuse
ARTICLE VI LEGISLATIVE DEPARTMENT of discretion in taking judicial notice of the
Section 5. (1) The House of Representatives shall be Oakwood incident.
composed of not more than two hundred and fifty MAGDALO contends that it was grave abuse of
members, unless otherwise fixed by law, who shall be discretion for the COMELEC to have denied the
elected from legislative districts apportioned among Petition for Registration not on the basis of facts or
the provinces, cities, and the Metropolitan Manila evidence on record, but on mere speculation and
area in accordance with the number of their respective conjectures. This argument cannot be given any
inhabitants, and on the basis of a uniform and merit. Under the Rules of Court, judicial notice may
progressive ratio, and those who, as provided by law, be taken of matters that are of public knowledge, or
shall be elected through a party-list system of are capable of unquestionable demonstration. Further,
registered national, regional, and sectoral parties or Executive Order No. 292, otherwise known as the
organizations. Revised Administrative Code, specifically empowers
xxxxxxxxx administrative agencies to admit and give probative
value to evidence commonly acceptable by
ARTICLE IX CONSTITUTIONAL reasonably prudent men, and to take notice of
COMMISSIONS judicially cognizable facts. Thus, in Saludo v.
C. The Commission on Elections American Express,this Court explained as follows:
Section 2. The Commission on Elections shall
exercise the following powers and functions: The concept of facts of common knowledge in the
xxxxxxxxx context of judicial notice has been explained as those
facts that are so commonly known in the community
(5) Register, after sufficient publication, political as to make it unprofitable to require proof, and so
parties, organizations, or coalitions which, in addition certainly known x x x as to make it indisputable
to other requirements, must present their platform or among reasonable men.
program of government; and accredit citizens arms of The Oakwood incident was widely known and
the Commission on Elections. Religious extensively covered by the media made it a proper
denominations and sects shall not be subject of judicial notice. Thus, the COMELEC did
registered. Those which seek to achieve their goals not commit grave abuse of discretion when it treated
through violence or unlawful means, or refuse to these facts as public knowledge, and took cognizance
uphold and adhere to this Constitution, or which thereof without requiring the introduction and
are supported by any foreign government shall reception of evidence thereon.
likewise be refused registration. x x x.
B. The COMELEC did not commit grave abuse of
RA No. 7941, otherwise known as the Party-List discretion in finding that MAGDALO uses
System Act, reads in part: violence or unlawful means to achieve its goals.
In the instant Petition, MAGDALO claims that it did
Thus, to join electoral contests, a party or not resort to violence when it took over Oakwood
organization must undergo the two-step process of because (a) no one, either civilian or military, was
registration and accreditation, as this Court explained held hostage; (b) its members immediately evacuated
in Liberal Party v. COMELEC: the guests and staff of the hotel; and (c) not a single
x x x Registration is the act that bestows juridical shot was fired during the incident.
personality for purposes of our election laws;
accreditation, on the other hand, relates to Under Article IX-C, Section 2(5) of the 1987
the privileged participation that our election laws Constitution, parties, organizations and coalitions that
grant to qualified registered parties. seek to achieve their goals through violence or
unlawful means shall be denied registration. This
x x x Accreditation can only be granted to a registered disqualification is reiterated in Section 61 of B.P. 881,
political party, organization or coalition; stated which provides that no political party which seeks to
otherwise, a registration must first take place before a achieve its goal through violence shall be entitled to
request for accreditation can be made. Once accreditation.
registration has been carried out, accreditation is the
next natural step to follow. Violence is the unjust or unwarranted exercise of
force, usually with the accompaniment of vehemence,
Considering the constitutional and statutory authority outrage or fury. It also denotes physical force
of the COMELEC to ascertain the eligibility of parties unlawfully exercised; abuse of force; that force which
or organizations seeking registration and is employed against common right, against the laws,
accreditation, the pertinent question now is whether and against public liberty. The Oakwood incident
its exercise of this discretion was so capricious or was one that was attended with violence. As publicly
whimsical as to amount to lack of jurisdiction. In announced by the leaders of MAGDALO during the
view of the facts available to the COMELEC at the siege, their objectives were to express their
time it issued its assailed Resolutions, this Court rules dissatisfaction with the administration of former
President Arroyo, and to divulge the alleged
Page 14 of 70
corruption in the military and the supposed sale of finding of liability is that the respondents have
arms to enemies of the state. Ultimately, they wanted been found to be administratively guilty by
the President, her cabinet members, and the top substantial evidence the quantum of proof
officials of the AFP and the PNP to resign. To achieve required in an administrative proceeding. The
these goals, MAGDALO opted to seize a hotel requirement of the Revised Rules of Criminal
occupied by civilians, march in the premises in full Procedure that the proposed witness should not
battle gear with ammunitions, and plant explosives in appear to be the most guilty is obviously in line with
the building. These brash methods by which the character and purpose of a criminal proceeding,
MAGDALO opted to ventilate the grievances of its and the much stricter standards observed in these
members and withdraw its support from the cases. They are standards entirely different from
government constituted clear acts of violence. those applicable in administrative proceedings.

The assertions of MAGDALO that no one was held Further, there is a well-established distinction
hostage or that no shot was fired do not mask its use between the quantum of proof required for
of impelling force to take over and sustain the administrative proceedings and that for criminal
occupation of Oakwood. Neither does its express actions, to wit:
renunciation of the use of force, violence and other As an administrative proceeding, the evidentiary bar
unlawful means in its Petition for Registration and against which the evidence at hand is measured is not
Program of Government obscure the actual the highest quantum of proof beyond reasonable
circumstances surrounding the encounter. The doubt, requiring moral certainty to support
deliberate brandishing of military power, which affirmative findings. Instead, the lowest standard of
included the show of force, use of full battle gear, substantial evidence, that is, such relevant evidence
display of ammunitions, and use of explosive devices, as a reasonable mind will accept as adequate to
engendered an alarming security risk to the public. At support a conclusion, applies.
the very least, the totality of these brazen acts
fomented a threat of violence that preyed on the In the case at bar, the challenged COMELEC
vulnerability of civilians. The COMELEC did not, Resolutions were issued pursuant to its administrative
therefore, commit grave abuse of discretion when it power to evaluate the eligibility of groups to join the
treated the Oakwood standoff as a manifestation of elections as political parties, for which the evidentiary
the predilection of MAGDALO for resorting to threshold of substantial evidence is applicable. In
violence or threats thereof in order to achieve its finding that MAGDALO resorts to violence or
objectives. unlawful acts to fulfill its organizational objectives,
the COMELEC did not render an assessment as to
C. The finding that MAGDALO seeks to achieve whether the members of petitioner committed crimes,
its goals through violence or unlawful means did as respondent was not required to make that
not operate as a prejudgment of Criminal Case determination in the first place. Its evaluation was
No. 03-2784. limited only to examining whether MAGDALO
MAGDALO contends that the finding of the possessed all the necessary qualifications and none of
COMELEC that the former pursues its goals through disqualifications for registration as a political party.
violence or unlawful means was tantamount to an In arriving at its assailed ruling, the COMELEC only
unwarranted verdict of guilt for several crimes, which had to assess whether there was substantial evidence
in effect, preempted the proceedings in Criminal Case adequate to support this conclusion.
No. 03-2784 and violated the right to presumption of
innocence. This argument cannot be sustained. On the other hand, Criminal Case No. 03-2784 is a
criminal action charging members of MAGDALO
The power vested by Article IX-C, Section 2(5) of the with coup dtat following the events that took place
Constitution and Section 61 of BP 881 in the during the Oakwood siege. As it is a criminal case,
COMELEC to register political parties and ascertain proof beyond reasonable doubt is necessary.
the eligibility of groups to participate in the elections Therefore, although the registration case before the
is purely administrative in character. In exercising COMELEC and the criminal case before the trial
this authority, the COMELEC only has to assess court may find bases in the same factual
whether the party or organization seeking circumstances, they nevertheless involve entirely
registration or accreditation pursues its goals by separate and distinct issues requiring different
employing acts considered as violent or unlawful, evidentiary thresholds.
and not necessarily criminal in nature. Although
this process does not entail any determination of
administrative liability, as it is only limited to the This Court finds that the COMELEC did not commit
evaluation of qualifications for registration, the grave abuse of discretion in denying the Petition for
ruling of this Court in Quarto v. Marcelo is Registration filed by MAGDALO. However, in view
nonetheless analogously applicable: of the subsequent amnesty granted in favor of the
members of MAGDALO, the events that transpired
An administrative case is altogether different from a during the Oakwood incident can no longer be
criminal case, such that the disposition in the former interpreted as acts of violence in the context of the
does not necessarily result in the same disposition for disqualifications from party registration.
the latter, although both may arise from the same set
of facts. The most that we can read from the *Issue on Mootness
Page 15 of 70
whether this case has been rendered moot and RULING
academic by the conduct of the 10 May 2010 National NO.
and Local Elections. Although the subject Petition for The writ of habeas data is an independent and
Registration filed by MAGDALO was intended for summary remedy designed to protect the image,
the elections on even date, it specifically asked for privacy, honor, information, and freedom of
accreditation as a regional political party for purposes information of an individual, and to provide a forum
of subsequent elections. to enforce one’s right to the truth and to informational
xxxxxx privacy. It seeks to protect a person’s right to control
The moot and academic principle is not a magical information regarding oneself, particularly in
formula that can automatically dissuade the courts in instances in which such information is being collected
resolving a case. Courts will decide cases, otherwise through unlawful means in order to achieve unlawful
moot and academic, if: first, there is a grave violation ends. It must be emphasized that in order for the
of the Constitution; second, the exceptional character privilege of the writ to be granted, there must exist a
of the situation and the paramount public interest is nexus between the right to privacy on the one hand,
involved; third, when [the] constitutional issue raised and the right to life, liberty or security on the other.
requires formulation of controlling principles to guide In this case, the Court ruled that Gamboa was unable
the bench, the bar, and the public; and fourth, the case to prove through substantial evidence that her
is capable of repetition yet evading review. inclusion in the list of individuals maintaining PAGs
made her and her supporters susceptible to
The second and fourth exceptions are clearly present harassment and to increased police surveillance. In
in the case at bar. The instant action brings to the fore this regard, respondents sufficiently explained that
matters of public concern, as it challenges the very the investigations conducted against her were in
notion of the use of violence or unlawful means as a relation to the criminal cases in which she was
ground for disqualification from party registration. implicated. As public officials, they enjoy the
Moreover, considering the expressed intention of presumption of regularity, which she failed to
MAGDALO to join subsequent elections, as well as overcome. [T]he state interest of dismantling PAGs
the occurrence of supervening events pertinent to the far outweighs the alleged intrusion on the private life
case at bar, it remains prudent to examine the issues of Gamboa, especially when the collection and
raised and resolve the arising legal questions once and forwarding by the PNP of information against her was
for all. pursuant to a lawful mandate. Therefore, the privilege
of the writ of habeas data must be denied.

DISPOSITIVE PORTION: 21. Animos vs. PVAO


G.R. No. 79156, June 22, 1989
WHEREFORE, the instant Petition is DISMISSED.
The 26 October 2009 and 4 January 2010 Resolutions FACTS: Isidro Animos is a World War II veteran,
of the Commission on Elections are hereby having been a member of the USAFFE and the
AFFIRMED, without prejudice to the filing anew of guerilla forces thereafter. Originally, the case was a
a Petition for Registration by MAGDALO. suit for mandamus by the petitioners against PVAO,
for the payment of full pension benefits, retroactive to
1947, under Republic Act No. 65, as amended.
However, the petitioner’s claim was denied on the
18 – NA basis that Animos’ disability was only considered
19. Gamboa v. Chan, G.R. No. 193636, 24 partial, rather than total, according to the “Rules on
July 2012 Disability Ratings”, thus precluding the maximum
31JUL payment of his pension benefits. The petitioner
FACTS submits that the rating system adopted by PVAO is
Gamboa alleged that the Philippine National Police in null and void.
Ilocos Norte (PNP–Ilocos Norte) conducted a series
of surveillance operations against her and her ISSUE: Whether or not the complaint against PVAO
aides, and classified her as someone who keeps a can be considered a suit against the state.
Private Army Group (PAG). Purportedly without the
benefit of data verification, PNP–Ilocos Norte HELD: No. The doctrine of immunity from the suit
forwarded the information gathered on her to the will not apply and may not be invoked where the
Zeñarosa Commission, thereby causing her inclusion public official is being sued in his private and
in the Report’s enumeration of individuals personal capacity as an ordinary citizen. When
maintaining PAGs. Contending that her right to officers and agents of the government are sued in their
privacy was violated and her reputation maligned and individual capacity, the cloak of protection from the
destroyed, Gamboa filed a Petition for the issuance of government is removed. According to the doctrine in
a writ of habeas data against respondents in their Ruiz vs. Cabahug: “We hold that under the facts and
capacities as officials of the PNP-Ilocos Norte. circumstances alleged in the amended complaint,
ISSUE which should be taken on its face value, the suit is not
Whether or not the petition for the issuance of writ of one against the Government, or a claim against it, but
habeas data is proper when the right to privacy is one against the officials to compel them to act in
invoked as opposed to the state’s interest in accordance with the rights to be established by the
preserving the right to life, liberty or security. contending architects, or to prevent them from
Page 16 of 70
making payment and recognition until the contending
architects have established their respective rights and (2) Yes, the mere press statements of respondents
interests in the funds retained and in the credit for the DOJ Secretary and the NTC constituted a form of
work done”. Hence, the complaint cannot be content-based prior restraint that has transgressed the
considered a suit against the state because it is a well- Constitution. It is not decisive that the press
settled principle of law that we may consider a public statements made by respondents were not reduced in
official liable in his personal private capacity for the or followed up with formal orders or circulars. It is
damage caused by his acts when done with malice and sufficient that the press statements were made by
in bad faith, or beyond the scope of his authority and respondents while in the exercise of their official
jurisdiction. functions. Any act done, such as a speech uttered, for
and on behalf of the government in an official
capacity is covered by the rule on prior restraint. The
22. Francisco Chavez vs. Raul M. Gonzales and concept of an “act” does not limit itself to acts already
NTC | G.R. No. 168338 | February 15, 2008 converted to a formal order or official
circular. Otherwise, the non formalization of an act
Facts: As a consequence of the public release of into an official order or circular will result in the easy
copies of the “Hello Garci” compact disc audiotapes circumvention of the prohibition on prior restraint.
involving a wiretapped mobile phone conversation
between then-President Gloria Arroyo and Comelec AZCUNA CONCURRING OPINION
Commissioner Virgilio Garcillano, respondent DOJ SEPARATE CONCURRING OPINION
Secretary Gonzales warned reporters that those who
had copies of the CD and those broadcasting or
publishing its contents could be held liable under the AZCUNA, J.:
Anti-Wiretapping Act. He also stated that persons
possessing or airing said tapes were committing a
continuing offense, subject to arrest by anybody. I vote to GRANT the petition on the ground that the
Finally, he stated that he had ordered the NBI to go challenged NTC and DOJ warnings violate Sec. 10,
after media organizations “found to have caused the Art. XVI of the Constitution which states:
spread, the playing and the printing of the contents of
a tape.” Meanwhile, respondent NTC warned TV and Sec. 10. The State shall provide the policy
radio stations that their broadcast/airing of such false environment for the full development of Filipino
information and/or willful misrepresentation shall be capability and the emergency of communication
a just cause for the suspension, revocation and/or structures suitable to the needs and aspirations of the
cancellation of the licenses or authorizations issued to nation and the balanced flow of information into, out
the said media establishments. Petitioner Chavez of, and across the country, in accordance with a policy
filed a petition under Rule 65 against respondents that respects the freedom of speech and of the press.
Secretary Gonzales and the NTC directly with the
Supreme Court.
This provision was precisely crafted to meet the needs
Issues: (1) Will a purported violation of law such as and opportunities of the emerging new pathways of
the Anti-Wiretapping Law justify straitjacketing the communications, from radio and tv broadcast to the
exercise of freedom of speech and of the press? flow of digital information via cables, satellites and
(2) Did the mere press statements of respondents the internet.
DOJ Secretary and the NTC constitute a form of
content-based prior restraint that has transgressed the The purpose of this new statement of directed State
Constitution? policy is to hold the State responsible for a policy
environment that provides for (1) the full
Held: (1) No, a purported violation of law such as the development of Filipino capability, (2) the emergence
Anti-Wiretapping Law will not justify of communication structures suitable to the needs and
straitjacketing the exercise of freedom of speech and aspirations of the nation and the balanced flow of
of the press. A governmental action that restricts information, and (3) respect for the freedom of speech
freedom of speech or of the press based on content is and of the press.
given the strictest scrutiny, with
the government having the burden of overcoming the The regulatory warnings involved in this case work
presumed unconstitutionality by the clear and present against a balanced flow of information in our
danger rule. This rule applies equally to all kinds of communication structures and do so without
media, including broadcast media. Respondents, who respecting freedom of speech by casting a chilling
have the burden to show that these acts do not abridge effect on the media. This is definitely not the policy
freedom of speech and of the press, failed to hurdle environment contemplated by the Constitution.
the clear and present danger test. For this failure of
the respondents alone to offer proof to satisfy the
clear and present danger test, the Court has no option ADOLFO S. AZCUNA
but to uphold the exercise of free speech and free Associate Justice
press. There is no showing that the feared violation of
the anti-wiretapping law clearly endangers
the national security of the State. 23. Executive Order No. 185, s. 2015
Page 17 of 70
23. EXECUTIVE ORDER NO. 185 SECTION 2. Levy on Articles. Upon the effectivity
MODIFYING THE NOMENCLATURE AND of this Order, all articles which are specifically listed
RATES OF DUTY ON CERTAIN IMPORTED in the aforesaid Annex and are entered into, or
ARTICLES AS PROVIDED FOR UNDER THE withdrawn from warehouses in the Philippines for
TARIFF AND CUSTOMS CODE OF THE consumption shall be levied the MFN rates of duty as
PHILIPPINES, AS AMENDED, IN ORDER TO therein prescribed.
IMPLEMENT THE PHILIPPINE TARIFF SECTION 3. Repeal. All issuances, orders, rules,
COMMITMENTS ON CERTAIN PRODUCTS and regulations, or parts thereof, which are
INCLUDED IN THE ENVIRONMENTAL GOODS inconsistent with any of the provisions of this Order
LIST UNDER THE ASIA-PACIFIC are hereby repealed or modified accordingly.
ECONOMIC COOPERATION SECTION 4. Separability. Should any provision of
WHEREAS, under the 2007 Sydney and 2011 this Order be declared invalid or unconstitutional, the
Honolulu Declarations, the Asia-Pacific Economic other provisions unaffected thereby shall remain valid
Cooperation (APEC) leaders made a commitment to and subsisting.
address issues on climate change, review the progress SECTION 5. Effectivity. This Order shall take effect
of the World Trade Organization (WTO) Doha immediately following its complete publication in the
Development Agenda negotiations on the Official Gazette or in a newspaper of general
liberalization of trade in environmental goods and circulation.
services, and take concrete steps to achieve past DONE, in the City of Manila, this 26th day of June,
ambitions and make green growth a reality, consistent in the year of Our Lord, Two Thousand and Fifteen.
with their WTO obligations;
WHEREAS, in its 2009 Annual Report, the APEC II – GENERAL CONSIDERATIONS
Committee on Trade and Investment endorsed the
APEC Environmental Goods and Services (EGS)
Work Program to help in reaching an agreement on 1. DOCTRINE OF CONSTITUTIONAL
actions to support sustainable growth in the region, SUPREMACY
advance work to increase utilization and
dissemination of EGS, reduce existing barriers and 24. MANILA PRINCE HOTEL vs GSIS (1997)
refrain from introducing new barriers to trade and 267 SCRA 408 – Political Law – Constitutional Law
investment in EGS, and enhance the capabilities of – Supremacy of the Constitution – Filipino First
economies to develop their EGS sectors; Policy
WHEREAS, under the 2012 Vladivostok Self-Executing Provisions of the Constitution – Par.
Declaration, the APEC leaders endorsed the APEC 2, Sec. 10, Art. XII
List of Environmental Goods enumerating 54 Pursuant to the privatization program of the
environmental goods that “directly and positively government, the Government Service Insurance
contribute to green growth and sustainable System (GSIS) decided to sell 30-51% of the Manila
development objectives,” with a commitment to Hotel Corporation. Two bidders participated, Manila
reduce applied rates to 5% or less on these Prince Hotel (MPH) and the Malaysian Firm Renong
environmental goods by the end of 2015, taking into Berhad (RB). MPH’s bid was at P41.58/per share
account “economies’ economic circumstances, while RB’s bid was at P44.00/share. RB was the
without prejudice to their position in the WTO;” highest bidder hence it was logically considered as the
WHEREAS, on 19 May 2015, the National winning bidder but is yet to be declared so. Pending
Economic and Development Authority (NEDA) declaration, MPH matches RB’s bid and invoked the
Board recommended the reduction of the Most- Filipino First Policy enshrined under par. 2, Sec. 10,
Favored-Nation (MFN) rates of duty on certain tariff Art. XII of the 1987 Constitution which provides:
lines under the APEC List of Environmental Goods Section 10. The Congress shall, upon
by 2015; and recommendation of the economic and planning
WHEREAS, Section 401 of Presidential Decree agency, when the national interest dictates, reserve to
(PD) No. 1464, or the Tariff and Customs Code of the citizens of the Philippines or to corporations or
Philippines (TCCP), as amended, empowers the associations at least sixty per centum of whose capital
President, upon the recommendation of NEDA, to is owned by such citizens, or such higher percentage
increase, reduce or remove existing rates of import as Congress may prescribe, certain areas of
duty, as well as to modify the tariff nomenclature. investments. The Congress shall enact measures that
NOW, THEREFORE, I, BENIGNO S. AQUINO will encourage the formation and operation of
III, President of the Philippines, by virtue of the enterprises whose capital is wholly owned by
power vested in me by law, do hereby order: Filipinos.
SECTION 1. Rates of Import Duty. The articles In the grant of rights, privileges, and concessions
specifically listed in Annex A hereof, as classified covering the national economy and patrimony, the
under Section 104 of the TCCP, as amended, shall be State shall give preference to qualified Filipinos.
subject to the MFN rate of duty in accordance with The State shall regulate and exercise authority over
the schedule indicated opposite each article. The rates foreign investments within its national jurisdiction
of import duty on tariff headings and subheadings and in accordance with its national goals and
which are not enumerated and those which are listed priorities.
but represented by the symbol “xxx” shall remain in But GSIS refused to accept said offer. In turn MPH
force and effect. filed a petition for TRO against GSIS to avoid the

Page 18 of 70
perfection/consummation of the sale to RB. TRO was superseding the previous House Impeachment Rules
granted. approved by the 11th Congress.
RB then assailed the TRO issued in favor of MPH On 22 July 2002, the House of Representatives
arguing among others that: adopted a Resolution, which directed the Committee
Par. 2, Sec. 10, Art. XII of the 1987 Constitution on Justice “to conduct an investigation, in aid of
needs an implementing law because it is merely a legislation, on the manner of disbursements and
statement of principle and policy (not self-executing); expenditures by the Chief Justice of the Supreme
Even if said passage is self-executing, Manila Hotel Court of the Judiciary Development Fund (JDF).
does not fall under national patrimony. On 2 June 2003, former President Joseph E. Estrada
ISSUE: Whether or not RB should be admitted as the filed an impeachment complaint (first impeachment
highest bidder and hence be proclaimed as the legit complaint) against Chief Justice Hilario G. Davide Jr.
buyer of shares. and seven Associate Justices of the Supreme Court for
HELD: No. MPH should be awarded the sale “culpable violation of the Constitution, betrayal of the
pursuant to Art 12 of the 1987 Const. This is in light public trust and other high crimes.” The complaint
of the Filipino First Policy. was endorsed by House Representatives, and was
Par. 2, Sec. 10, Art. 12 of the 1987 Constitution is self referred to the House Committee on Justice on 5
executing. The Constitution is the fundamental, August 2003 in accordance with Section 3(2) of
paramount and supreme law of the nation, it is Article XI of the Constitution. The House Committee
deemed written in every statute and contract. on Justice ruled on 13 October 2003 that the first
Manila Hotel falls under national patrimony. impeachment complaint was “sufficient in form,” but
Patrimony in its plain and ordinary meaning pertains voted to dismiss the same on 22 October 2003 for
to heritage. When the Constitution speaks of national being insufficient in substance.
patrimony, it refers not only to the natural resources The following day or on 23 October 2003, the second
of the Philippines, as the Constitution could have very impeachment complaint was filed with the Secretary
well used the term natural resources, but also to General of the House by House Representatives
the cultural heritage of the Filipinos. It also refers to against Chief Justice Hilario G. Davide, Jr., founded
our intelligence in arts, sciences and letters. on the alleged results of the legislative inquiry
Therefore, we should develop not only our lands, initiated by above-mentioned House Resolution. The
forests, mines and other natural resources but also the second impeachment complaint was accompanied by
mental ability or faculty of our people. Note that, for a “Resolution of Endorsement/Impeachment” signed
more than 8 decades (9 now) Manila Hotel has bore by at least 1/3 of all the Members of the House of
mute witness to the triumphs and failures, loves and Representatives.
frustrations of the Filipinos; its existence is impressed Various petitions for certiorari, prohibition, and
with public interest; its own historicity associated mandamus were filed with the Supreme Court against
with our struggle for sovereignty, independence and the House of Representatives, et. al., most of which
nationhood. petitions contend that the filing of the second
Herein resolved as well is the term Qualified Filipinos impeachment complaint is unconstitutional as it
which not only pertains to individuals but to violates the provision of Section 5 of Article XI of the
corporations as well and other juridical Constitution that “[n]o impeachment proceedings
entities/personalities. The term “qualified Filipinos” shall be initiated against the same official more than
simply means that preference shall be given to those once within a period of one year.”
citizens who can make a viable contribution to the Issues:
common good, because of credible competence and Whether or not the offenses alleged in the Second
efficiency. It certainly does NOT mandate the impeachment complaint constitute valid impeachable
pampering and preferential treatment to Filipino offenses under the Constitution.
citizens or organizations that are incompetent or Whether or not Sections 15 and 16 of Rule V of the
inefficient, since such an indiscriminate preference Rules on Impeachment adopted by the 12th Congress
would be counter productive and inimical to the are unconstitutional for violating the provisions of
common good. Section 3, Article XI of the Constitution.
In the granting of economic rights, privileges, and Whether the second impeachment complaint is barred
concessions, when a choice has to be made between a under Section 3(5) of Article XI of the Constitution.
“qualified foreigner” and a “qualified Filipino,” the Held:
latter shall be chosen over the former.” This issue is a non-justiciable political question which
is beyond the scope of the judicial power of the
2. INTERPRETATION OF THE Supreme Court under Section 1, Article VIII of the
CONSTITUTION Constitution.
Any discussion of this issue would require the Court
25. Ernesto B. Francisco Jr. vs Nagmamalasakit na to make a determination of what constitutes an
Manananggol ng mga Manggagawang Pilipino impeachable offense. Such a determination is a purely
MARCH 31, 2018 BY LAWISKOOL political question which the Constitution has left to
GR No. 160261 the sound discretion of the legislation. Such an intent
10 November 2003 is clear from the deliberations of the Constitutional
Facts: Commission.
On 28 November 2001, the 12th Congress of the Courts will not touch the issue of constitutionality
House of Representatives adopted and approved the unless it is truly unavoidable and is the very lis
Rules of Procedure in Impeachment Proceedings, mota or crux of the controversy.
Page 19 of 70
The Rule of Impeachment adopted by the House of resolution, as the case may be, is not sufficient in
Congress is unconstitutional. substance.
Section 3 of Article XI provides that “The Congress In cases where a verified complaint or a resolution of
shall promulgate its rules on impeachment to impeachment is filed or endorsed, as the case may be,
effectively carry out the purpose of this section.” by at least one-third (1/3) of the Members of the
Clearly, its power to promulgate its rules on House, impeachment proceedings are deemed
impeachment is limited by the phrase “to effectively initiated at the time of the filing of such verified
carry out the purpose of this section.” Hence, these complaint or resolution of impeachment with the
rules cannot contravene the very purpose of the Secretary General. (emphasis, underscoring and
Constitution which said rules were intended to italics supplied)
effectively carry out. Moreover, Section 3 of Article Section 17. Bar Against Initiation of Impeachment
XI clearly provides for other specific limitations on Proceedings. −Within a period of one (1) year
its power to make rules. from the date impeachment proceedings are
It is basic that all rules must not contravene the deemed initiated as provided in Section 16 hereof,
Constitution which is the fundamental law. If as no impeachment proceedings, as such, can be
alleged Congress had absolute rule making power, initiated against the same official. (emphasis,
then it would by necessary implication have the underscoring and italics supplied)
power to alter or amend the meaning of the
Constitution without need of referendum.
It falls within the one year bar provided in the 3. PREAMBLE OF THE 1987 CONSTITUTION
Constitution.
Having concluded that the initiation takes place by the 27. AGLIPAY vs RUIZ – 1937
act of filing of the impeachment complaint and 64 Phil. 201 – Political Law – Appropriation –
referral to the House Committee on Justice, the initial Religious Sect – Religious Freedom
action taken thereon, the meaning of Section 3 (5) of The 33rd International Eucharistic Congress
Article XI becomes clear. Once an impeachment organized by the Roman Catholic Church took place
complaint has been initiated in the foregoing manner, sometime in 1936. In commemoration thereof. then
another may not be filed against the same official Director of Posts, Juan Ruiz, initiated the production
within a one year period following Article XI, Section of certain stamps the design of which would have in
3(5) of the Constitution. their center a chalice, with grape and stalks of wheat
Considering that the first impeachment complaint, as border design. Eventually, the stamps were
was filed by former President Estrada against Chief produced and some were sold pursuant to Act No.
Justice Hilario G. Davide, Jr., along with seven 4052, which provides for appropriation.
associate justices of this Court, on June 2, 2003 and Gregorio Aglipay, the head of the Philippine
referred to the House Committee on Justice on Independent Church, assailed the production and sale
August 5, 2003, the second impeachment complaint of such stamps. Aglipay contends that the funding of
filed by Representatives Gilberto C. Teodoro, Jr. and said stamps commemorative to a particular religious
Felix William Fuentebella against the Chief Justice event is in violation of Sec 13, Article 6 of the
on October 23, 2003 violates the constitutional Philippine Constitution which prohibits the
prohibition against the initiation of impeachment appropriation or usage of public money for the use or
proceedings against the same impeachable officer benefit of any church or denomination.
within a one-year period. ISSUE: Whether or not the production of the said
Hence, Sections 16 and 17 of Rule V of the Rules of stamps violate the Constitution.
Procedure in Impeachment Proceedings which were HELD: No. The sale of stamps is not in violation of
approved by the House of Representatives on the Constitution. In fact, what was emphasized on the
November 28, 2001 are unconstitutional. stamps was not the religious event itself but rather the
Consequently, the second impeachment complaint City of Manila as being the seat of such event. Act
against Chief Justice Hilario G. Davide, Jr. which was No. 4052 on the other hand did not appropriate any
filed by Representatives Gilberto C. Teodoro, Jr. and public money to a religious event. Act No. 4052
Felix William B. Fuentebella with the Office of the appropriated the sum of P60,000.00 for the cost of
Secretary General of the House of Representatives on plates and printing of postage stamps with new
October 23, 2003 is barred under paragraph 5, section designs and other expenses incident thereto, and
3 of Article XI of the Constitution. merely authorizes the Director of Posts, with the
Section 16. Impeachment Proceedings Deemed approval of the Secretary of Public Works and
Initiated. ─ In cases where a Member of the House Communications, to dispose of the amount
files a verified complaint of impeachment or a citizen appropriated in the manner indicated and “as often as
files a verified complaint that is endorsed by a may be deemed advantageous to the Government”.
Member of the House through a resolution of The fact that the fund is being used for such is only
endorsement against an impeachable officer, incidental to the function of Director of Posts and
impeachment proceedings against such official under his discretion.
are deemed initiated on the day the Committee on On religious freedom
Justice finds that the verified complaint and or The Supreme Court noted however that the elevating
resolution against such official, as the case may be, influence of religion is recognized here as elsewhere.
is sufficient in substance, or on the date the House Evidence would be our preamble where we implored
votes to overturn or affirm the finding of the said the aid of divine providence to establish an ideal
Committee that the verified complaint and or government. If should also be further noted that
Page 20 of 70
religious freedom as a constitutional mandate is not Section 2. All waters within the baselines provided
an inhibition of profound reverence to religion. for in Section one hereof are considered inland or
internal waters of the Philippines.
Section 3. This Act shall take effect upon its
approval.
III – STATE Approved: June 17, 1961.

CONCEPT OF THE STATE Republic Act No. 9522 March 10, 2009
AN ACT TO AMEND CERTAIN PROVISIONS
ARTICLE I OF REPUBLIC ACT NO. 3046, AS AMENDED
ARTICLE 1 – CONSTI BY REPUBLIC ACT NO. 5446, TO DEFINE
NATIONAL TERRITORY THE ARCHIPELAGIC BASELINE OF THE
The national territory comprises the Philippine PHILIPPINES AND FOR OTHER PURPOSES
archipelago, with all the islands and waters embraced Be it enacted by the Senate and House of
therein, and all other territories over which the Representatives of the Philippines in Congress
Philippines has sovereignty or jurisdiction, consisting assembled::
of its terrestrial, fluvial and aerial domains, including Section 1. Section 1 of Republic Act No. 3046,
its territorial sea, the seabed, the subsoil, the insular entitled "An Act to Define the Baselines of the
shelves, and other submarine areas. The waters Territorial Sea of the Philippines", as amended by
around, between, and connecting the islands of the Section 1 of Republic Act No. 5446, is hereby
archipelago, regardless of their breadth and amended to read as follows:
dimensions, form part of the internal waters of the Section 1. The baselines of the Philippines
Philippines. archipelago are hereby defined and described
specifically as follows:
REPUBLIC ACT No. 3046
(as amended by RA 5446)
AN ACT DEFINE THE BASELINES OF THE 28. MAGALLONA vs ERMITA
TERRITORIAL SEA OF THE PHILIPPINES. 655 SCRA 476 – Political Law – National Territory –
WHEREAS, the Constitution of the Philippines RA 9522 is Constitutional
describes the national territory as comprising all the In March 2009, Republic Act 9522, an act defining
territory ceded to the United States by the Treaty of the archipelagic baselines of the Philippines was
Paris concluded between the United States and Spain enacted – the law is also known as the Baselines Law.
on December 10, 1898, the limits of which are set This law was meant to comply with the terms of the
forth in Article III of said treaty, together with all the third United Nations Convention on the Law of the
islands embraced in the treaty concluded at Sea (UNCLOS III), ratified by the Philippines in
Washington, between the United States and Spain on February 1984.
November 7, 1900, and in the treaty concluded Professor Merlin Magallona et al questioned the
between the United States and Great Britain on validity of RA 9522 as they contend, among others,
January 2, 1930, and all the territory over which the that the law decreased the national territory of the
Government of the Philippine Islands exercised Philippines hence the law is unconstitutional. Some
jurisdiction at the time of the adoption of the of their particular arguments are as follows:
Constitution; a. the law abandoned the demarcation set by the
WHEREAS, all the waters within the limits set forth Treaty of Paris and other ancillary treaties – this also
in the above-mentioned treaties have always been resulted to the exclusion of our claim over Sabah;
regarded as part of the territory of the Philippine b. the law, as well as UNCLOS itself, describes the
Islands; Philippine waters as “archipelagic” waters which, in
WHEREAS, all the waters around, between and international law, opens our waters landward of the
connecting the various islands of the Philippines baselines to maritime passage by all vessels (innocent
archipelago, irrespective of their width or dimension, passage) and aircrafts (overflight), undermining
have always been considered as necessary Philippine sovereignty and national security,
appurtenances of the land territory, forming part of contravening the country’s nuclear-free policy, and
the inland or internal waters of the Philippines; damaging marine resources, in violation of relevant
WHEREAS, all the waters beyond the outermost constitutional provisions;
islands of the archipelago but within the limits of the c. the classification of the Kalayaan Island Group
boundaries set forth in the aforementioned treaties (KIG), as well as the Scarborough Shoal (bajo de
comprise the territorial sea of the Philippines; masinloc), as a “regime of islands” pursuant to
WHEREAS, the baselines from which the territorial UNCLOS results in the loss of a large maritime area
sea of the Philippines is determined consist of straight but also prejudices the livelihood of subsistence
lines joining appropriate points of the outermost fishermen.
islands of the archipelago; and ISSUE: Whether or not the contentions of Magallona
WHEREAS, the said baselines should be clarified et al are tenable.
and specifically defined and described for the HELD: No. The Supreme Court emphasized that RA
information of all concerned; Now, therefor, 9522, or UNCLOS, itself is not a means to acquire, or
Section 1. The baselines for the territorial sea of the lose, territory. The treaty and the baseline law has
Philippines are hereby defined and described nothing to do with the acquisition, enlargement, or
specifically as follows: diminution of the Philippine territory. What controls
Page 21 of 70
when it comes to acquisition or loss of territory is the b. UNCLOS may term our waters as “archipelagic
international law principle on occupation, accretion, waters” and that we may term it as our “internal
cession and prescription and NOT the execution waters”, but the bottom line is that our country
of multilateral treaties on the regulations of sea-use exercises sovereignty over these waters and
rights or enacting statutes to comply with the treaty’s UNCLOS itself recognizes that. However, due to our
terms to delimit maritime zones and continental observance of international law, we allow the
shelves. exercise of others of their right of innocent
The law did not decrease the demarcation of our passage. No modern State can validly invoke its
territory. In fact it increased it. Under the old law sovereignty to absolutely forbid innocent passage that
amended by RA 9522 (RA 3046), we adhered with is exercised in accordance with customary
the rectangular lines enclosing the Philippines. The international law without risking retaliatory measures
area that it covered was 440,994 square nautical from the international community.
miles (sq. na. mi.). But under 9522, and with the c. The classification of the KIG (or the Spratly’s), as
inclusion of the exclusive economic zone, the extent well as the Scarborough Shoal, as a regime of islands
of our maritime was increased to 586,210 sq. na. did not diminish our maritime area. Under UNCLOS
mi. (See image below for comparison) and under the baselines law, since they are regimes of
If any, the baselines law is a notice to the international islands, they generate their own maritime zones – in
community of the scope of the maritime space and short, they are not to be enclosed within the baselines
submarine areas within which States parties exercise of the main archipelago (which is the Philippine
treaty-based rights. Island group). This is because if we do that, then we
will be enclosing a larger area which would already
depart from the provisions of UNCLOS – that the
demarcation should follow the natural contour of the
archipelago.
Nevertheless, we still continue to lay claim over the
KIG and the Scarborough Shoal through effective
occupation.
NOTES:
Under UNCLOS and the baselines law, we have three
levels of maritime zones where we exercise treaty-
based rights:
a. territorial waters – 12 nautical miles from the
baselines; where we exercise sovereignty
b. contiguous zone – 24 nautical miles from the
baselines; jurisdiction where we can enforce customs,
fiscal, immigration, and sanitation laws (CFIS).
c. exclusive economic zone – 200 nautical miles from
the baselines; where we have the right to exploit the
living and non-living resources in the exclusive
economic zone
Note: a fourth zone may be added which is
the continental shelf – this is covered by Article 77 of
the UNCLOS.

REGALIAN DOCTRINE

29. Cruz vs Secretary of DENR


Natural Resources and Environmental Law;
Constitutional Law; IPRA; Regalian Doctrine

GR. No. 135385, Dec. 6, 2000

FACTS:
Petitioners Isagani Cruz and Cesar Europa filed a suit
for prohibition and mandamus as citizens and
Anent their particular contentions: taxpayers, assailing the constitutionality of certain
a. The law did not abandon the Sabah claim. This is provisions of Republic Act No. 8371, otherwise
evident on the provision of Section 2 of RA 9522: known as the Indigenous People’s Rights Act of 1997
Section 2. The definition of the baselines of the (IPRA) and its implementing rules and regulations
territorial sea of the Philippine Archipelago as (IRR). The petitioners assail certain provisions of the
provided in this Act is without prejudice to the IPRA and its IRR on the ground that these amount to
delineation of the baselines of the territorial sea an unlawful deprivation of the State’s ownership over
around the territory of Sabah, situated in North lands of the public domain as well as minerals and
Borneo, over which the Republic of the Philippines other natural resources therein, in violation of the
has acquired dominion and sovereignty. regalian doctrine embodied in section 2, Article XII
of the Constitution.
Page 22 of 70
whatever possession they had cannot ripen into
ISSUE: ownership. RTC Ruled in favor of Yap et al. The OSG
Do the provisions of IPRA contravene the appealed.
Constitution? G.R. No. 173775
During the pendency of G.R. No. 167707, in May
HELD: 2006, then President Gloria Macapagal-
No, the provisions of IPRA do not contravene the Arroyo issued Proclamation No. 1064 classifying
Constitution. Examining the IPRA, there is nothing in Boracay Island into four hundred (400) hectares of
the law that grants to the ICCs/IPs ownership over the reserved forest land (protection purposes) and six
natural resources within their ancestral domain. hundred twenty-eight and 96/100 (628.96) hectares of
Ownership over the natural resources in the ancestral agricultural land (alienable and disposable). The
domains remains with the State and the rights granted Proclamation likewise provided for a fifteen-meter
by the IPRA to the ICCs/IPs over the natural buffer zone on each side of the centerline of roads and
resources in their ancestral domains merely gives trails, reserved for right-of-way and which shall form
them, as owners and occupants of the land on which part of the area reserved for forest land protection
the resources are found, the right to the small scale purposes.
utilization of these resources, and at the same time, a Subsequently, Dr. Orlando Sacay, and other Boracay
priority in their large scale development and landowners in Boracay filed with the Supreme Court
exploitation. (SC) an original petition for prohibition, mandamus,
and nullification of Proclamation No. 1064. They
Additionally, ancestral lands and ancestral domains alleged that the Proclamation infringed on their “prior
are not part of the lands of the public domain. They vested rights” over portions of Boracay. They have
are private lands and belong to the ICCs/IPs by native been in continued possession of their respective lots
title, which is a concept of private land title that in Boracay since time immemorial. They have also
existed irrespective of any royal grant from the State. invested billions of pesos in developing their lands
However, the right of ownership and possession by and building internationally renowned first class
the ICCs/IPs of their ancestral domains is a limited resorts on their lots.
form of ownership and does not include the right to The OSG again opposed Sacay’s petition. The OSG
alienate the same. argued that Sacay et al do not have a vested right over
their occupied portions in the island. Boracay is an
unclassified public forest land pursuant to Section
30. COLLADO VS CA (N/A) 3(a) of PD No. 705. Being public forest, the claimed
portions of the island are inalienable and cannot be
the subject of judicial confirmation of imperfect title.
31. SEC vs DENR VS YAP It is only the executive department, not the courts,
NOTE: This case is consolidated with G.R. No. which has authority to reclassify lands of the public
167707 (Secretary of DENR vs Yap). This case is domain into alienable and disposable lands. There is
more popularly referred to as “Boracay a need for a positive government act in order to
Landowners vs DENR” release the lots for disposition.
568 SCRA 164 – Civil Law – Land Titles and Deeds ISSUES: Whether Proclamation No. 1801 and PTA
– Land Classifications – Boracay Cases – Positive Circular No. 3-82 pose any legal obstacle for Yap et
Act by the Government in Reclassifying Lands al and Sacay et al, and all those similarly situated, to
These are two consolidated cases. acquire title to their occupied lands in Boracay Island.
G.R. No. 167707 HELD: Yes. The SC ruled against Yap et al and
Boracay Mayor Jose Yap et al filed for declaratory Sacay et al. The Regalian Doctrine dictates that all
relief to have a judicial confirmation of imperfect title lands of the public domain belong to the State, that
or survey of land for titling purposes for the land the State is the source of any asserted right to
they’ve been occupying in Boracay. Yap et al alleged ownership of land and charged with the conservation
that Proclamation No. 1801 and PTA Circular No. 3- of such patrimony. All lands that have not been
82 raised doubts on their right to secure titles over acquired from the government, either by purchase or
their occupied lands. They declared that they by grant, belong to the State as part of the inalienable
themselves, or through their predecessors-in-interest, public domain.
had been in open, continuous, exclusive, and A positive act declaring land as alienable and
notorious possession and occupation in Boracay since disposable is required. In keeping with the
June 12, 1945, or earlier since time immemorial. They presumption of State ownership, there must be
declared their lands for tax purposes and paid realty a positive act of the government, such as an official
taxes on them. proclamation, declassifying inalienable public land
The Republic, through the Office of the Solicitor into disposable land for agricultural or other
General (OSG), opposed the petition for declaratory purposes. In the case at bar, no such proclamation,
relief. The OSG countered that Boracay Island was executive order, administrative action, report, statute,
an unclassified landof the public domain. It formed or certification was presented. The records are bereft
part of the mass of lands classified as “public forest,” of evidence showing that, prior to 2006, the portions
which was not available for disposition pursuant to of Boracay occupied by private claimants were
Section 3(a) of Presidential Decree (PD) No. 705 or subject of a government proclamation that the land is
the Revised Forestry Code. Since Boracay Island had alienable and disposable. Absent such well-nigh
not been classified as alienable and disposable, incontrovertible evidence, the Court cannot accept the
Page 23 of 70
submission that lands occupied by private claimants The instant petition for review assails the Decision
were already open to disposition before 2006. Matters and Resolution of the Court of Appeals dated 18 July
of land classification or reclassification cannot be 2001 and 18 March 2002 in CA-G.R. CV No. 64097,
assumed. reversing and setting aside the Decision of the
Also, private claimants also contend that their Regional Trial Court of Cebu, Branch 11, Cebu City
continued possession of portions of Boracay Island in Civil Case No. CEB-17173.
for the requisite period of ten (10) years under Act The antecedents follow.
No. 926 ipso facto converted the island into private On 18 October 1967, Socorro Orcullo (Orcullo) filed
ownership. Private claimants’ continued possession her application for Free Patent for Lot No. 1434 of
under Act No. 926 does not create a presumption Cad-315-D, a parcel of land with an area of 12.8477
that the land is alienable. It is plain error for hectares located in Barangay Abugon, Sibonga, Cebu.
petitioners to argue that under the Philippine Bill Thereafter, on 14 February 1971, the Secretary of
of 1902 and Public Land Act No. 926, mere Agriculture and Natural Resources issued Free Patent
possession by private individuals of lands creates No. 473408 for Lot No. 1434, while the Registry of
the legal presumption that the lands are alienable Deeds for the Province of Cebu issued Original
and disposable. Certificate of Title (OCT) No. 0-6667 over the said
Private claimants are not entitled to apply for lot.1 Subsequently, the subject lot was sold2 to SAAD
judicial confirmation of imperfect title under CA Agro- Industries, Inc. (petitioner) by one of Orcullo's
No. 141. Neither do they have vested rights over the heirs.
occupied lands under the said law. There are two Sometime in 1995, the Republic of the Philippines,
requisites for judicial confirmation of imperfect or through the Solicitor General, filed a complaint3 for
incomplete title under CA No. 141, namely: annulment of title and reversion of the lot covered by
(1) open, continuous, exclusive, and notorious Free Patent No. 473408 and OCT No. 0-6667 and
possession and occupation of the subject land by reversion of Lot No. 1434 of Cad-315-D to the mass
himself or through his predecessors-in-interest under of the public domain, on the ground that the issuance
a bona fide claim of ownership since time of the said free patent and title for Lot No. 1434 was
immemorial or from June 12, 1945; and irregular and erroneous, following the discovery that
(2) the classification of the land as alienable and the lot is allegedly part of the timberland and forest
disposable land of the public domain. reserve of Sibonga, Cebu. The discovery was made
The tax declarations in the name of private claimants after Pedro Urgello filed a letter-complaint with the
are insufficient to prove the first element of Regional Executive Director of the Forest
possession. The SC noted that the earliest of the tax Management Sector, Department of Environment and
declarations in the name of private claimants were Natural Resources (DENR) Region VII, Cebu City,
issued in 1993. Being of recent dates, the tax about the alleged illegal cutting of mangrove trees and
declarations are not sufficient to convince this Court construction of dikes within the area covered by
that the period of possession and occupation Urgello's Fishpond Lease Agreement.4 On 14 July
commenced on June 12, 1945. 1995, Urgello filed a complaint-in-intervention
Yap et al and Sacay et al insist that they have a vested against the heirs of Orcullo, adopting the allegations
right in Boracay, having been in possession of the of respondent.5 However, the heirs failed to file their
island for a long time. They have invested millions of answer to the complaint and were thus declared in
pesos in developing the island into a tourist spot. They default.6
say their continued possession and investments give In its Decision7 dated 15 May 1999, the trial court
them a vested right which cannot be unilaterally dismissed the complaint, finding that respondent
rescinded by Proclamation No. 1064. failed to show that the subject lot is part of the
The continued possession and considerable timberland or forest reserve or that it has been
investment of private claimants do not automatically classified as such before the issuance of the free
give them a vested right in Boracay. Nor do these give patent and the original title. According to the trial
them a right to apply for a title to the land they are court, the issuance of the free patent and title was
presently occupying. The SC is constitutionally regular and in order, and must be accorded full faith.
bound to decide cases based on the evidence Considering the validity of the free patent and the
presented and the laws applicable. As the law and OCT, petitioner's purchase of the property was also
jurisprudence stand, private claimants are ineligible declared legal and valid. The trial court also denied
to apply for a judicial confirmation of title over their the complaint-in-intervention filed by Urgello.
occupied portions in Boracay even with their On appeal, the Court of Appeals in its
continued possession and considerable investment in Decision8 reversed and set aside the trial court's
the island. judgment. It held that timber or forest lands, to which
the subject lot belongs, are not subject to private
32. G.R. No. 152570 September 27, 2006 ownership, unless these are first classified as
SAAD AGRO-INDUSTRIES, INC., petitioner, agricultural lands. Thus, absent any declassification
vs. of the subject lot from forest to alienable and
REPUBLIC OF THE PHILIPPINES, respondent. disposable land for agricultural purposes,9 the
x------------------------------------------------x officers erred in approving Orcullo's free patent
PEDRO URGELLO, intervenor-appellant. application and in issuing the OCT; hence, title to the
DECISION lot must be cancelled.10 Consequently, the Court of
TINGA, J.: Appeals invalidated the sale of the lot to petitioner.
However, it declared that Urgello's Fishpond Lease
Page 24 of 70
Agreement may continue until its expiration because Petitioner further contends that the projection survey
lease does not pass title to the lessee; but thereafter, conducted by the DENR to determine if the subject
the lease should not be renewed. Accordingly, the lot falls within the forest area "is not clear, precise and
Court of Appeals decreed: conclusive," since the foresters who conducted the
WHEREFORE, the decision appealed from is survey used a magnetic box compass, an unreliable
hereby REVERSED and SET ASIDE and another one and inaccurate instrument, whose results are easily
issued declaring Free Patent No. 473408 and the affected by high tension wires and stones with iron
corresponding OCT [No.] 0-6667 as NULL and minerals.18
VOID ab initio. Finally, petitioner claims that respondent failed to
SAAD Agro-Industries, Inc. is directed to surrender overcome the presumption of regularity of the
the owner's duplicate copy of OCT [No.] 0-6667 to issuance of the free patent and title in favor of Socorro
the Register of Deeds of Cebu City. Orcullo.
The Register of Deeds of Cebu City is hereby ordered In sum, petitioner asserts that respondent failed to
to cancel OCT [No.] 0-6667 and all other transfer show that the subject lot is inside the timberland
certificates of title that may have been subsequently block, thereby casting doubt on the accuracy of the
issued. survey conducted by the Bureau of Forestry and the
Lot No. 1434, CAD 315[-]D located at Barangay opinions of DENR officers. Since respondent is the
Abugon, Sibonga, Cebu, subject matter of this case, original plaintiff in the reversion case, the burden is
is hereby REVERTED as part of [the] public domain on it to prove that the subject lot is part of the
and to be classified as timberland.11 timberland block, petitioner adds.
Petitioner's motion for reconsideration, claiming There is merit in the petition.
insufficiency of evidence and failure to consider Under the Regalian doctrine or jura regalia, all lands
pertinent laws, proved futile as it was dismissed for of the public domain belong to the State, and the State
lack of merit. The Court of Appeals categorically is the source of any asserted right to ownership in land
stated that there was a preponderance of evidence and charged with the conservation of such
showing that the subject lot is within the timberland patrimony.19 Under this doctrine, lands not otherwise
area.12 appearing to be clearly within private ownership are
Petitioner now claims that the Court of Appeals erred presumed to belong to the State.20 In instances where
in relying on the DENR officer's testimony. It claims a parcel of land considered to be inalienable land of
that the testimony was a mere opinion to the effect the public domain is found under private ownership,
that if there was no classification yet of an area, such the Government is allowed by law to file an action for
area should be considered as a public forest. Such reversion,21 which is an action where the ultimate
opinion was premised on the officer's construction of relief sought is to revert the land to the government
a provision of Presidential Decree (P.D.) No. 705, under the Regalian doctrine. Considering that the land
otherwise known as the Revised Forestry Code,13 the subject of the action originated from a grant by the
pertinent portion of which reads: government, its cancellation is a matter between the
Those still to be classified under the present system grantor and the grantee.22
shall continue to remain as part of the public forest.14 It has been held that a complaint for reversion
Petitioner points out that P.D. No. 705 took effect on involves a serious controversy, involving a question
19 May 1975, or long after the issuance of the free of fraud and misrepresentation committed against the
patent and title in question. Thus, the provision government and it is aimed at the return of the
stating that all public lands should be considered disputed portion of the public domain. It seeks to
as "part of the public forests" until a land cancel the original certificate of registration, and
classification team has declassified them is applicable nullify the original certificate of title, including the
only after the effectivity of P.D. No. 705 and cannot transfer certificate of title of the successors-in-interest
be made retroactive to cover and prejudice vested because the same were all procured through fraud and
rights acquired prior to the effectivity of said law, misrepresentation.23 Thus, the State, as the party
petitioner concludes.15 It adds that if the subject lot alleging the fraud and misrepresentation that attended
was encompassed by the term "public forest," the the application of the free patent, bears that burden of
same should have been designated as a "Timberland proof. Fraud and misrepresentation, as grounds for
Block," not as Cadastral Lot No. 1434, CAF-315-D, cancellation of patent and annulment of title, should
Sibonga Cadastre which was the designation made by never be presumed but must be proved by clear and
the Republic prior to 1972.16 convincing evidence, mere preponderance of
Petitioner also questions the Court of Appeals' evidence not even being adequate.24
reliance on the land classification map (L.C. Map) It is but judicious to require the Government, in an
presented by respondent. The trial court had action for reversion, to show the details attending the
previously declared L.C. Map No. 2961 as issuance of title over the alleged inalienable land and
inadmissible, finding that "the plaintiff has not duly explain why such issuance has deprived the State of
proved the authenticity and contents." According to the claimed property.
petitioner, the L.C. Map presented in court is neither In the instant case, the Solicitor General claimed that
a certified true copy nor one attested to be a true copy "Free Patent No. 473408 and Original Certificate of
by any DENR official having legal custody of the Title No. 0-6667 were erroneously and irregularly
original thereof, and thus should not have been made obtained as the Bureau of Lands (now Lands
the basis of the cancellation of the free patent and Management Bureau) did not acquire jurisdiction
title.17 over the land subject thereof, nor has it the power and
authority to dispose of the same through [a] free
Page 25 of 70
patent grant, hence, said patent and title are null and awarded to Orcullo. Thus, it finds no application in
void ab initio."25 It was incumbent upon respondent the instant case. Prior forestry laws, including P.D.
to prove that the free patent and original title were No. 389,27 which was revised by P.D. No. 705, does
truly erroneously and irregularly obtained. not contain a similar provision. Article 4 of the Civil
Unfortunately, respondent failed to do so. Code provides that "laws shall have no retroactive
The Court finds that the findings of the trial court effect unless the contrary is provided." The Court
rather than those of the appellate court are more in does not infer any intention on the part of then
accord with the law and jurisprudence. President Marcos to ordain the retroactive application
In concluding that the subject parcel of land falls of Sec. 13 of P.D. No. 705. Thus, even assuming for
within the timberland or forest reserve, the Court of the nonce that subject parcel was unclassified at the
Appeals relied on the testimony of Isabelo R. Montejo time Orcullo applied for a free patent thereto, the fact
that as it had remained unclassified until 1980 and remains that when the free patent and title were issued
consequently became an unclassified forest zone, it thereon in 1971, respondent in essence segregated
was incapable of private appropriation. The pertinent said parcel from the mass of public domain. Thus, it
portions of Montejo's testimony read: can no longer be considered unclassified and forming
Q: And in that particular [R]evised Forestry Code, part of the public forest as provided in P.D. No. 705.
there is that statement that unless classified by a land Respondent's main basis for asserting that the subject
classification team, an area can never be released. lot is part of the timberland or forest reserve is a
A: Yes sir. purported L.C. Map No. 2961.28 However, at the
xxx hearing on 6 June 1997, the trial court denied
Q: Prior to 1980, there was no classification was [sic] admission of the map for the purpose of showing that
ever of the lands of the public domain in the town of the subject lot falls within a timberland reserve after
Sibonga? respondent had failed to submit either a certified true
A: Yes, sir. copy or an official publication thereof.29 The Court
Q: In other words, nobody knew in the whole DNR observes that the document adverted to is a mere
before and now DENR what areas were timberland photocopy of the purported original, and not the blue
and what areas are not timberland in the town of print as insisted by respondent.30 A mere photocopy
Sibonga prior to 1980? does not qualify as competent evidence of the
A: Yes, sir, that is why the law states that if there is existence of the L.C. Map. Under the best evidence
no classification should be [sic] considered as the rule, the original document must be produced, except:
public forest in order to protect the resources.26 1. When the original has been lost or destroyed, or
Obviously, respondent's counsel and witness were cannot be produced in court, without bad faith on the
referring to P.D. No. 705 particularly Section 13 part of the offeror;
thereof which reads: 2. When the original is in the custody or under the
CHAPTER II control of the party against whom the evidence is
CLASSIFICATION AND SURVEY offered, and the latter fails to produce it after
SEC. 13. System of Land Classification.—The reasonable notice;
Department Head shall study, devise, determine and 3. When the original consists of numerous accounts
prescribe the criteria, guidelines and methods for the or other documents which cannot be examined in
proper and accurate classification and survey of all court without great loss of time and the fact sought to
lands of the public domain into agricultural, industrial be established from them is only the general result of
or commercial, residential, settlement, mineral, the whole; and
timber or forest, and grazing lands, and into such 4. When the original is a public record in the custody
other classes as now or may hereafter be provided by of a public officer or is recorded in a public office.31
law, rules and regulations. In this case, respondent claims that the presentation
In the meantime, the Department Head shall simplify of the original L.C. Map is unnecessary since it is in
through inter-bureau action the present system of the custody of a public officer or is recorded in the
determining which of the unclassified lands of the public office.32 Evidence, indeed, is admissible when
public domain are needed for forest purposes and the original of a document is in the custody of a public
declare them as permanent forest to form part of the officer or is recorded in a public office. However, to
forest reserves. He shall declare those classified and prove its contents, there is a need to present a certified
determined not to be needed for forest purposes as copy issued by the public officer in custody
alienable and disposable lands, the administrative thereof.33 In addition, while the L.C. Map may be
jurisdiction and management of which shall be considered a public document and prima
transferred to the Bureau of Lands: Provided, That facie evidence of the facts stated therein,34 the map,
mangrove and other swamps not needed for shore to be admissible for any purpose, must be evidenced
protection and suitable for fishpond purposes shall be by an official publication thereof or by a copy attested
released to, and be placed under the administrative by the officer having legal custody of the record.35
jurisdiction and management of, the Bureau of The rules of admissibility must be applied uniformly.
Fisheries and Aquatic Resources. Those still to be The same rule holds true when the Government is one
classified under the Present system shall continue of the parties. The Government, when it comes to
to remain as part of the public forest. (Emphasis court to litigate with one of its citizens, must submit
supplied.) to the rules of procedure and its rights and privileges
Reliance on this provision is highly misplaced. P.D. at every stage of the proceedings are substantially in
No. 705 was promulgated only on 19 May 1975, or every respect the same as those of its citizens; it
four (4) years after the free patent and title were cannot have a superior advantage. This is so because
Page 26 of 70
when a sovereignty submits itself to the jurisdiction land, unless private interests have intervened
of the court and participates therein, its claims and before such reservation is made.41 (Emphasis
rights are justiciable by every other principle and rule supplied.)
applicable to the claims and rights of the private Obviously, private interests have intervened before
parties under similar circumstances.36 Failure to abide classification was made pursuant to P.D. No. 705. Not
by the rules on admissibility renders the L.C. Map only has Orcullo by herself and through her
submitted by respondent inadmissible as proof to predecessors-in-interest cultivated and possessed the
show that the subject lot is part of the forest reserve. subject lot since 1930, a free patent was also awarded
Some officers from the CENRO office in Argao, to her and a title issued in her name as early as 1971.
Cebu testified that they personally saw the subject lot In fact, it appears that the issuance of the free patent
and that it falls within the timberland or forest reserve. and certificate of title was regular and in order.
Ultimately, however, the basis of their declaration is Orcullo complied with the requisites for the
the L.C. Map which respondent failed to present in acquisition of free patent provided under
accordance with the rules on admissibility. Two Commonwealth Act No. 141 (Public Land Act), as
foresters in fact testified that the subject lot was a certified by the Director of Lands and approved by the
mangrove area.37 The foresters who conducted the Secretary of Agriculture and Natural Resources.42
survey may have been competent and their techniques Besides, the records do not show that respondent has
reliable; nevertheless, the observation that mangroves considered the lot in question as forest reserve prior
grow in the subject lot is not conclusive as to the to the issuance of Free Patent No. 473408 and OCT
nature of the land at present or at the time the free No. 0-6667. To declare the land now as forest land on
patent and title were issued. Assuming that the area is the authority of L.C. Map No. 2961 approved only in
covered by mangroves when they surveyed it, there is 1980, and opinions based on the said map, would
no proof that it was not planted with trees and crops unduly deprive petitioner of their registered property.
at the time Orcullo applied for free patent. The Regalian doctrine is well-enshrined not only in
Respondent was also unable to establish that the the present Constitution, but also in the 1935 and
subject lot has "very deep and muddy soil" or are 1973 Constitutions. The Court has always recognized
"mudflats," such that it is unsuitable for fruit and non- and upheld the Regalian doctrine as the basic
fruit bearing trees.38 Yet these are factual matters foundation of the State's property regime.
which the Court does not generally delve into. As it Nevertheless, in applying this doctrine, we must not
is, a mere declaration from the said officers, without lose sight of the fact that in every claim or right by the
any other supporting evidence, is not sufficient to Government against one of its citizens, the paramount
establish that the area in question is part of the forest considerations of fairness and due process must be
reserve. observed. Respondent in this case failed to show that
Even assuming that the L.C. Map submitted by the subject lot is part of timberland or forest reserve it
respondent is admissible in evidence, still the land in adverted to. In the face of the uncontroverted status of
question can hardly be considered part of the Free Patent No. 473408 and OCT No. 0-6667 as valid
timberland or forest reserve. L.C. Map No. 2961, and regular issuances, respondent's insistence on the
which purports to be the "correct map of the areas classification of the lot as part of the forest reserve
demarcated as permanent forest pursuant of the must be rejected.
provisions of P.D. No. 705 as amended"39 was made WHEREFORE, the petition is GRANTED. The
only in 1980. Thus, the delineation of the areas was Decision of the Court of Appeals dated 16 July 2001
made nine (9) years after Orcullo was awarded the and the Resolution dated 18 March 2002 are
free patent over the subject lot. REVERSED and SET ASIDE. The Decision of the
In Republic v. Court of Appeals,40 the Court, finding Regional Trial Court dated 15 May 1999 dismissing
that the disputed land was classified as timberland 25 the complaint for reversion and the complaint-in-
years after private individuals had commenced their intervention is REINSTATED.
continuous possession and cultivation thereof in good SO ORDERED.
faith, declared that they have the better right. The
Court held:
It is not disputed that the aforesaid Land 2. PEOPLE
Classification Project No. 3, classifying the 22-
hectare area as timberland, was certified by the CITIZENSHIP
Director of Lands only on December 22, 1924, Republic Act 8171
whereas the possession thereof by private Files:
respondents and their predecessor-in-interest Republic Act 8171
commenced as early as 1909. While the AN ACT PROVIDING FOR THE
Government has the right to classify portions of REPATRIATION OF FILIPINO WOMEN WHO
public land, the primary right of a private HAVE LOST THEIR PHILIPPINE
individual who possessed and cultivated the land CITIZENSHIP BY MARRIAGE TO ALIENS
in good faith much prior to such classification AND OF NATURAL-BORN FILIPINOS.
must be recognized and should not be prejudiced Section 1. Filipino women who have lost their
by after-events which could not have been Philippine citizenship by marriage to aliens and
anticipated. Thus, We have held that the natural-born Filipinos who have lost their Philippine
Government, in the first instance may, by citizenship, including their minor children, on
reservation, decide for itself what portions of account of political or economic necessity, may
public land shall be considered forestry reacquire Philippine citizenship
Page 27 of 70
through repatriation in the manner provided in upon myself voluntarily without mental reservation
Section 4 of Commonwealth Act No. 63, as amended: or purpose of evasion."
Provided, That the applicant Natural born citizens of the Philippines who, after the
is not a: effectivity of this Act, become citizens of a foreign
(1) Person opposed to organized government or country shall retain their Philippine citizenship upon
affiliated with any association or group of persons taking the aforesaid oath.
who uphold and teach doctrines opposing organized Section 4. Derivative Citizenship - The unmarried
government; child, whether legitimate, illegitimate or adopted,
(2) Person defending or teaching the necessity or below eighteen (18) years of age, of those who re-
propriety of violence, personal assault, or associatEon acquire Philippine citizenship upon effectivity of this
for the predominance of their ideas; Act shall be deemed citizenship of the Philippines.
(3) Person convictad of crimes involving moral Section 5. Civil and Political Rights and
turpitude; or Liabilities - Those who retain or re-acquire
(4) Person suffering from mental alienation or Philippine citizenship under this Act shall enjoy full
incurablecontagious diseases. civil and political rights and be subject to all attendant
Sec. 2. Repatriation shall be effected by taking the liabilities and responsibilities under existing laws of
necessary oath of allegiance to the Republic of the the Philippines and the following conditions:
Philippines and registration in the proper civil registry (1) Those intending to exercise their right of surffrage
and in the Bureau or Immigration. The Bureau of must Meet the requirements under Section 1, Article
Immigration shall thereupon cancel the pertinent V of the Constitution, Republic Act No. 9189,
alien otherwise known as "The Overseas Absentee Voting
certificate of registration and issue the certificate of Act of 2003" and other existing laws;
identification as Filipino citizen to the repatriated (2) Those seeking elective public in the Philippines
citizen. shall meet the qualification for holding such public
Sec. 3. All laws, decrees, orders, rules and office as required by the Constitution and existing
regulations, or parts thereof inconsistent with this Act laws and, at the time of the filing of the certificate of
are hereby repealed or amended accordingly. candidacy, make a personal and sworn renunciation
Sec. 4. This Act shall take effect thirty (30) days after of any and all foreign citizenship before any public
its publication in a newspaper of general circulation. officer authorized to administer an oath;
(3) Those appointed to any public office shall
subscribe and swear to an oath of allegiance to the
Republic of the Philippines and its duly constituted
Republic Act No. 9225 August 29, 2003 authorities prior to their assumption of
AN ACT MAKING THE CITIZENSHIP OF office: Provided, That they renounce their oath of
PHILIPPINE CITIZENS WHO ACQUIRE allegiance to the country where they took that oath;
FOREIGN CITIZENSHIP PERMANENT. (4) Those intending to practice their profession in the
AMENDING FOR THE PURPOSE Philippines shall apply with the proper authority for a
COMMONWEALTH ACT. NO. 63, AS license or permit to engage in such practice; and
AMENDED AND FOR OTHER PURPOSES (5) That right to vote or be elected or appointed to any
Be it enacted by the Senate and House of public office in the Philippines cannot be exercised
Representatives of the Philippines in Congress by, or extended to, those who:
assembled: (a) are candidates for or are occupying any public
Section 1. Short Title – this act shall be known as the office in the country of which they are naturalized
"Citizenship Retention and Re-acquisition Act of citizens; and/or
2003." (b) are in active service as commissioned or non-
Section 2. Declaration of Policy - It is hereby commissioned officers in the armed forces of the
declared the policy of the State that all Philippine country which they are naturalized citizens.
citizens of another country shall be deemed not to Section 6. Separability Clause - If any section or
have lost their Philippine citizenship under the provision of this Act is held unconstitutional or
conditions of this Act. invalid, any other section or provision not affected
Section 3. Retention of Philippine Citizenship - Any thereby shall remain valid and effective.
provision of law to the contrary notwithstanding, Section 7. Repealing Clause - All laws, decrees,
natural-born citizenship by reason of their orders, rules and regulations inconsistent with the
naturalization as citizens of a foreign country are provisions of this Act are hereby repealed or modified
hereby deemed to have re-acquired Philippine accordingly.
citizenship upon taking the following oath of Section 8. Effectivity Clause – This Act shall take
allegiance to the Republic: effect after fifteen (15) days following its publication
"I _____________________, solemny swear (or in theOfficial Gazette or two (2) newspaper of general
affrim) that I will support and defend the Constitution circulation.
of the Republic of the Philippines and obey the laws
and legal orders promulgated by the duly constituted
authorities of the Philippines; and I hereby declare i. DEFINITION OF CITIZENSHIP
that I recognize and accept the supreme authority of
the Philippines and will maintain true faith and 33. TECSON vs COMELEC
allegiance thereto; and that I imposed this obligation Facts: Victorino X. Fornier, petitioner initiated a
petition before the COMELEC to disqualify FPJ and
Page 28 of 70
to deny due course or to cancel his certificate of JOSE TAN CHONG, petitioner-appellee,
candidacy upon the thesis that FPJ made a material vs.
misrepresentation in his certificate of candidacy by SECRETARY OF LABOR, respondent-appellant.
claiming to be a natural-born Filipino citizen when in This is an appeal taken by the Solicitor-General from
truth, according to Fornier, his parents were the decision of the Court of Jose Tan Chong for a writ
foreigners; his mother, Bessie Kelley Poe, was an of habeas corpus to secure his release from the
American, and his father, Allan Poe, was a Spanish custody of the Secretary of Labor.
national, being the son of Lorenzo Pou, a Spanish It appears that the petitioner was born in San Pablo,
subject. Granting, petitioner asseverated, that Allan F. Laguna, in the month of July, 1915, of a Chinese
Poe was a Filipino citizen, he could not have father named Tan Chong Hong and a Filipino mother
transmitted his Filipino citizenship to FPJ, named Antonia Mangahis; that his parents were
the latter being an illegitimate child of an alien legally married; that sometime in 1925 when the
mother. Petitioner based the allegation of the petitioner was about ten years old he was taken by his
illegitimate birth of respondent on two assertions - parents to China; that on January 25, 1940, he arrived
first, Allan F. Poe contracted a prior marriage to a at the port of Manila and sought entry as a native born
certain Paulita Gomez before his marriage citizen. The board of Special Inquiry assigned to hear
to Bessie Kelley and, second, even if no such prior his case, denied him admission on the alleged ground
marriage had existed, Allan F. Poe, that he is a Chinese citizen, and on appeal, the
married Bessie Kelly only a year after the birth of Secretary of labor affirmed the decision of the Board
respondent. and ordered the deportation of the petitioner to the
port from whence he came. The petitioner sued for a
writ of habeas corpus in the Court of First Instance of
Issue: Whether or Not FPJ is a natural born Filipino Manila which was granted. Hence, the present appeal
citizen. by the Solicitor-General.
The petitioner, having been born in the Philippines
before the approval of our Constitution, of a Chinese
Held: It is necessary to take on the matter of whether father and a Filipino mother, is a Filipino citizen. This
or not respondent FPJ is a natural-born citizen, which, conclusion is in harmony with the policy embodied in
in turn, depended on whether or not the father of the Constitution (par. 4 sec. 1, Art. IV; Torres vs. Tan
respondent, Allan F. Poe, would have himself been a Chim, G. R. No. 46593, promulgated Feb. 3, 1940;
Filipino citizen and, in the affirmative, whether or not Gallofin vs. Ordoñez, G.R. No. 46782, promulgated
the alleged illegitimacy of respondent prevents him June 27, 1940). His sojourn in China did not
from taking after the Filipino citizenship of his adversely affect his Philippine citizenship, it
putative father. Any conclusion on the Filipino appearing that ever since he was twelve years old he
citizenship of Lorenzo Pou could only be drawn from wanted to return to the Philippines but his father
the presumption that having died in 1954 at 84 years would not allow him to come, and he did not have
old, Lorenzo would have been born sometime in the means to pay for his transportation back to the
year 1870, when the Philippines was under Spanish Philippines until the date of his return. Animus
rule, and that San Carlos, Pangasinan, his place of revertendi existed here. (Lim Teco vs. Collector of
residence upon his death in 1954, in the absence of Customs, 23 Phil., 84; Muñoz vs. Collector of
any other evidence, could have well been his place of Customs, 20 Phil., 494; Lorenzo vs. McCoy, 15 Phil.,
residence before death, such that Lorenzo Pou would 559.)lawphil.net
have benefited from the "enmasse Filipinization" that The judgment of the lower court is accordingly
the Philippine Bill had effected in 1902. That affirmed, without pronouncement regarding costs. So
citizenship (of Lorenzo Pou), if acquired, would ordered.
thereby extend to his son, Allan F. Poe, father of
respondent FPJ. The 1935 Constitution, during which iii. IMPORTANCE OF CITIZENSHIP
regime respondent FPJ has seen first light, confers Topic: Dual Citizenship/ Dual Allegiance
citizenship to all persons whose fathers are JAPSON VS. COMELEC
Filipino citizens regardless of whether such Facts:
children are legitimate or illegitimate. Both petitioner Manuel B. Japzon (Japzon) and
private respondent Jaime S. Ty (Ty) were candidates
But while the totality of the evidence may not for the Office of Mayor of the Municipality of
establish conclusively that respondent FPJ is a General Macarthur, Eastern Samar, in the local
natural-born citizen of the Philippines, the elections held on 14 May 2007.
evidence on hand still would preponderate in his Japzon instituted SPA No. 07-568 by filing before
favor enough to hold that he cannot be held guilty the COMELEC a Petition[5] to disqualify and/or
of having made a material misrepresentation in his cancel Ty's Certificate of Candidacy on the ground of
certificate of candidacy in violation of Section 78, material misrepresentation. Japzon averred in his
in relation to Section 74, of the Omnibus Election Petition that Ty was a former natural-born Filipino,
Code. having been born on 9 October 1943 in what was then
Pambujan Sur, Hernani Eastern Samar (now the
Municipality of General Macarthur, Easter Samar) to
ii. Elements of Citizenship spouses Ang Chim Ty (a Chinese) and Crisanta
Aranas Sumiguin (a Filipino).
34. G.R. No. L-47616 October 15, 1941
Page 29 of 70
Ty eventually migrated to the United States of He had reacquired his Philippine citizenship and
America (USA) and became a citizen thereof. Ty had renounced his American citizenship, and he had been
been residing in the USA for the last 25 years. When a resident of the Municipality of General Macarthur,
Ty filed his Certificate of Candidacy on 28 March Eastern Samar, for more than one year prior to the 14
2007, he falsely represented therein that he was a May 2007 elections. Therefore, Ty sought the
resident of Barangay6, Poblacion, General dismissal of Japzon's Petition in SPA No. 07-568.
Macarthur, Eastern Samar, for one year before 14 Ty acquired the highest number of votes and was
May 2007, and was not a permanent resident or declared Mayor of the Municipality of General
immigrant of any foreign country. Macarthur, Eastern Samar, by the Municipal Board of
While Ty may have applied for the reacquisition Canvassers on 15 May 2007.[7]
of his Philippine citizenship, he never actually resided The COMELEC First Division found that Ty
in Barangay 6, Poblacion, General Macarthur, complied with the requirements of Sections 3 and 5
Eastern Samar, for a period of one year immediately of Republic Act No. 9225 and reacquired his
preceding the date of election as required under Philippine citizenship, to wit:
Section 39 of Republic Act No. 7160, otherwise Philippine citizenship is an indispensable requirement
known as the Local Government Code of 1991 for holding an elective public office, and the purpose
Inspite of having reacquisition in his Philippine of the citizenship qualification is none other than to
citizenship, Ty continued to make trips to the USA, ensure that no alien, i.e., no person owing allegiance
the most recent of which was on 31 October 2006 to another nation, shall govern our people and our
lasting until 20 January 2007. country or a unit of territory thereof.
Ty already took his Oath of Allegiance to the Evidences revealed that Ty executed an Oath of
Republic of the Philippines, he continued to comport Allegiance before Noemi T. Diaz, Vice Consul of the
himself as an American citizen as proven by his travel Philippine Consulate General, Los Angeles,
records. He had also failed to renounce his foreign California, U.S.A. on October 2, 2005 and executed
citizenship as required by Republic Act No. 9225, a Renunciation of Foreign Citizenship on March
otherwise known as the Citizenship Retention and 19, 2007 in compliance with R.A. [No.] 9225.
Reacquisition Act of 2003, or related laws. Moreover, neither is Ty a candidate for or occupying
Japzon prayed for in his Petition that the public office nor is in active service as commissioned
COMELEC order the disqualification of Ty from or non-commissioned officer in the armed forces in
running for public office and the cancellation of the the country of which he was naturalized citizen
latter's Certificate of Candidacy. Ty did not commit material misrepresentation in
Ty admitted that he was a natural-born Filipino stating in his Certificate of Candidacy that he was a
who went to the USA to work and subsequently resident of Barangay 6, Poblacion, General
became a naturalized American citizen. Ty claimed, Macarthur, Eastern Samar, for at least one year before
however, that prior to filing his Certificate of the elections on 14 May 2007. It reasoned that:
Candidacy for the Office of Mayor of the Although Ty has lost his domicile in [the] Philippines
Municipality of General Macarthur, Eastern Samar, when he was naturalized as U.S. citizen in 1969, the
on 28 March 2007, he already performed the reacquisition of his Philippine citizenship and
following acts: (1) with the enactment of Republic subsequent acts thereof proved that he has been a
Act No. 9225, granting dual citizenship to natural- resident of Barangay 6, Poblacion, General
born Filipinos, Ty filed with the Philippine Consulate Macarthur, Eastern Samar for at least one (1) year
General in Los Angeles, California, USA, an before the elections held on 14 May 2007 as he
application for the reacquisition of his Philippine represented in his certificate of candidacy.
citizenship; (2) on 2 October 2005, Ty executed an The petition was denied and COMELEC was in
Oath of Allegiance to the Republic of the Philippines favor of the defendant failing to obtain a favorable
before Noemi T. Diaz, Vice Consul of the Philippine resolution from the COMELEC, Japzon proceeded to
Consulate General in Los Angeles, California, USA; file the instant Petition for Certiorari, that the
(3) Ty applied for a Philippine passport indicating in COMELEC had committed grave abuse of discretion
his application that his residence in the Philippines and lack of discretion for dismissing the petition.
was at A. Mabini St., Barangay 6, Poblacion, General Japzon prays for the Court to annul and set aside
Macarthur, Eastern Samar. Ty's application was the Resolutions dated 31 July 2007 and 28 September
approved and he was issued on 26 October 2005 a 2007 of the COMELEC First Division and en banc,
Philippine passport; (4) on 8 March 2006, Ty respectively; to issue a new resolution denying due
personally secured and signed his Community Tax course to or canceling Ty's Certificate of Candidacy;
Certificate (CTC) from the Municipality of General and to declare Japzon as the duly elected Mayor of the
Macarthur, in which he stated that his address was Municipality of General Macarthur, Eastern Samar.
at Barangay 6, Poblacion, General Macarthur, Ty sought the dismissal of the present Petition.
Eastern Samar; (5) thereafter, on 17 July 2006, Ty According to Ty, the COMELEC already found
was registered as a voter in Precinct sufficient evidence to prove that Ty was a resident of
0013A, Barangay 6, Poblacion, General Macarthur, the Municipality of General Macarthur, Eastern
Eastern Samar; (6) Ty secured another CTC dated 4 Samar, one year prior to the 14 May 2007 local
January 2007 again stating therein his address elections. The Court cannot evaluate again the very
as Barangay 6, Poblacion, General Macarthur, same pieces of evidence without violating the well-
Eastern Samar; and (7) finally, Ty executed on 19 entrenched rule that findings of fact of the
March 2007 a duly notarized Renunciation of Foreign COMELEC are binding on the Court.
Citizenship.
Page 30 of 70
The Office of the Solicitor General (OSG), place where one is elected, does not constitute loss of
meanwhile, is of the position that Ty failed to meet residence.[24] The Court also notes, that even with his
the one-year residency requirement set by law to trips to other countries, Ty was actually present in the
qualify him to run as a mayoralty candidate in the 14 Municipality of General Macarthur, Eastern Samar,
May 2007 local elections.The Court finds no merit in Philippines, for at least nine of the 12 months
the Petition at bar. preceding the 14 May 2007 local elections. Even if
. On 19 March 2007, he personally executed a length of actual stay in a place is not necessarily
Renunciation of Foreign Citizenship before a notary determinative of the fact of residence therein, it does
public. By the time he filed his Certificate of strongly support and is only consistent with Ty's
Candidacy for the Office of Mayor of the avowed intent in the instant case to establish
Municipality of General Macarthur, Eastern Samar, residence/domicile in the Municipality of General
on 28 March 2007, he had already effectively Macarthur, Eastern Samar.
renounced his American citizenship, keeping solely Japzon repeatedly brings to the attention of this
his Philippine citizenship. Court that Ty arrived in the Municipality of General
Macarthur, Eastern Samar, on 4 May 2006 only to
comply with the one-year residency requirement, so
The Court of Appeals set aside the appealed orders Ty could run as a mayoralty candidate in the 14 May
of the COMELEC and the Court of Appeals and 2007 elections. In Aquino v. COMELEC,[25] the Court
annulled the election of the respondent as Municipal did not find anything wrong in an individual changing
Mayor of Bolinao, Pangasinan on the ground that residences so he could run for an elective post, for as
respondent's immigration to the United States in 1984 long as he is able to prove with reasonable certainty
constituted an abandonment of his domicile and that he has effected a change of residence for election
residence in the Philippines. Being a green card law purposes for the period required by law. As this
holder, which was proof that he was a permanent Court already found in the present case, Ty has
resident or immigrant of the United States, and in the proven by substantial evidence that he had established
absence of any waiver of his status as such before he residence/domicile in the Municipality of General
ran for election on January 18, 1988, respondent was Macarthur, Eastern Samar, by 4 May 2006, a little
held to be disqualified under §68 of the Omnibus over a year prior to the 14 May 2007 local elections,
Election Code of the Philippines (Batas Pambansa in which he ran as a candidate for the Office of the
Blg. 881). Mayor and in which he garnered the most number of
votes.

ISSUE:
Whether or not the defedant has complied with the To successfully challenge Ty's disqualification,
residency requirement for elective positions. Japzon must clearly demonstrate that Ty's
ineligibility is so patently antagonistic to
RULING: constitutional and legal principles that overriding
such ineligibility and thereby giving effect to the
Yes, the defendant solely complied the residency apparent will of the people would ultimately create
requirements for elective position. greater prejudice to the very democratic institutions
It bears to point out that Republic Act No. 9225 and juristic traditions that our Constitution and laws
governs the manner in which a natural-born Filipino so zealously protect and promote. In this case, Japzon
may reacquire or retain[17] his Philippine citizenship failed to substantiate his claim that Ty is ineligible to
despite acquiring a foreign citizenship, and provides be Mayor of the Municipality, the instant Petition
for his rights and liabilities under such circumstances. for Certiorari is dismiss.
A close scrutiny of said statute would reveal that it
does not at all touch on the matter of residence of the iv. NATURAL BORN CITIZENS
natural-born Filipino taking advantage of its 34. LIMKAICHONG vs COMELEC
provisions. Republic Act No. 9225 imposes no G.R. No. 178831-32, 30 July 2009 [Citizenship;
residency requirement for the reacquisition or Naturalization; C.A. No. 473]
retention of Philippine citizenship; nor does it
mention any effect of such reacquisition or retention FACTS:
of Philippine citizenship on the current residence of Two petitions were consolidated on the issue about
the concerned natural-born Filipino. Clearly, the qualifications of Jocelyn Limkaichong to run for,
Republic Act No. 9225 treats citizenship be elected to, and assume and discharge the position
independently of residence. This is only logical and as Representative of the 1st District of Negros
consistent with the general intent of the law to allow Oriental. The contention of the parties who sought her
for dual citizenship. disqualification is that she is not a natural-born
There is no basis for this Court to require Ty to citizen, hence, she lacks the citizenship requirement
stay in and never leave at all the Municipality of in Section 6, Article VI of the 1987 Constitution. In
General Macarthur, Eastern Samar, for the full one- the election that ensued, she was voted for by the
year period prior to the 14 May 2007 local elections constituents of Negros Oriental and garnered the
so that he could be considered a resident thereof. To highest votes. She was eventually proclaimed as the
the contrary, the Court has previously ruled that winner and has since performed her duties and
absence from residence to pursue studies or practice responsibilities as Member of the House of
a profession or registration as a voter other than in the Representatives.
Page 31 of 70
On September 18, 2006, the petitioner filed an
The proponents against Limkaichong's qualification unsworn Declaration of Renunciation of Australian
stated that she is not a natural-born citizen because Citizenship before the Department of Immigration
her parents were Chinese citizens at the time of her and Indigenous Affairs, Canberra, Australia, which in
birth. They went on to claim that the proceedings for turn issued the Order dated September 27, 2006
the naturalization of Julio Ong Sy, her father, never certifying that she has ceased to be an Australian
attained finality due to procedural and substantial citizen.6
defects. The petitioner ran for Mayor in her hometown of
Caba, La Union in the 2007 elections. She lost in her
ISSUES: bid. She again sought elective office during the May
1) Whether or not the citizenship of Limkaichong's 10, 2010 elections this time for the position of Vice-
parents may be questioned in an election case. Mayor. She obtained the highest numbers of votes
2) Whether or not the HRET should assume and was proclaimed as the winning candidate. She
jurisdiction over the disqualification case. took her oath of office on May 13, 2010.
3) Whether or not the 10-day prescriptive period Soon thereafter, private respondents Robelito V.
under 1998 HRET Rules apply to disqualification Picar, Wilma P. Pagaduan7 and Luis M.
based on citizenship. Bautista,8 (private respondents) all registered voters
of Caba, La Union, filed separate petitions for quo
RULINGS: warranto questioning the petitioner’s eligibility
1) No. The proper proceeding in cancelling the before the RTC. The petitions similarly sought the
naturalization certificate of one person should be in petitioner’s disqualification from holding her elective
accordance with Section 18 of CA No. 473. Clearly post on the ground that she is a dual citizen and that
under the law and jurisprudence, it is the State, she failed to execute a "personal and sworn
through the Solicitor General or the representative renunciation of any and all foreign citizenship before
designated by statute, that may question in the any public officer authorized to administer an oath"
appropriate denaturalization proceeding. as imposed by Section 5(2) of R.A. No. 9225.
The petitioner denied being a dual citizen and averred
2) Yes. Limkaichong was proclaimed by the that since September 27, 2006, she ceased to be an
Provincial Board of Canvassers, she had taken her Australian citizen. She claimed that the Declaration
oath of office, and she was allowed to officially of Renunciation of Australian Citizenship she
assume office on July 23, 2007. Accordingly, the executed in Australia sufficiently complied with
House of Representatives Electoral Tribunal, and no Section 5(2), R.A. No. 9225 and that her act of
longer the COMELEC, should now assume the running for public office is a clear abandonment of
jurisdiction over the disqualification case. Section 17, her Australian citizenship.
Article VI of the 1987 Constitution and in Section The trial decision ordered by the trial court declaring
2509 of the OEC underscore the exclusivity of the Condon disqualified and ineligible to hold office of
Electoral Tribunal's jurisdiction over election vice mayor of Caba La union and nullified her
contests relating to its members. proclamation as the winning candidate.
After that the decision was appealed to the comelec,
3) No. The ten-day prescriptive period under the 1998 but the appeal was dismissed y the second division
HRET Rules does not apply to disqualification based and affirmed the decision of the trial court.
on citizenship, because qualifications for public The petitioner contends that since she ceased to be an
office are continuing requirements and must be Australian citizen on September 27, 2006, she no
possessed not only at the time of appointment or longer held dual citizenship and was only a Filipino
election or assumption of office but during the citizen when she filed her certificate of candidacy as
officer's entire tenure. early as the 2007 elections. Hence, the "personal and
v. Naturalized Citizens sworn renunciation of foreign citizenship" imposed
37. TEODORA SOBEJANA-CONDON, Petitioner, by Section 5(2) of R.A. No. 9225 to dual citizens
vs. COMMISSION ON ELECTIONS, LUIS M. seeking elective office does not apply to her.
BAUTISTA, ROBELITO V. PICAR and WILMA P.
PAGADUAN,Respondents. Issue: W/N petitioner disqualified from running for
elective office due to failure to renounce her
Facts: Australian Citizenship in accordance with Sec. 5 (2)
The petitioner is a natural-born Filipino citizen of R.A 9225
having been born of Filipino parents on August 8,
1944. On December 13, 1984, she became a Ruling:
naturalized Australian citizen owing to her marriage R.A. No. 9225 allows the retention and re-acquisition
to a certain Kevin Thomas Condon. of Filipino citizenship for natural-born citizens who
On December 2, 2005, she filed an application to re- have lost their Philippine citizenship18 by taking an
acquire Philippine citizenship before the Philippine oath of allegiance to the Republic.
Embassy in Canberra, Australia pursuant to Section 3 Natural-born citizens of the Philippines who, after the
of R.A. No. 9225 otherwise known as the effectivity of this Act, become citizens of a foreign
"Citizenship Retention and Re-Acquisition Act of country shall retain their Philippine citizenship upon
2003."5 The application was approved and the taking the aforesaid oath.
petitioner took her oath of allegiance to the Republic The oath is an abbreviated repatriation process that
of the Philippines on December 5, 2005. restores one’s Filipino citizenship and all civil and
Page 32 of 70
political rights and obligations concomitant born Filipino citizen since she was a FOUNDLING
therewith, subject to certain conditions imposed in and that her bioligical parents cannot be proved as
Section 5. Filipinos. The Comelec en banc cancelled her
Section 5, paragraph 2 provides: candidacy on the ground that she is in want of
(2) Those seeking elective public office in the citizenship and residence requirements and that she
Philippines shall meet the qualification for holding committed misrepresentation in her COC.
such public office as required by the Constitution and On CERTIORARI, the SUPREME
existing laws and, at the time of the filing of the COURT, reversed the ruling and held a vote of 9-6
certificate of candidacy, make a personal and sworn that POE is qualified as candidate for Presidency.
renunciation of any and all foreign citizenship before ISSUES:
any public officer authorized to administer an oath. (1) Whether or not Grace Poe- Llamanzares is a
On September 18, 2006, or a year before she initially natural- born Filipino citizen
sought elective public office, she filed a renunciation (2) Whether or not Poe satisfies the 10-year residency
of Australian citizenship in Canberra, Australia. requirement.
Admittedly, however, the same was not under oath HELD:
contrary to the exact mandate of Section 5(2) that the YES. GRACE POE is considerably a natural-born
renunciation of foreign citizenship must be sworn Filipino Citizen. For that, she satisfied the
before an officer authorized to administer oath. constitutional reqt that only natural-born Filipinos
The supreme court said that, the renunciation of her may run for Presidency.
Australian citizenship was invalid due to it was not (1) there is high probability that Poe’s parents are
oath before any public officer authorized to Filipinos, as being shown in her physical features
administer it rendering the act of Condon void. which are typical of Filipinos, aside from the fact that
WHEREFORE, in view of all the foregoing, the she was found as an infant in Jaro, Iloilo, a
petition is hereby DISMISSED. The Resolution municipality wherein there is 99% probability that
dated September 6, 2011 of the Commission on residents there are Filipinos, consequently providing
Elections en bane in EAC (AE). 99% chance that Poe’s bilogical parents are Filipinos.
Said probability and circumstancial evidence are
vi. STATELESS PERSONS AND FOUNDLINGS admissible under Rule 128, Sec 4 of the Rules on
Evidence.
38 . POE- LLAMANZARES vs. COMELEC (2) The SC pronounced that FOUNDLINGS are as a
GR Nos. 221697 , GR No. 221698-700 class, natural born- citizens as based on the
March 8,2016 deliberations of the 1935 Constitutional Convention,
Perez, J.: wherein though its enumeration is silent as to
FACTS: foundlings, there is no restrictive language either to
In her COC for Presidency on the May 2016 elections, definitely exclude the foundlings to be natural born
Grace Poe declared that she is a natural-born citizen citizens.
of the Philippines and that her residence up to day (3) That Foundlings are automatically conferred with
before May 9, 2016 would be 10 years and 11 months the natural-born citizenship as to the country where
counted from May 24, 2005. they are being found, as covered and supported by the
Grace Poe was born in 1968., found as newborn UN Convention Law.
infant in Jaro,Iloilo and was legally adopted by As to the residency issue, Grace Poe satisfied the
RONALD ALLAN KELLY POE (FPJ) and JESUS 10-year residency because she satisfied the
SONORA POE (SUSAN ROCES) in 1974. She requirements of ANIMUS MANENDI (intent to
immigrated to the US in 1991 after her marriage to remain permanently) coupled with ANIMUS NON
Theodore Llamanzares who was then based at the US. REVERTENDI (intent of not returning to US) in
Grace Poe then became a naturalized American acquiring a new domicile in the Philippines. Starting
citizen in 2001. May 24,2005, upon returning to the Philippines,
On December 2004, he returned to the Philippines Grace Poe presented overwhelming evidence of her
due to his father’s deteriorating medical condition, actual stay and intent to abandon permanently her
who then eventually demice on February 3,2005. She domicile in the US, coupled with her eventual
then quitted her job in the US to be with her grieving application to reacquire Filipino Citizenship under
mother and finally went home for good to the RA 9225. Hence, her candidacy for Presidency was
Philippines on MAY 24, 2005. granted by the SC.
On JULY 18, 2006, the BI granted her petition vii – Dual Citizenship and Dual Allegiance
declaring that she had reacquired her Filipino 39. Mercado v. Manzano Case Digest [G.R. No.
citizenship under RA 9225. She registered as a voter 135083. May 26, 1999]
and obtained a new Philippine Passport. FACTS:
In 2010, before assuming her post as appointes
Chairperson of the MTRCB , she renounced her Petitioner Ernesto Mercado and Eduardo Manzano
American citizenship to satisfy the RA 9225 were both candidates for Vice-Mayor of Makati in the
requirements as to Reacquistion of Filipino May 11, 1998 elections.
Citizenship. From then on, she stopped using her
American passport. Based on the results of the election, Manzano
Petitions were filed before the COMELEC to deny or garnered the highest number of votes. However, his
cancel her candidacy on the ground particularly proclamation was suspended due to the pending
among others, that she cannot be considered a natural petition for disqualification filed by Ernesto Mercado
Page 33 of 70
on the ground that he was not a citizen of the naturalization takes an oath that he renounces his
Philippines but of the United States. loyalty to any other country or government and
solemnly declares that he owes his allegiance to the
From the facts presented, it appears that Manzano is Republic of the Philippines, the condition imposed by
both a Filipino and a US citizen. law is satisfied and complied with. The
determination whether such renunciation is valid or
The Commission on Elections declared Manzano fully complies with the provisions of our
disqualified as candidate for said elective position. Naturalization Law lies within the province and is an
exclusive prerogative of our courts. The latter should
However, in a subsequent resolution of the apply the law duly enacted by the legislative
COMELEC en banc, the disqualification of the department of the Republic. No foreign law may or
respondent was reversed. Respondent was held to should interfere with its operation and application.
have renounced his US citizenship when he attained
the age of majority and registered himself as a voter The court ruled that the filing of certificate of
in the elections of 1992, 1995 and 1998. candidacy of respondent sufficed to renounce his
American citizenship, effectively removing any
Manzano was eventually proclaimed as the Vice- disqualification he might have as a dual citizen. By
Mayor of Makati City on August 31, 1998. declaring in his certificate of candidacy that he is a
Filipino citizen; that he is not a permanent resident or
Thus the present petition. immigrant of another country; that he will defend and
support the Constitution of the Philippines and bear
true faith and allegiance thereto and that he does so
ISSUE: without mental reservation, private respondent has, as
far as the laws of this country are concerned,
Whether or not a dual citizen is disqualified to hold effectively repudiated his American citizenship and
public elective office in the philippines. anything which he may have said before as a dual
citizen.

RULING: On the other hand, private respondent’s oath of


allegiance to the Philippines, when considered with
The court ruled that the phrase "dual citizenship" in the fact that he has spent his youth and adulthood,
R.A. 7160 Sec. 40 (d) and R.A. 7854 Sec. 20 must be received his education, practiced his profession as an
understood as referring to dual allegiance. Dual artist, and taken part in past elections in this country,
citizenship is different from dual allegiance. The leaves no doubt of his election of Philippine
former arises when, as a result of the application of citizenship.
the different laws of two or more states, a person is
simultaneously considered a national by the said His declarations will be taken upon the faith that he
states. Dual allegiance on the other hand, refers to a will fulfill his undertaking made under oath. Should
situation in which a person simultaneously owes, by he betray that trust, there are enough sanctions for
some positive act, loyalty to two or more states. While declaring the loss of his Philippine citizenship
dual citizenship is involuntary, dual allegiance is a through expatriation in appropriate proceedings. In
result of an individual's volition. Article IV Sec. 5 of Yu v. Defensor-Santiago, the court sustained the
the Constitution provides "Dual allegiance of citizens denial of entry into the country of petitioner on the
is inimical to the national interest and shall be dealt ground that, after taking his oath as a naturalized
with by law." citizen, he applied for the renewal of his Portuguese
passport and declared in commercial documents
Consequently, persons with mere dual citizenship do executed abroad that he was a Portuguese national. A
not fall under this disqualification. Unlike those with similar sanction can be taken against any one who, in
dual allegiance, who must, therefore, be subject to electing Philippine citizenship, renounces his foreign
strict process with respect to the termination of their nationality, but subsequently does some act
status, for candidates with dual citizenship, it should constituting renunciation of his Philippine
suffice if, upon the filing of their certificates of citizenship.
candidacy, they elect Philippine citizenship to
terminate their status as persons with dual citizenship The petition for certiorari is DISMISSED for lack of
considering that their condition is the unavoidable merit.
consequence of conflicting laws of different states.
40. [ B.M. No. 1678, December 17, 2007 ]
By electing Philippine citizenship, such candidates at PETITION FOR LEAVE TO RESUME
the same time forswear allegiance to the other country PRACTICE OF LAW, BENJAMIN M. DACANAY,
of which they are also citizens and thereby terminate PETITIONER
their status as dual citizens. It may be that, from the Facts:
point of view of the foreign state and of its laws, such Petitioner was admitted to the Philippine bar in
an individual has not effectively renounced his March 1960. He practiced law until he migrated to
foreign citizenship. That is of no moment. Canada in December 1998 to seek medical attention
for his ailments. He subsequently applied for
When a person applying for citizenship by Canadian citizenship to avail of Canada’s free
Page 34 of 70
medical aid program. His application was approved petitioner garnered the highest number of votes for
and he became a Canadian citizen in May 2004. the position of Vice Mayor. Thereafter, COMELEC
On July 14, 2006, pursuant to Republic Act (RA) finally issued its Resolution disqualifying the
9225 (Citizenship Retention and Re-Acquisition Act petitioner. Petitioner filed a Motion for
of 2003), petitioner reacquired his Philippine Reconsideration which was dismissed for lack of
citizenship. On that day, he took his oath of allegiance merit.
as a Filipino citizen before the Philippine Consulate Issue:
General in Toronto, Canada. Thereafter, he returned Whether or not petitioner has validly comply the
to the Philippines and now intends to resume his law citizenship requirement as required by law for
practice. persons seeking public office.
Ruling:
Issue: No. R.A 9225 requires that natural-born citizens of
Whether petitioner Benjamin M. Dacanay lost his the Philippines, who are already naturalized citizens
membership in the Philippine bar when he gave up his of a foreign country, must take the following oath of
Philippine citizenship allegiance to the Republic of the Philippines to
reacquire or retain their Philippine citizenship. It
Ruling: specifically provides that public office in the
The Constitution provides that the practice of all Philippines should meet the Constitutional
professions in the Philippines shall be limited to requirements and existing laws. At the time of the
Filipino citizens save in cases prescribed by law. filing of the certificate of candidacy, make a personal
Since Filipino citizenship is a requirement for and sworn renunciation of any and all foreign
admission to the bar, loss thereof terminates citizenship before any public officer authorized to
membership in the Philippine bar and, consequently, administer an oath. Filipinos reacquiring or retaining
the privilege to engage in the practice of law. In other their Philippine citizenship under Republic Act No.
words, the loss of Filipino citizenship ipso jure 9225 should not only take their oath of allegiance to
terminates the privilege to practice law in the the Republic of the Philippines, but also to explicitly
Philippines. The practice of law is a privilege denied renounce their foreign citizenship if they wish to run
to foreigners. for elective posts in the Philippines. A candidate in
The exception is when Filipino citizenship is lost by Philippine elections must only have one citizenship,
reason of naturalization as a citizen of another country that is, Philippine citizenship. This the petitioner fails
but subsequently reacquired pursuant to RA 9225. to do.
This is because “all Philippine citizens who become A candidate who failed to comply with the election
citizens of another country shall be deemed not to requirements applicable to dual citizens and received
have lost their Philippine citizenship under the the highest number of votes for an elective position
conditions of [RA 9225].” Therefore, a Filipino does not dispense with, or amount to a waiver of, the
lawyer who becomes a citizen of another country is citizenship requirement. The will of the people as
deemed never to have lost his Philippine citizenship expressed through the ballot cannot cure the
if he reacquires it in accordance with RA 9225. ineligibility, especially if they mistakenly believed
Although he is also deemed never to have terminated that the candidate was qualified. The rules on
his membership in the Philippine bar, no automatic citizenship qualifications of a candidate must be
right to resume law practice accrues. strictly applied. The application of the constitutional
Under RA 9225, if a person intends to practice the and statutory provisions on disqualification is not a
legal profession in the Philippines and he reacquires matter of popularity. The appeal was DISMISSED.
his Filipino citizenship pursuant to its provisions Comelec Resolution was AFFIRMED and petitioner
“(he) shall apply with the proper authority for a was DISQUALIFIED.
license or permit to engage in such practice.
viii- Natural Born citizen and Public Office
41. Jacot v. Comelec 42. Arnado vs COMELEC
G.R. No. 179848 November 27, 2008 G.R. No. 210164 – Political Law – Election Law –
Republic Act No. 9225 – Qualifications of Local
Facts: Elective Candidates – Citizenship Requirements –
Petitioner Jacot assails Comelec Resolution affirming Dual Allegiance
his disqualification from running for the position of Remedial Law – Formal Offer of Evidence – Evidence
Vice-Mayor for failure to comply the citizenship not Offered
requirement. Petitioner was a natural born citizen of Rommel Arnado was a natural-born Filipino. Later,
the Philippines, who became a naturalized U.S however, he became an American citizen.
citizen. Petitioner sought to reacquire his Philippine On July 10, 2008, he re-acquired his Filipino
citizenship under R.A No. 9225. Six months after, citizenship by executing an oath of allegiance to the
petitioner filed his Certificate of Candidacy. Philippines.
Respondent Dal filed a Petition for On April 3, 2009, he executed an affidavit renouncing
Disqualification before the COMELEC against his American citizenship.
petitioner arguing that the latter failed to renounce his On November 30, 2009, he filed a certificate of
US citizenship, as required under Section 5(2) of candidacy (COC) for mayor of Kauswagan, Lanao
Republic Act No. 9225 for holding such public office del Norte for the May 10, 2010 elections.
as required by the Constitution and existing A rival candidate (Linog Balua) then filed a
laws. When the local and national elections were held disqualification case against Arnado on the ground
Page 35 of 70
that Arnado used his US passport after renouncing his recanted or withdrawn when he again used his US
US citizenship in April 2009. It was argued that such passport.
act of using a US passport constitutes dual allegiance In fact, Arnado did not controvert the allegations that
and that is a ground for disqualification under the he used his US passport in January 2010 and March
Local Government Code. In short, it was argued that 2010. As such, he remained a US citizen and is
Arnado remained a US citizen. therefore disqualified to run for public office.
In his defense, Arnado argued that he is qualified to What Arnado could have done, for the purposes of
run for public office because he complied with the running in the 2013 elections, was to renounce again
requirements of Republic Act No. 9225 which (for the third time) his US citizenship. But he never
provides that a former Filipino citizen may run for did that hence he was rightfully disqualified in the
elective public office if (1) they meet the 2013 elections too.
qualifications for the elective office they desire, and Note also that assuming that Arnado never used his
(2) make a personal and sworn renunciation of any US passport in January 2010 and March 2010, he is
and all foreign citizenships – which must be done still disqualified.
before the filing of the COC. Arnado averred that his use of his US passport prior
Arnado explained that his use of his US passport after to November 2009 was cured when he again made a
April 2009 was because of the fact that he did not second renunciation of his US citizenship on
know yet that he had been issued already a Philippine November 30, 2009. However, the Affidavit of
passport; that when he received said Philippine Renunciation he offered in court during trial was a
passport, he used it since then; that at any rate, mere photocopy of the original. Under the Best
Arnado, on November 30, 2009, again executed an Evidence Rule (Section 3, Rule 130, Revised Rules of
Affirmation of Renunciation with Oath of Allegiance Court), the original must be presented unless the same
before a notary public. is lost. In this case, the original was never alleged to
Balua however presented proof that Arnado again have been lost. Further, the said Affidavit was being
used his US passport in January 2010 and in March used belatedly by Arnado. In fact, it was never
2010. formally offered. Under Section 34, Rule 132 of the
Eventually, the Commission on Elections disqualified Revised Rules of Court, “The court shall consider no
Arnado, who won the 2010 elections, and declared evidence which has not been formally offered.”
another rival candidate as the rightful mayor. This
was affirmed by the Supreme Court (G.R. No. B. MODES OF ACQUIRING CITIZENSHIP
195649). i. JUS SOLI AND JUS SANGUINES
Later, on October 1, 2012, Arnado filed his COC for 44. G.R. No. 210164 – Political Law – Election Law
mayor for the May 2013 elections. Another rival – Republic Act No. 9225 – Qualifications of Local
candidate (Casan Maquiling) filed a petition to Elective Candidates – Citizenship Requirements –
disqualify Arnado based on the ruling in G.R. No. Dual Allegiance
195649. While the case was pending, Arnado won Remedial Law – Formal Offer of Evidence – Evidence
the 2013 elections as he even acquired 84% of the not Offered
votes cast for mayor in Kauswagan. Rommel Arnado was a natural-born Filipino. Later,
Later however, the COMELEC disqualified Arnado however, he became an American citizen.
from running in the May 2013 Elections and his On July 10, 2008, he re-acquired his Filipino
declaration as Mayor of Kauswagan was voided. citizenship by executing an oath of allegiance to the
Arnado sued the COMELEC as he argued that the Philippines.
COMELEC acted with grave abuse of discretion. He On April 3, 2009, he executed an affidavit renouncing
averred that he was able to comply with the his American citizenship.
requirements of RA 9225; and that his On November 30, 2009, he filed a certificate of
disqualification only disenfranchised 84% of the candidacy (COC) for mayor of Kauswagan, Lanao
Kauswagan voters. del Norte for the May 10, 2010 elections.
ISSUE: Whether or not the arguments raised by A rival candidate (Linog Balua) then filed a
Arnado are tenable. disqualification case against Arnado on the ground
HELD: No. that Arnado used his US passport after renouncing his
1. Firstly, the fact that he obtained a landslide victory US citizenship in April 2009. It was argued that such
does not override the requirements set by law. The act of using a US passport constitutes dual allegiance
fact that he garnered 84% of the total votes cast in and that is a ground for disqualification under the
Kauswagan cannot override the constitutional and Local Government Code. In short, it was argued that
statutory requirements for qualifications and Arnado remained a US citizen.
disqualifications. Election victory cannot be used as a In his defense, Arnado argued that he is qualified to
magic formula to bypass election eligibility run for public office because he complied with the
requirements; otherwise, certain provisions of laws requirements of Republic Act No. 9225 which
pertaining to elections will become toothless. provides that a former Filipino citizen may run for
2. The COMELEC did not act with grave abuse of elective public office if (1) they meet the
discretion when it disqualified Arnado. Arnado failed qualifications for the elective office they desire, and
to comply with the requirements of RA 9225. (2) make a personal and sworn renunciation of any
Although he did swear allegiance to the Philippines and all foreign citizenships – which must be done
and renounced his US citizenship prior to filing his before the filing of the COC.
COC in November 2009, such acts were deemed Arnado explained that his use of his US passport after
April 2009 was because of the fact that he did not
Page 36 of 70
know yet that he had been issued already a Philippine Renunciation he offered in court during trial was a
passport; that when he received said Philippine mere photocopy of the original. Under the Best
passport, he used it since then; that at any rate, Evidence Rule (Section 3, Rule 130, Revised Rules of
Arnado, on November 30, 2009, again executed an Court), the original must be presented unless the same
Affirmation of Renunciation with Oath of Allegiance is lost. In this case, the original was never alleged to
before a notary public. have been lost. Further, the said Affidavit was being
Balua however presented proof that Arnado again used belatedly by Arnado. In fact, it was never
used his US passport in January 2010 and in March formally offered. Under Section 34, Rule 132 of the
2010. Revised Rules of Court, “The court shall consider no
Eventually, the Commission on Elections disqualified evidence which has not been formally offered.”
Arnado, who won the 2010 elections, and declared
another rival candidate as the rightful mayor. This ii- Judicial Declaration of Phil Citizenship
was affirmed by the Supreme Court (G.R. No.
195649). 45. REPUBLIC V. SAGUN (G.R. NO. 187567;
Later, on October 1, 2012, Arnado filed his COC for FEBRUARY 15, 2012)
mayor for the May 2013 elections. Another rival
candidate (Casan Maquiling) filed a petition to FACTS: Nora Fe Sagun is the legitimate child of
disqualify Arnado based on the ruling in G.R. No. Albert S. Chan, a Chinese national, and Marta
195649. While the case was pending, Arnado won Borromeo, a Filipino citizen. She was born on August
the 2013 elections as he even acquired 84% of the 8, 1959 in Baguio City and did not elect Philippine
votes cast for mayor in Kauswagan. citizenship upon reaching the age of majority. In
Later however, the COMELEC disqualified Arnado 1992, at the age of 33 and after getting married to
from running in the May 2013 Elections and his Alex Sagun, she executed an Oath of Allegiance to
declaration as Mayor of Kauswagan was voided. the Republic of the Philippines. Said document was
Arnado sued the COMELEC as he argued that the notarized by Atty. Cristeta Leungon but was not
COMELEC acted with grave abuse of discretion. He recorded and registered with the Local Civil Registrar
averred that he was able to comply with the of Baguio City.
requirements of RA 9225; and that his
disqualification only disenfranchised 84% of the Sometime in September 2005, respondent applied for
Kauswagan voters. a Philippine passport. Her application was denied due
ISSUE: Whether or not the arguments raised by to the citizenship of her father and there being no
Arnado are tenable. annotation on her birth certificate that she has elected
HELD: No. Philippine citizenship. Consequently, she sought a
1. Firstly, the fact that he obtained a landslide victory judicial declaration of her election of Philippine
does not override the requirements set by law. The citizenship averring that she was raised as a Filipino
fact that he garnered 84% of the total votes cast in and she is a registered voter of Precinct No. 0419A of
Kauswagan cannot override the constitutional and Barangay Manuel A. Roxas in Baguio City and had
statutory requirements for qualifications and voted in local and national elections as shown in the
disqualifications. Election victory cannot be used as a Voter Certification. She asserted that by virtue of her
magic formula to bypass election eligibility positive acts, she has effectively elected Philippine
requirements; otherwise, certain provisions of laws citizenship and such fact should be annotated on her
pertaining to elections will become toothless. record of birth so as to entitle her to the issuance of a
2. The COMELEC did not act with grave abuse of Philippine passport.
discretion when it disqualified Arnado. Arnado failed
to comply with the requirements of RA 9225. After conducting a hearing, the trial court rendered
Although he did swear allegiance to the Philippines the assailed Decision on April 3, 2009 granting the
and renounced his US citizenship prior to filing his petition and declaring respondent a Filipino citizen.
COC in November 2009, such acts were deemed
recanted or withdrawn when he again used his US Upon payment of the required fees, the Local Civil
passport. Registrar of Baguio City is hereby directed to
In fact, Arnado did not controvert the allegations that annotate [on] her birth certificate, this judicial
he used his US passport in January 2010 and March declaration of Filipino citizenship of said petitioner.
2010. As such, he remained a US citizen and is
therefore disqualified to run for public office. Petitioner, through the OSG, directly filed the instant
What Arnado could have done, for the purposes of recourse via a petition for review on certiorari before
running in the 2013 elections, was to renounce again us. Petitioner points out that while respondent
(for the third time) his US citizenship. But he never executed an oath of allegiance before a notary public,
did that hence he was rightfully disqualified in the there was no affidavit of her election of Philippine
2013 elections too. citizenship. Additionally, her oath of allegiance
Note also that assuming that Arnado never used his which was not registered with the nearest local civil
US passport in January 2010 and March 2010, he is registry was executed when she was already 33 years
still disqualified. old or 12 years after she reached the age of majority.
Arnado averred that his use of his US passport prior
to November 2009 was cured when he again made a ISSUES:
second renunciation of his US citizenship on
November 30, 2009. However, the Affidavit of Is an action or proceeding for judicial declaration
Page 37 of 70
of Philippine citizenship procedurally and Philippine citizenship should not be understood as
jurisdictionally permissible? having a curative effect on any irregularity in the
acquisition of citizenship for those covered by the
Is an election of Philippine citizenship, made 1935 Constitution. If the citizenship of a person was
twelve (12) years after reaching the age of subject to challenge under the old charter, it remains
majority, considered to have been made within a subject to challenge under the new charter even if the
reasonable time as interpreted by jurisprudence? judicial challenge had not been commenced before
HELD: Under our laws, there can be no action or the effectivity of the new Constitution.
proceeding for the judicial declaration of the
citizenship of an individual. Courts of justice exist for Based on the foregoing circumstances, respondent
settlement of justiciable controversies, which imply a clearly failed to comply with the procedural
given right, legally demandable and enforceable, an requirements for a valid and effective election of
act or omission violative of said right, and a remedy, Philippine citizenship. Respondent cannot assert that
granted or sanctioned by law, for said breach of right. the exercise of suffrage and the participation in
As an incident only of the adjudication of the rights election exercises constitutes a positive act of election
of the parties to a controversy, the court may pass of Philippine citizenship since the law specifically
upon, and make a pronouncement relative to their lays down the requirements for acquisition of
status. Otherwise, such a pronouncement is beyond citizenship by election. The mere exercise of suffrage,
judicial power. continuous and uninterrupted stay in the Philippines,
and other similar acts showing exercise of Philippine
Clearly, it was erroneous for the trial court to make a citizenship cannot take the place of election of
specific declaration of respondents Filipino Philippine citizenship. Hence, respondent cannot now
citizenship as such pronouncement was not within the be allowed to seek the intervention of the court to
court's competence. confer upon her Philippine citizenship when clearly
she has failed to validly elect Philippine citizenship.
As to the propriety of respondent's petition seeking a As we held in Ching, the prescribed procedure in
judicial declaration of election of Philippine electing Philippine citizenship is certainly not a
citizenship, it is imperative that we determine tedious and painstaking process. All that is required
whether respondent is required under the law to make of the elector is to execute an affidavit of election of
an election and if so, whether she has complied with Philippine citizenship and, thereafter, file the same
the procedural requirements in the election of with the nearest civil registry. Having failed to
Philippine citizenship. comply with the foregoing requirements, respondents
petition before the trial court must be denied. (Re:
When respondent was born on August 8, 1959, the Application For Admission to the Philippine Bar.
governing charter was the 1935 Constitution, which Vicente D. Ching, Bar Matter No. 914, October 1,
declares as citizens of the Philippines those whose 1999, 316 SCRA 1, 7-8.)
mothers are citizens of the Philippines and elect
Philippine citizenship upon reaching the age of GRANTED
majority. Sec. 1, Art. IV of the 1935 Constitution
reads: iii- Philippine Citizenship by Naturalization
COMMONWEALTH ACT No. 473
Section 1. The following are citizens of the AN ACT TO PROVIDE FOR THE
Philippines: ACQUISITION OF PHILIPPINE CITIZENSHIP
x x x x BY NATURALIZATION, AND TO REPEAL
ACTS NUMBERED TWENTY-NINE
(4) Those whose mothers are citizens of the HUNDRED AND TWENTY-SEVEN AND
Philippines and, upon reaching the age of majority, THIRTY-FOUR HUNDRED AND FORTY-
elect Philippine citizenship. EIGHT.
Be it enacted by the National Assembly of the
Under Article IV, Section 1(4) of the 1935 Philippines:
Constitution, the citizenship of a legitimate child born Section 1. Title of Act. – This Act shall be known and
of a Filipino mother and an alien father followed the may be cited as the "Revised Naturalization Law."
citizenship of the father, unless, upon reaching the age Section 2. Qualifications. – Subject to section four of
of majority, the child elected Philippine citizenship. this Act, any person having the following
The right to elect Philippine citizenship was qualifications may become a citizen of the Philippines
recognized in the 1973 Constitution when it provided by naturalization:
that [t]hose who elect Philippine citizenship pursuant First. He must be not less than twenty-one years of
to the provisions of the Constitution of nineteen age on the day of the hearing of the petition;
hundred and thirty-five are citizens of the Philippines. Second. He must have resided in the Philippines for a
Likewise, this recognition by the 1973 Constitution continuous period of not less than ten years;
was carried over to the 1987 Constitution which states Third. He must be of good moral character and
that [t]hose born before January 17, 1973 of Filipino believes in the principles underlying the Philippine
mothers, who elect Philippine citizenship upon Constitution, and must have conducted himself in a
reaching the age of majority are Philippine citizens. It proper and irreproachable manner during the entire
should be noted, however, that the 1973 and 1987 period of his residence in the Philippines in his
Constitutional provisions on the election of
Page 38 of 70
relation with the constituted government as well as citizenship, the applicant for Philippine citizenship
with the community in which he is living. shall file with the Bureau of Justice4 a declaration
Fourth. He must own real estate in the Philippines under oath that it is bona fide his intention to become
worth not less than five thousand pesos, Philippine a citizen of the Philippines. Such declaration shall set
currency, or must have some known lucrative trade, forth name, age, occupation, personal description,
profession, or lawful occupation; place of birth, last foreign residence and allegiance,
Fifth. He must be able to speak and write English or the date of arrival, the name of the vessel or aircraft,
Spanish and any one of the principal Philippine if any, in which he came to the Philippines, and the
languages; and place of residence in the Philippines at the time of
Sixth. He must have enrolled his minor children of making the declaration. No declaration shall be valid
school age, in any of the public schools or private until lawful entry for permanent residence has been
schools recognized by the Office of Private established and a certificate showing the date, place,
Education1 of the Philippines, where the Philippine and manner of his arrival has been issued. The
history, government and civics are taught or declarant must also state that he has enrolled his
prescribed as part of the school curriculum, during the minor children, if any, in any of the public schools or
entire period of the residence in the Philippines private schools recognized by the Office of Private
required of him prior to the hearing of his petition for Education5 of the Philippines, where Philippine
naturalization as Philippine citizen. history, government, and civics are taught or
Section 3. Special qualifications. The ten years of prescribed as part of the school curriculum, during the
continuous residence required under the second entire period of the residence in the Philippines
condition of the last preceding section shall be required of him prior to the hearing of his petition for
understood as reduced to five years for any petitioner naturalization as Philippine citizen. Each declarant
having any of the following qualifications: must furnish two photographs of himself.
Having honorably held office under the Government Section 6. Persons exempt from requirement to make
of the Philippines or under that of any of the a declaration of intention. – Persons born in the
provinces, cities, municipalities, or political Philippines and have received their primary and
subdivisions thereof; secondary education in public schools or those
Having established a new industry or introduced a recognized by the Government and not limited to
useful invention in the Philippines; any race or nationality, and those who have
Being married to a Filipino woman; resided continuously in the Philippines for a
Having been engaged as a teacher in the Philippines period of thirty years or more before filing their
in a public or recognized private school not application, may be naturalized without having to
established for the exclusive instruction of children of make a declaration of intention upon complying with
persons of a particular nationality or race, in any of the other requirements of this Act. To such
the branches of education or industry for a period of requirements shall be added that which
not less than two years; establishes that the applicant has given primary
Having been born in the Philippines. and secondary education to all his children in the
Section 4. Who are disqualified. - The following public schools or in private schools recognized by
cannot be naturalized as Philippine citizens: the Government and not limited to any race or
Persons opposed to organized government or nationality. The same shall be understood
affiliated with any association or group of persons applicable with respect to the widow and minor
who uphold and teach doctrines opposing all children of an alien who has declared his intention to
organized governments; become a citizen of the Philippines, and dies before
Persons defending or teaching the necessity or he is actually naturalized.6
propriety of violence, personal assault, or Section 7. Petition for citizenship. – Any person
assassination for the success and predominance of desiring to acquire Philippine citizenship shall file
their ideas; with the competent court, a petition in triplicate,
Polygamists or believers in the practice of polygamy; accompanied by two photographs of the petitioner,
Persons convicted of crimes involving moral setting forth his name and surname; his present and
turpitude; former places of residence; his occupation; the place
Persons suffering from mental alienation or incurable and date of his birth; whether single or married and
contagious diseases; the father of children, the name, age, birthplace and
Persons who, during the period of their residence in residence of the wife and of each of the children; the
the Philippines, have not mingled socially with the approximate date of his or her arrival in the
Filipinos, or who have not evinced a sincere desire to Philippines, the name of the port of debarkation, and,
learn and embrace the customs, traditions, and ideals if he remembers it, the name of the ship on which he
of the Filipinos; came; a declaration that he has the qualifications
Citizens or subjects of nations with whom the United required by this Act, specifying the same, and that he
States 2and the Philippines are at war, during the is not disqualified for naturalization under the
period of such war; provisions of this Act; that he has complied with the
Citizens or subjects of a foreign country other than the requirements of section five of this Act; and that he
United States 3whose laws do not grant Filipinos the will reside continuously in the Philippines from the
right to become naturalized citizens or subjects date of the filing of the petition up to the time of his
thereof. admission to Philippine citizenship. The petition must
Section 5. Declaration of intention. – One year prior be signed by the applicant in his own handwriting and
to the filing of his petition for admission to Philippine be supported by the affidavit of at least two credible
Page 39 of 70
persons, stating that they are citizens of the of the Court, no appeal has been filed, or if, upon
Philippines and personally know the petitioner to be appeal, the decision of the court has been confirmed
a resident of the Philippines for the period of time by the Supreme Court,14 and the said decision has
required by this Act and a person of good repute and become final, the clerk of the court which heard the
morally irreproachable, and that said petitioner has in petition shall issue to the petitioner a naturalization
their opinion all the qualifications necessary to certificate which shall, among other things, state the
become a citizen of the Philippines and is not in any following: The file number of the petition, the number
way disqualified under the provisions of this Act. The of the naturalization certificate, the signature of the
petition shall also set forth the names and post-office person naturalized affixed in the presence of the clerk
addresses of such witnesses as the petitioner may of the court, the personal circumstances of the person
desire to introduce at the hearing of the case. The naturalized, the dates on which his declaration of
certificate of arrival, and the declaration of intention intention and petition were filed, the date of the
must be made part of the petition. decision granting the petition, and the name of the
Section 8. Competent court.—The Court of First judge who rendered the decision. A photograph of the
Instance of the province in which the petitioner has petitioner with the dry seal affixed thereto of the court
resided at least one year immediately preceding the which granted the petition, must be affixed to the
filing of the petition shall have exclusive original certificate.
jurisdiction to hear the petition. Before the naturalization certificate is issued, the
Section 9. Notification and appearance.— petitioner shall, in open court, take the following oath:
Immediately upon the filing of a petition, it shall be "I, . . . . . . . . . . . . . . . . . . . . . . . . . . . . . . . . . . . ,
the duty of the clerk of the court to publish the same solemnly swear that I renounce absolutely and forever
at petitioner's expense, once a week for three all allegiance and fidelity to any foreign prince,
consecutive weeks, in the Official Gazette, and in one potentate, state or sovereignty, and particularly to the
of the newspapers of general circulation in the . . . . . . . . . . . . . . . . . . of which at this time I am a
province where the petitioner resides, and to have subject or citizen; that I will support and defend the
copies of said petition and a general notice of the Constitution of the Philippines and that I will obey the
hearing posted in a public and conspicuous place in laws, legal orders and decrees promulgated by the
his office or in the building where said office is duly constituted authorities of the Commonwealth15of
located, setting forth in such notice the name, the Philippines; [and I hereby declare that I recognize
birthplace and residence of the petitioner, the date and and accept the supreme authority of the United States
place of his arrival in the Philippines, the names of of America in the Philippines and will maintain true
the witnesses whom the petitioner proposes to faith and allegiance thereto;16 and that I impose this
introduce in support of his petition, and the date of the obligation upon myself voluntarily without mental
hearing of the petition, which hearing shall not be reservation or purpose of evasion.
held within ninety days from the date of the last "So help me God."
publication of the notice. The clerk shall, as soon as Section 13. Record books.—The clerk of the court
possible, forward copies of the petition, the sentence, shall keep two books; one in which the petition and
the naturalization certificate, and other pertinent data declarations of intention shall be recorded in
to the Department of the Interior, 7 the Bureau of chronological order, noting all proceedings thereof
Justice,8 the Provincial Inspector9 of the Philippine from the filing of the petition to the final issuance of
Constabulary of the province and the justice of the the naturalization certificate; and another, which shall
peace10 of the municipality wherein the petitioner be a record of naturalization certificates each page of
resides. which shall have a duplicate which shall be duly
Section 10. Hearing of the petition.—No petition attested by the clerk of the court and delivered to the
shall be heard within the thirty days preceding any petitioner.
election. The hearing shall be public, and the
Solicitor-General, either himself or through his REPUBLIC ACT NO. 9139 June 08, 2001
delegate or the provincial fiscal concerned, shall AN ACT PROVIDING FOR THE
appear on behalf of the Commonwealth11 of the ACQUISITION OF PHILIPPINE CITIZENSHIP
Philippines at all the proceedings and at the hearing. FOR CERTAIN ALIENS BY
If, after the hearing, the court believes, in view of the ADMINISTRATIVE NATURALIZATION AND
evidence taken, that the petitioner has all the FOR OTHER PURPOSES
qualifications required by, and none of the Be it enacted by the Senate and the House of
disqualifications specified in this Act and has Representatives of the Philippines in Congress
complied with all requisites herein established, it assembled:
shall order the proper naturalization certificate to be Section 1. Short Title. - This Act shall be known as
issued and the registration of the said naturalization "The Administrative Naturalization Law of 2000."
certificate in the proper civil registry as required in Section 2. Declaration of Policy. - The State shall
section ten of Act Numbered Three thousand seven control and regulate the admission and integration of
hundred and fifty-three.12 aliens into its territory and body politic including the
Section 11. Appeal.—The final sentence may, at the grant of citizenship to aliens. Towards this end, aliens
instance of either of the parties, be appealed to the born and residing in the Philippines may be granted
Supreme Court.13 Philippine citizenship by administrative proceedings
Section 12. Issuance of the Certificate of subject to certain requirements dictated by national
Naturalization.—If, after the lapse of thirty days from security and interest.
and after the date on which the parties were notified
Page 40 of 70
Section 3. Qualifications. - Subject to the provisions (h) Citizens or subjects of a foreign country whose
of the succeeding section, any person desiring to avail laws do not grant Filipinos the right to be naturalized
of the benefits of this Act must meet the following citizens or subjects thereof.
qualifications: Section 5. Petition for Citizenship. - (1) Any person
(a) The applicant must be born in the Philippines and desiring to acquire Philippine citizenship under this
residing therein since birth; Act shall file with the Special Committee on
(b) The applicant must not be less than eighteen (18) Naturalization created under Section 6 hereof, a
years of age, at the time of filing of his/her petition; petition of five (5) copies legibly typed and signed,
(c) The applicant must be of good moral character and thumbmarked and verified by him/her, with the
believes in the underlying principles of the latter's passport-sized photograph attached to each
Constitution, and must have conducted copy of the petition, and setting forth the following:
himself/herself in a proper and irreproachable manner (a) The petitioner's name and surname, and any other
during his/her entire period of residence in the name he/she has used or by which he/she is known;
Philippines in his relation with the duly constituted (b) The petitioner's present and former places of
government as well as with the community in which residence;
he/she is living; (c) The petitioner's place and date of birth, the names
(d) The applicant must have received his/her primary and citizenship of his/her parents and their residences;
and secondary education in any public school or (d) The petitioner's trade, business, profession or
private educational institution dully recognized by the occupation, and if married, also that of his/her spouse;
Department of Education, Culture and Sports, where (e) Whether the petitioner is single or married or
Philippine history, government and civics are taught his/her marriage is annulled. If married, petitioner
and prescribed as part of the school curriculum and shall state the date and place of his/her marriage, and
where enrollment is not limited to any race or the name, date of birth, birthplace, citizenship and
nationality: Provided, That should he/she have minor residence of his/her spouse; and if his marriage is
children of school age, he/she must have enrolled annulled, the date of decree of annulment of marriage
them in similar schools; and the court which granted the same;
(e) The applicant must have a known trade, business, (f) If the petitioner has children, the name, date and
profession or lawful occupation, from which he/she birthplace and residences of his/her children ;
derives income sufficient for his/her support and if (g) A declaration that the petitioner possesses all the
he/she is married and/or has dependents, also that of qualifications and none of the disqualifications under
his/her family: Provided, however, That this shall not this Act;
apply to applicants who are college degree holders but (h) A declaration that the petitioner shall never be a
are unable to practice their profession because they public charge; and
are disqualified to do so by reason of their citizenship; (i) A declaration that it is the petitioner's true and
(f) The applicant must be able to read, write and speak honest intention to acquire Philippine citizenship and
Filipino or any of the dialects of the Philippines; and to renounce absolutely and forever any prince,
(g) The applicant must have mingled with the potentate, State or sovereign, and particularly the
Filipinos and evinced a sincere desire to learn and country of which the applicant is a citizen or subject.
embrace the customs, traditions and ideals of the (2) The application shall be accompanied by:
Filipino people. (a) Duplicate original or certified photocopies of
Section 4. Disqualifications, - The following are not petitioner's birth certificate;
qualified to be naturalized as Filipino citizens under (b) Duplicate original or certified photocopies of
this Act: petitioner's alien certificate of registration and native
(a) Those opposed to organized government or born certificate of residence;
affiliated with any association of group of persons (c) Duplicate original or certified photocopies of
who uphold and teach doctrines opposing all petitioner's marriage certified, if married, or the death
organized governments; certificate of his spouse, if widowed, or the court
(b) Those defending or teaching the necessity of or decree annulling his marriage, if such was the fact;
propriety of violence, personal assault or (d) Duplicate original or certified photocopies of birth
assassination for the success or predominance of their certificates, alien certificate of registration or native
ideas; born certificate of residence if any, of petitioner's
(c) Polygamists or believers in the practice of minor children, wherever applicable;
polygamy; (e) Affidavit of financial capacity by the petitioner,
(d) Those convicted of crimes involving moral and sworn statements on the good moral character of
turpitude; the petitioner by at least two (2) Filipino citizens of
(e) Those suffering from mental alienation or good reputation in his/her place of residence stating
incurable contagious diseases; that they have personally known the petitioner for at
(f) Those who, during the period of their residence in least a period of ten (10) years and that said petitioner
the Philippines, have not mingled socially with has in their own opinion all the qualifications
Filipinos, or who have not evinced a sincere desire to necessary to become a citizen of the Philippines and
learn and embrace the customs, traditions and ideals is not in any way disqualified under the provisions of
of the Filipinos; this Act;
(g) Citizens or subjects with whom the Philippines is (f) A medical certificate that petitioner is not a user of
at war, during the period of such war; and prohibited drugs or otherwise a drug dependent and
that he/she is not afflicted with acquired immune
deficiency syndrome (AIDS);
Page 41 of 70
(g) School diploma and transcript of records of the of the petition and its annexes, and to determine the
petitioner in the schools he attended in the truthfulness of the statements and declarations made
Philippines. Should the petitioner have minor in the petition and its annexes.
children, a certification that his children are enrolled If the Committee shall have received any information
in a school where Philippine history, government and adverse to the petition, the Committee shall allow the
civics are taught and are part of the curriculum; and petitioner to answer, explain or refute the
(h) If gainfully employed, the income tax return for information.
the past three (3) years. Thereafter, if the Committee believes, in view of the
Section 6. Special Committee on Naturalization. - facts before it, that the petitioner has all the
There shall be constituted a Special Committee on qualifications and none of the disqualifications
Naturalization herein referred to as the "Committee", required for Philippine citizenship under this Act, it
with the Solicitor General as chairman, the Secretary shall approve the petition and henceforth, notify the
of Foreign Affairs, or his representative, and the petitioner of the fact of such approval. Otherwise, the
National Security Adviser, as members, with the Committee shall disapprove the same.
power to approve, deny or reject applications for Section 9. Decree of Naturalization and
naturalization as provided in this Act. Naturalization Processing Fee. -Within thirty (30)
The Committee shall meet, as often as practicable, to days from the receipt of the notice of the approval of
consider applications for naturalization. For this his/her petition, the applicant shall pay to the
purpose, the chairman and members shall receive an Committee a naturalization fee of One hundred
honorarium of Two thousand pesos (P2,000.00) and thousand pesos (P100,000.00) payable as follows:
One thousand five hundred pesos (P1,500.00), Fifty thousand pesos (P50,000.00) upon the approval
respectively, per meeting attended. of the petition and Fifty thousand pesos (P50,000.00)
Section 7. Powers/Functions of the Special upon the taking of the oath of allegiance to the
Committee on Naturalization. - An alien who Republic of the Philippines, forthwith, a certificate of
believes that he has all the qualifications, and none of naturalization shall be issued. Within sixty (60) days
the disqualifications, may file an application for from the issuance of the certificate, the petitioner
naturalization with the secretariat of the Special shall take an oath of allegiance in the proper forum
Committee on Naturalization, and a processing fee of upon proof of payment of the required naturalization
Forty thousand pesos (P40,000.00). Thereafter, the processing fee and certificate of naturalization.
petition shall be stamped to indicate the date of filing Should the applicant fail to take the abovementioned
and a corresponding docket number. Within fifteen oath of allegiance within said period of time, the
(15) days from the receipt of the petition, the approval of the petition shall be deemed abandoned.
Committee shall determine whether the petition is Section 10. Duty of the Bureau of Immigration. -
complete in substance and in form. If such petition is Within five (5) days after the applicant has taken his
complete, the Committee shall immediately publish oath of allegiance as required in the preceding
pertinent portions of the petition indicating the name, section, the BI shall forward a copy of the petitioner's
qualifications and other personal circumstances of the oath to the proper local civil registrar. Thereafter, the
applicant, once a week for three (3) consecutive BI shall cancel the alien certificates of registration of
weeks in a newspaper of general circulation, and have the applicant.
copies of the petition posted in any public or Section 11. Status of Alien Wife and Minor
conspicuous area. The Committee shall immediately Children. - After the approval of the petition for
furnish the Department of Foreign Affairs (DFA), the administrative naturalization in cancellation of
Bureau of Immigration (BI), the civil registrar of the applicant's alien certificate of registration, applicant's
petitioner's place of residence and tile National alien lawful wife and minor children may file a
Bureau of Investigation (NBI) copies of the petition petition for cancellation of their alien certificates of
and its supporting documents. These agencies shall registration with the Committee subject to the
have copies of the petition posted in any public or payment of the filing fee of Twenty thousand pesos
conspicuous area in their buildings, offices and (P20,000.00) and naturalization fee of Forty thousand
premises, and shall, within thirty (30) days from the pesos (P40,000.00) payable as follows: Twenty
receipt of the petition, submit to the Committee a thousand pesos (P20,000.00) upon the approval of the
report stating whether or not petitioner has any petition and Twenty thousand pesos (P20,000.00)
derogatory record on file or any such relevant and upon the taking of the oath of allegiance to the
material information which might be adverse to Republic of the Philippines.
petitioner's application for citizenship. Section 12. Status of Alien Husband and Minor
If the petition is found by the Committee to be Children. - If the applicant is a married woman, the
wanting in substance and form, the petition shall be approval of her petition for administrative
dismissed without prejudice. naturalization will not benefit her alien husband but
Section 8. Approval or Disapproval of the Petition. - her minor children may file a petition for cancellation
Within sixty (60) days from receipt of the report of of their alien certificates of registration with the BI
the agencies which were furnished a copy of the subject to the requirements of existing laws.
petition or the date of the last publication of the Section 13. Cancellation of the Certificate of
petition, whichever comes in later, the Committee Naturalization. - The Special Committee may cancel
shall consider and review all relevant and material certificates of naturalization issued under this Act in
information it has received pertaining to the petition, the following cases:
and may, for the purpose call the petitioner for (a) If it finds that the naturalized person or his duly
interview to ascertain his/her identity, the authenticity authorized representative made any false statement or
Page 42 of 70
misrepresentation or committed any violation of law, the Philippines Law Center and another twenty-five
rules and regulations in connection with the petition percent (25%) shall be allotted for the publication of
for naturalization, or if he otherwise obtains the Journal of the House of Representatives. Said
Philippine citizenship fraudulently or illegally, the amount shall be treated as receipts automatically
certificate of naturalization shall be cancelled; appropriated.
(b) If the naturalized person or his wife, or any or his Section 17. Implementing Rules and Regulations. -
minor children who acquire Filipino citizenship by The Special Committee on Naturalization is hereby
virtue of his naturalization shall, within five (5) years authorized to promulgate such rules and regulations
next following the grant of Philippine citizenship, as may be needed for the proper implementation of
establish permanent residence in a foreign country, the provisions of this Act.
that individual's certificate of naturalization or Section 18. Repealing Clause. -All provisions of
acquired citizenship shall be cancelled or existing laws, orders, decrees, rules and regulations
revoked: Provided, That the fact of such person's contrary to or inconsistent with this Act are hereby
remaining for more than one (1) year in his country repealed or modified accordingly.
of origin, or two (2) years in any foreign country, shall Section 19. Separability CIause. - If any part, section
be considered prima facie evidence of intent to or provision of this Act is declared invalid or
permanently reside therein; unconstitutional, the part, section or provision not
(c) If the naturalized person or his wife or child with affected thereby shall continue to be in force and
acquired citizenship allows himself or herself to be effect.
used as a dummy in violation of any constitutional or Section 20. Effectivity Clause. - This Act shall take
legal provision requiring Philippine citizenship as a effect after fifteen (15) days following its publication
condition for the exercise, use or enjoyment of a right, in at least two (2) newspapers of general circulation.
franchise or privilege, the certificate of naturalization Approved,
or acquired citizenship shall be cancelled or revoked; (Sgd)
and
(d) If the naturalized person or his wife or child with
acquired citizenship commits any act inimical to AQUILINO Q. PIMENTEL JR.
national security, the certificate of naturalization or President of the Senate
acquired citizenship shall be cancelled or revoked. 46.
In case the naturalized person holds any hereditary Rep vs ONG - 2012
title, or belong to any order of nobility, he shall make
an express renunciation of his title or membership in Naturalization laws are strictly construed in the
this order of nobility before the Special Committee or governments favor and against the applicant.[1] The
its duly authorized representative, and such applicant carries the burden of proving his full compliance
renunciation shall be included in the records of his with the requirements of law.[2]
application for citizenship.
Section 14. Penalties. - Any person who shall Before the Court is the Republics appeal of the appellate
fraudulently make, falsify, forge, change, alter, or courts Decision[3] dated May 13, 2006 in CA-G.R. CV No.
cause or aid any person to do the same, or who shall 74794, which affirmed the trial courts grant of citizenship
purposely aid and assist in falsely making, forging, to respondent Kerry Lao Ong (Ong). The Court of Appeals
falsifying, changing or altering a naturalization (CA) held:
certificate issued under this proceeding for the
purpose of making use thereof, or in order that the With all the foregoing, We find no cogent reason to reverse
same may be used by another person or persons, and the decision of the court a quo.
any person who shall purposely aid and assist another WHEREFORE, the decision of
in obtaining a naturalization certificate in violation of the Regional Trial Court of Cebu City, 7th Judicial Region,
this Act, shall be punished by a fine of not more than Branch 9 in its Decision dated November 23, 2001, is
Five hundred thousand pesos (P500,OOO.OO) and AFFIRMED in toto and the instant appeal is
by imprisonment for not more than five (5) years, and DISMISSED.
in the case that the person convicted is a naturalized
citizen, his certificate of naturalization shall, if not SO ORDERED.[4]
earlier cancelled by the Special Committee, be
ordered cancelled.
Section 15. Any person who failed to register his/her Factual Antecedents
birth with the concerned city or municipal civil
registrar may, within two (2) years from the
effectivity of this Act, file a petition for the On November 26, 1996, respondent Ong, then 38 years
acquisition of the Philippine old,[5] filed a Petition for Naturalization.[6] The case was
citizenship: Provided, That the applicant possesses docketed as Nat. Case No. 930 and assigned to Branch 9 of
all the qualifications and none of the disqualifications the RegionalTrial Court of Cebu City. As decreed by
under this Act and subject to the requirements of Commonwealth Act No. 473, as amended by Republic Act
existing laws. No. 530, known as the Revised Naturalization Law,[7] the
Section 16. Special Disposition of the Filing Fee. - petition was published in the Official Gazette[8] and a
An amount equivalent to twenty five percent (25%) newspaper of general circulation,[9] and posted in a public
of the filing fee to be paid by the applicants pursuant place for three consecutive weeks,[10] six months before the
to Section 7 hereof shall accrue to the University of initial hearing.[11] The Office of the Solicitor General
Page 43 of 70
entered its appearance and authorized[12] the city prosecutor Bureau of Investigation,[48] the Philippine National
to appear on its behalf.[13] Accordingly, Fiscals Ester Police,[49] the trial courts,[50] and the barangay,[51] he has no
Veloso and Perla Centino participated in the proceedings criminal record or pending criminal charges.[52]
below.
Respondent Ong was born at Respondent presented Rudy Carvajal (Carvajal) and
the Cebu General Hospital in Cebu City to Chinese Bernard Sepulveda (Sepulveda) as his character
citizens Siao Hwa Uy Ong and Flora Ong on March 4, witnesses. At that time, Sepulveda was the vice-mayor of
1958.[14] He is registered as a resident alien and possesses Borbon, Cebu.[53] He has known Ong since 1970 because
an alien certificate of registration[15] and a native-born Ong is the close friend of Sepulvedas brother.[54] He
certificate of residence[16] from the Bureau of Immigration. testified that Ong is very helpful in the community and
He has been continuously and permanently residing[17] in adopts the Filipino culture.[55] Meanwhile, Carvajal
the Philippines from birth up to the present.[18] Ong can testified that he has known Ong since the 1970s because
speak[19] and write in Tagalog, English, Cebuano, they were high school classmates.[56] He testified that Ong
and Amoy.[20] He took his elementary[21] and high is morally irreproachable and possesses all the
school[22] studies at the SacredHeart School for Boys qualifications to be a good citizen of
in Cebu City, where social studies, Pilipino, religion, and the Philippines.[57] Carvajal is a businessman engaged in
the Philippine Constitution are taught. He then obtained a leasing office spaces.[58]
degree in Bachelor of Science in Management from the
Ateneo De Manila University on March 18, 1978.[23] On November 23, 2001, the trial court granted Ongs
petition. Among other things, the trial court held that:
On February 1, 1981, he married Griselda S. Yap, also a
Chinese citizen.[24] They have four children,[25] namely, xxxx
Kerri Gail (born on April 15, 1983),[26] Kimberley Grace
(born on May 15, 1984),[27] Kyle Gervin (born on By the testimonial and documentary evidence adduced by
November 4, 1986),[28] and Kevin Griffith (born on August the [respondent], the following facts had been
21, 1993),[29] who were all born and established.[59]
raised in the Philippines. The children of school age were
enrolled[30] at the Sacred Heart School for xxxx
Boys and Sacred Heart School for Girls.[32] At the time
[31]

of the filing of the petition, Ong, his wife, and children x x x [Respondent] is a businessman/business manager
were living at No. 55 Eagle Street, Sto. Nio Village, engaged in lawful trade and business since 1989 from
Banilad, Cebu City. which he derives an average annual income of more than
One Hundred Fifty Thousand Pesos (Exhibit U, V, W, and
Ong has lived at the following addresses:[33] X with sub-markings); x x x[60]

1. Manalili Street, Cebu City (when Ong was in


Grade 2)[34] The dispositive portion of the trial courts Decision reads:
2. Crystal Compound Guadalupe, Cebu City (until
1970)[35]
3. No. 671 A.S. Fortuna Street, Cebu City (until From the evidence presented by [respondent], this Court
1992)[36] believes and so holds that [respondent] possesses all the
4. No. 55 Eagle Street, Sto. Nio Village, qualifications and none of the disqualifications provided
Banilad, Cebu City (until 1998);[37] and for by law to become a citizen of the Philippines.
5. No. 50 Roselle Street, North Town Homes,
Nasipit, Talamban, Cebu City (present).[38] WHEREFORE, premises considered, the petition is
hereby GRANTED. Accordingly, [respondent] KERRY
Ong alleged in his petition that he has been a LAO ONG is hereby admitted as citizen of the Republic
businessman/business manager since 1989, earning an of the Philippines.
average annual income of P150,000.00.[39] When he
testified, however, he said that he has been a businessman SO ORDERED.[61]
since he graduated from college in 1978.[40] Moreover,
Ong did not specify or describe the nature of his
business. [41] Republics Appeal
As proof of his income, Ong presented four tax returns for
the years 1994 to 1997.[42] Based on these returns, Ongs
gross annual income was P60,000.00 for On January 31, 2003, the Republic, through the Solicitor
1994; P118,000.00 for 1995; P118,000.00 for 1996; General, appealed
and P128,000.00 for 1997. to the CA. The Republic faulted the trial court for granting
Ongs petition despite his failure to prove that he possesses
Respondent further testified that he socializes[43] with a known lucrative trade, profession or lawful occupation as
Filipinos; celebrates the Sinulog, fiestas, birthdays, and required under Section 2, fourth paragraph of the Revised
Christmas.[44] He is a member of the Alert/ React VII Naturalization Law.[62]
Communications Group and the Masonic organization.[45]
Respondent Ong presented a health certificate to The Republic posited that, contrary to the trial courts
prove[46] that he is of sound physical and mental finding, respondent Ong did not prove his allegation that
health.[47] As shown by the clearances from the National he is a businessman/business manager earning an average
Page 44 of 70
income of P150,000.00 since 1989. His income tax returns Revised Naturalization Law, are his tax returns for the
belie the value of his income. Moreover, he failed to years 1994 to 1997, which show that Ong earns
present evidence on the nature of his profession or trade, from P60,000.00 to P128,000.00 annually. This declared
which is the source of his income. Considering that he has income is far from the legal requirement of lucrative
four minor children (all attending exclusive private income. It is not sufficient to provide for the needs of a
schools), he has declared no other property and/or bank family of six, with four children of school age.[73]
deposits, and he has not declared owning a family home,
his alleged income cannot be considered lucrative. Under Moreover, none of these tax returns describes the source of
the circumstances, the Republic maintained that Ongs income, much less can they describe the lawful
respondent Ong is not qualified as he does not possess a nature thereof.[74] The Republic also noted that Ong did not
definite and existing business or trade.[63] even attempt to describe what business he is engaged
in. Thus, the trial and appellate courts shared conclusion
Respondent Ong conceded that the Supreme Court has that Ong is a businessman is grounded entirely on
adopted a higher standard of income for applicants for speculation, surmises or conjectures.[75]
naturalization.[64] He likewise conceded that the legal The Republic thus prays for the reversal of the appellate
definition of lucrative income is the existence of an courts Decision and the denial of Ongs petition for
appreciable margin of his income over his expenses.[65] It is naturalization.[76]
his position that his income, together with that of his wife,
created an appreciable margin over their Respondents Arguments
expenses.[66] Moreover, the steady increase in his income,
as evidenced in his tax returns, proved that he is gainfully Respondent asks for the denial of the petition as it seeks a
employed.[67] review of factual findings, which review is improper in a
Rule 45 petition.[77] He further submits that his tax returns
The appellate court dismissed the Republics appeal. It support the conclusion that he is engaged in lucrative
explained: trade.[78]
In the case at bar, the [respondent] chose to present [pieces
of evidence] which relates [sic] to his lucrative trade, Our Ruling
profession or lawful occupation. Judging from the present The courts must always be mindful that naturalization
standard of living and the personal circumstances of the proceedings are imbued with the highest public
[respondent] using the present time as the index for the interest.[79] Naturalization laws should be rigidly enforced
income stated by the [respondent], it may appear that the and strictly construed in favor of the government and
[respondent] has no lucrative employment. However, We against the
must be mindful that the petition for naturalization was applicant.[80] The burden of proof rests upon the applicant
filed in 1996, which is already ten years ago. It is of judicial to show full and
notice that the value of the peso has taken a considerable complete compliance with the requirements of law.[81]
plunge in value since that time up to the
present.Nonetheless, if We consider the income earned at In the case at bar, the controversy revolves around
that time, the ages of the children of the [respondent], the respondent Ongs compliance with the qualification found
employment of his wife, We can say that there is an in Section 2, fourth paragraph of the Revised
appreciable margin of his income over his expenses as to Naturalization Law, which provides:
be able to provide for an adequate support.[68]
SECTION 2. Qualifications. Subject to section four of this
The appellate court denied the Republics motion for Act, any person having the following qualifications may
reconsideration[69] in its Resolution dated November 7, become a citizen of the Philippines by naturalization:
2006.[70]
xxxx
Issue
Fourth. He must own real estate in the Philippines worth
Whether respondent Ong has proved that he has some not less than five thousand pesos, Philippine currency,
known lucrative trade, profession or lawful occupation in or must have some known lucrative trade, profession, or
accordance with Section 2, fourth paragraph of the Revised lawful occupation;
Naturalization Law.
x x x x[82]

Petitioners Arguments
Based on jurisprudence, the qualification of some known
lucrative trade, profession, or lawful occupation means
Petitioner assigns as error the appellate courts ruling that not only that the person having the employment gets
there is an appreciable margin of (respondents) income enough for his ordinary necessities in life. It must be shown
over his expenses as to be able to provide for an adequate that the employment gives one an income such that there
support.[71] The Republic contends that the CAs conclusion is an appreciable margin of his income over his
is not supported by the evidence on record and by the expenses as to be able to provide for an adequate support
prevailing law.[72] in the event of unemployment, sickness, or disability to
work and thus avoid ones becoming the object of charity
The only pieces of evidence presented by Ong to prove that or a public charge.[83] His income should permit him and
he qualifies under Section 2, fourth paragraph of the the members of his family to live with reasonable
Page 45 of 70
comfort, in accordance with the prevailing standard of had been a businessman since he graduated from college,
living, and consistently with the demands of human which was in 1978.[91]
dignity, at this stage of our civilization.[84]
Further, the trial court, citing Exhibits U, V, W, and X
Moreover, it has been held that in determining the (which are Ongs tax returns), mistakenly found that Ong
existence of a lucrative derives an average annual income of more than One
income, the courts should consider only the applicants Hundred Fifty Thousand Pesos.[92] This conclusion is not
income; his or her spouses income should not be included supported by the evidence. The cited tax returns show that
in the assessment. The spouses additional income is Ongs gross annual income for the years 1994 to 1997
immaterial for under the law the petitioner should be the were P60,000.00, P118,000.00, P118,000.00,
one to possess some known lucrative trade, profession or and P128,000.00, respectively. The average annual
lawful occupation to qualify him to become a Filipino income from these tax returns is P106,000.00 only,
citizen.[85] Lastly, the Court has consistently held that the not P150,000.00 as the trial court held. It appears that the
applicants qualifications must be determined as of the time trial court again derived its conclusion from an assertion in
of the filing of his petition.[86] Ongs petition,[93] but not from the evidence.
As for the CA, it no longer ruled on the question whether
Going over the decisions of the courts below, the Court Ong has a known business or trade. Instead, it ruled on the
finds that the foregoing guidelines have not been issue whether Ongs income, as evidenced by his tax
observed. To recall, respondent Ong and his witnesses returns, can be considered lucrative in 1996. In
testified that Ong is a businessman but none of them determining this issue, the CA considered the ages of Ongs
identified Ongs business or described its nature. The Court children, the income that he earned in 1996, and the fact
finds it suspect that Ong did not even testify as to the nature that Ongs wife was also employed at that time. It then
of his business, whereas his witness Carvajal did with concluded that there is an appreciable margin of Ongs
respect to his own (leasing of office space). A comparison income over his expenses.[94]
of their respective testimonies is reproduced below: The Court finds the appellate courts decision
erroneous. First, it should not have included the spouses
Carvajals testimony income in its assessment of Ongs lucrative
income.[95] Second, it failed to consider the following
Q: You said earlier that you are a businessman? circumstances which have a bearing on Ongs expenses vis-
A: Yes, Sir. -vis his income: (a) that Ong does not own real property;
Q: How long have you been a businessman? (b) that his proven average gross annual income around the
A: Since 1980. time of his application, which was only P106,000.00, had
to provide for the education of his four minor children; and
Q: And what is the business you are engaged in? (c) that Ongs children were all studying in exclusive private
A: I am into leasing of office spaces.[87] schools in Cebu City. Third, the CA did not explain how it
arrived at the conclusion that Ongs income had an
Kerry Lao Ongs testimony appreciable margin over his known expenses.
Ongs gross income might have been sufficient to meet his
Q: What is your present occupation, Mr. Ong? familys basic needs, but there is simply no sufficient proof
A: Businessman. that it was enough to create an appreciable margin of
income over expenses. Without an appreciable margin of
Q: Since when have you engaged in that occupation? his income over his familys expenses, his income cannot
A: After graduation from college.[88] be expected to provide him and his family with adequate
support in the event of unemployment, sickness, or
The dearth of documentary evidence compounds the disability to work.[96]
inadequacy of the testimonial evidence. The applicant
provided no documentary evidence, like business permits, Clearly, therefore, respondent Ong failed to prove that he
registration, official receipts, or other business records to possesses the qualification of a known lucrative trade
demonstrate his proprietorship or participation in a provided in Section 2, fourth paragraph, of
business. Instead, Ong relied on his general assertions to the Revised Naturalization Law.[97]
prove his possession of some known lucrative trade,
profession or lawful occupation. Bare, general assertions The Court finds no merit in respondents submission that a
cannot discharge the burden of proof that is required of an Rule 45 petition precludes a review of the factual findings
applicant for naturalization. of the courts below.[98] In the first place, the trial court and
appellate courts decisions contain conclusions that
The paucity of evidence is unmistakable upon a reading of are bereft of evidentiary support or factual basis, which is
the trial courts decision. The trial court held that respondent a known exception[99] to the general rule that only questions
Ong is a businessman engaged in lawful trade and business of law may be entertained in a Rule 45 petition.
since 1989[89] but did not cite the evidence, which supports
such finding. After poring over the records, the Court finds Moreover, a review of the decisions involving petitions for
that the reason for the lack of citation is the absence of naturalization shows that the Court is not precluded from
evidence to support such conclusion. The trial courts reviewing the factual existence of the applicants
conclusion that Ong has been a businessman since 1989 is qualifications. In fact, jurisprudence holds that the entire
only an assertion found in Ongs petition for records of the naturalization case are open for
naturalization.[90] But, on the witness stand, Ong consideration in an appeal to this Court.[100] Indeed, [a]
did not affirm this assertion. Instead, he testified that he naturalization proceeding is so infused with public interest
Page 46 of 70
that it has been differently categorized and given special Azucena’s claim that she owns real property because
treatment. x x x [U]nlike in ordinary judicial contest, the aliens are precluded from owning lands in the
granting of a petition for naturalization does not preclude country. Finding the grounds relied upon by the OSG
the reopening of that case and giving the government to be evidentiary in nature, the RTC denied said
another opportunity to present new evidence. A decision or Motion.
order granting citizenship will not even constitute res Born in Malangas, Zamboanga del Sur on September
judicata to any matter or reason supporting a subsequent 28, 1941 to Chinese parents, Azucena has never
judgment cancelling the certification of naturalization departed the Philippines since birth. Azucena can
already granted, on the ground that it had been illegally or speak English, Tagalog, Visayan, and Chavacano.
fraudulently procured. For the same reason, issues even if Her primary, secondary, and tertiary education were
not raised in the lower court may be entertained on appeal. taken in Philippine schools. After earning a degree in
As the matters brought to the attention of this Court x x x education, she then practiced her teaching profession
involve facts contained in the disputed decision of the in several different schools in Mindanao.
lower court and admitted by the parties in their pleadings, In 1968, at the age of 26, Azucena married Santiago
the present proceeding may be considered adequate for the Batuigas (Santiago), a natural-born Filipino citizen.
purpose of determining the correctness or incorrectness of They have five children, all of whom studied in
said decision, in the light of the law and extant Philippine public and private schools and are all
jurisprudence.[101] In the case at bar, there is even no need professionals.
to present new evidence. A careful review of the extant After her stint as a teacher, Azucena and her husband,
records suffices to hold that respondent Ong has not proven as conjugal partners, engaged in the retail business of
his possession of a known lucrative trade, profession or and later on in milling/distributing rice, corn, and
lawful occupation to qualify for naturalization. copra. As proof of their income, Azucena submitted
their joint annual tax returns and balance sheets from
WHEREFORE, premises considered, the petition of the 2000- 2002 and from 2004-2005. During their marital
Republic of the Philippines is GRANTED. The Decision union, the Batuigas spouses bought parcels of land in
dated May 13, 2006 of the Court of Appeals in CA-G.R. Barrio Lombog, Margosatubig.
CV No. 74794 is REVERSED and SET ASIDE. The ISSUE:
Petition for Naturalization of Kerry Lao Ong Whether or not petitioner has validly complied the
is DENIED for failure to comply with Section 2, fourth citizenship requirement as required by law to become
paragraph, of Commonwealth Act No. 473, as amended. a naturalized citizen of the Philippines.
RULING:
SO ORDERED. Yes.
Under existing laws, an alien may acquire Philippine
iv- Effect of Naturalization on the Wife, citizenship through either judicial naturalization
Naturalization, Citizenship under CA 473 or administrative naturalization under
Republic Act No. 9139 (the “Administrative
49. Republic of the Philippines vs. Azucena Naturalization Law of 2000”). A third option,
Saavedra Batu(i)gas (DIGEST) called derivative naturalization, which is available to
GR No. 183110 alien women married to Filipino husbands is found
7 October 2013 under Section 15 of CA 473, which provides that:
TOPIC: “Any woman who is now or may hereafter be
Effect of Naturalization on the Wife, Naturalization, married to a citizen of the Philippines and who
Citizenship might herself be lawfully naturalized shall be
FACTS: deemed a citizen of the Philippines.”
This Petition for Review assails the Decision of the Under this provision, foreign women who are married
CA, which affirmed the Decision of the RTC that to Philippine citizens may be deemed ipso
granted the Petition for Naturalization of respondent facto Philippine citizens and it is neither necessary
Azucena Saavedra Batuigas (Azucena). for them to prove that they possess other
On December 2, 2002, Azucena filed a Petition for qualifications for naturalization at the time of their
Naturalization before the RTC of Zamboanga del Sur. marriage nor do they have to submit themselves to
She stated that she intends in good faith to become a judicial naturalization.
citizen of the Philippines and to renounce absolutely Records, however, show that in February 1980,
and forever all allegiance and fidelity to any foreign Azucena applied before the then Commission on
prince, potentate, state or sovereignty, and Immigration and Deportation (CID) for the
particularly to China; and that she will reside cancellation of her Alien Certificate of Registration
continuously in the Philippines from the time of the by reason of her marriage to a Filipino citizen. The
filing of her Petition up to the time of her CID granted her application. However, the Ministry
naturalization. of Justice set aside the ruling of the CID as it found
After all the jurisdictional requirements had been no sufficient evidence that Azucena’s husband is a
complied with, the Office of the Solicitor General Filipino citizen, as only their marriage certificate was
filed its Motion to Dismiss on the ground that presented to establish his citizenship. As the records
Azucena failed to allege that she is engaged in a before this Court show, Santiago’s Filipino
lawful occupation or in some known lucrative trade. citizenship has been adequately proven. Under
The OSG maintained that Azucena is not allowed judicial proceeding, Santiago submitted his birth
under the Retail Trade to engage directly or indirectly certificate indicating therein that he and his parents
in the retail trade. The OSG likewise disputed are Filipinos. He also submitted voter’s registration,
Page 47 of 70
land titles, and business registrations/licenses, all of their respective degrees in architecture and
which are public records. accountancy in Philippine schools.
Moreover, the Court acknowledged that the main On August 27, 1998, they filed with the Regional
objective of extending the citizenship privilege to an Trial Court of Manila a petition under Rule 108 of the
alien wife is to maintain a unity of allegiance among Rules of Court for correction of entries in their
family members, thus: certificates of birth. The case was docketed as Sp.
It is, therefore, not congruent with our cherished Proc. Case No. 98-90470. They alleged, inter alia, in
traditions of family unity and identity that a husband their petition that:
should be a citizen and the wife an alien, and that the (3) They were born in the Philippines and the
national treatment of one should be different from that legitimate children of CO BOON PENG;
of the other. (4) Co Boon Peng, who is formerly a citizen of China,
Azucena has clearly proven, under strict judicial was conferred Philippine citizenship by naturalization
scrutiny, that she is qualified for the grant of that under Presidential Decree No. 1055 and had taken his
privilege, and this Court will not stand in the way of oath of allegiance to the Republic of the Philippines
making her a part of a truly Filipino family. on 15th February, 1977 in the City of Manila;
WHEREFORE, the Petition is DENIED. The (5) At the time of birth of [the] petitioners, their father
Decision of the Court of which affirmed the Decision CO BOON PENG was still a Chinese citizen that is
of the Regional Trial Court, that granted the Petition why entry in their respective birth certificates as to
for Naturalization, is hereby AFFIRMED. Subject to their fathers citizenship was Chinese;
compliance with the period and the requirements (6) Upon granting of Philippine citizenship by
under Republic Act No. 530 which supplements the naturalization to Co Boon Peng in 1977, [the]
Revised Naturalization Law, let a Certificate of petitioners who were born in the Philippines and still
Naturalization be issued to AZUCENA SAAVEDRA minors at that time became Filipino citizens through
BATUIGAS after taking an oath of allegiance to the the derivative mode of naturalization. Our
Republic of the Philippines. Thereafter, her Alien Naturalization Law, specifically Section 15 of
Certificate of Registration should be cancelled. Commonwealth Act No. 473, as amended by
Commonwealth Act No. 535 which provides:
Minor children of persons naturalized under this law
v- Naturalization by Presidential Decree / Legislative who have been born in the Philippines shall be
Naturalization considered citizens thereof;
(7) The naturalization of petitioners father in 1977
49. [G.R. No. 138496. February 23, 2004] was an act or event affecting and concerning their
HUBERT TAN CO and ARLENE TAN civil status that must be recorded in the Civil Register,
CO, petitioners, vs. THE CIVIL REGISTER OF Article 407 of the New Civil Code of the Philippines
MANILA and any person having or claiming an which provides:
interest under the entry whose cancellation or Acts, events and judicial decrees concerning the civil
correction is sought, respondent. status of persons shall be recorded in the Civil
DECISION Register.[2]
CALLEJO, SR., J.: The petitioners prayed that, after due proceedings, the
Before the Court is the petition for review trial court render judgment correcting and changing
on certiorari filed by Hubert Tan Co and Arlene Tan the entries in their respective birth certificates as to
Co seeking to reverse and set aside the Order[1] dated the citizenship of their father Co Boon Peng, from
September 23, 1998 of the Regional Trial Court of Chinese to Filipino.[3]
Manila, Branch 26, dismissing their petition for On September 23, 1998, the court a quo issued an
correction of entries in the Civil Register. Likewise order dismissing the petition outright on the ground
sought to be reversed and set aside is the Order dated that the petition was insufficient, solely because the
April 27, 1999 of the court a quo denying the petitioners father Co Boon Peng applied for
petitioners motion for reconsideration of the said naturalization under LOI No. 270 and was conferred
order. Philippine citizenship by naturalization under PD No.
The factual antecedents are as follows: 1055 and not under Commonwealth Act (CA) No.
Hubert Tan Co was born on March 23, 1974. His 473. [4]
sister, Arlene Tan Co, was born on May 19, 1975. In The petitioners sought the reconsideration of the
their respective certificates of birth, it is stated that assailed order arguing that LOI No. 270 and CA No.
their parents Co Boon Peng and Lourdes Vihong K. 473 were designed to grant citizenship to deserving
Tan are Chinese citizens. aliens; hence, should be construed together. They
Thereafter, Co Boon Peng filed an application for his averred that the benefit of Section 15 of CA No. 473
naturalization as a citizen of the Philippines with the should also be granted to the petitioners whose father
Special Committee on Naturalization under Letter of was granted naturalization under LOI No.
Instruction (LOI) No. 270.His application was 270.However, the RTC issued an Order on April 27,
granted and he was conferred Philippine citizenship 1999, denying their motion for reconsideration for the
under Presidential Decree (P.D.) No. 1055. The following reasons: (a) although Commonwealth Act
Chairman of the Committee issued on February 15, No. 473 and Letter of Instructions No. 270 are statutes
1977 Certificate of Naturalization No. 020778 in his relating to the same subject matter, they do not
favor. Thus, on February 15, 1977, Co Boon Peng provide the same beneficial effects with respect to the
took his oath as a Philippine citizen. In the meantime, minor children of the applicant. Section 15 of CA No.
Hubert and Arlene Co finished college and earned 473 expressly provides for the effect of the
Page 48 of 70
naturalization on the wife and children of the status that must be recorded in the Civil Register
applicant while LOI No. 270 does not have any pursuant to Article 407 of the Civil Code.
proviso to that effect; (b) LOI No. 270 clearly refers In his Comment, the Solicitor General contends that
to qualified individuals only. The rules and the court a quo did not err in issuing the assailed
regulations promulgated by the Committee orders. Contrary to the petitioners theory, LOI No.
established pursuant to LOI No. 270 and the 270 and CA No. 473 are separate and distinct laws;
amendments issued by then President Ferdinand E. therefore, are not in pari materia. He points out that
Marcos (LOI Nos. 292 and 491) clearly speak of although LOI No. 270 and CA No. 473 both govern
qualified individuals only; no proviso therein referred the naturalization of aliens, CA No. 473 deals with the
to its effect on the wife and children of the individual; requirements and procedure for naturalization by
(c) Section 15 of CA No. 473 should not be deemed judicial decree; LOI No. 270, on the other hand, deals
and incorporated in and applied to LOI No. 270; and, with the requirements and procedure for
(d) the application of the so-called pari materia rule naturalization by presidential decree.
of construction made by the petitioners is misplaced, The Solicitor General further asserts that the
as what should be applied in the instant case is the rule petitioners contention that the naturalization of their
on strict construction of legislative grants or father is an event affecting and concerning their civil
franchise. The court a quo stressed that legislative status envisaged in Article 407 of the Civil Code has
grants, whether they be of property, rights or no legal basis. The correction sought and allowed
privileges, whether granted to corporations or under Rule 108 of the Rules of Court must be one that
individuals, must be strictly construed against the reflects a fact existing before or at the time of birth. In
grantee and in favor of the grantor. the petitioners case, the naturalization of their father
Aggrieved, the petitioners now come to this Court in 1977 took place long after they were
assailing the court a quos Order dismissing their born. Moreover, according to the Solicitor General,
petition outright and its Order denying their motion under LOI No. 270 and its amendatory laws, the
for the reconsideration of the same. naturalization of a father did not ipso facto render his
The petitioners contend that the trial court erred in children also naturalized. The petitioners thus cannot
holding that their petition was insufficient. They invoke Article 407 of the Civil Code and Rule 108 of
assert that contrary to the ruling of the trial court, they the Rules of Court to avoid strict compliance with the
are qualified to claim the benefit of Section 15 of CA naturalization laws.
No. 473, which provides that minor children of The petition is meritorious.
persons naturalized thereunder who were born in the The rule on statutory construction provides that:
Philippines shall likewise be considered citizens Statutes in pari materia should be read and construed
thereof. They contend that although LOI No. 270, together because enactments of the same legislature
under which the petitioners father was naturalized on the same subject are supposed to form part of one
does not contain a provision similar to Section 15 of uniform system; later statutes are supplementary or
CA No. 473, the latter provision should be deemed complimentary (sic) to the earlier enactments and in
incorporated therein. They point out that both laws the passage of its acts the legislature is supposed to
have the same purpose and objective, i.e., to grant have in mind the existing legislations on the subject
Philippine citizenship to qualified aliens permanently and to have enacted its new act with reference
residing in the Philippines. The petitioners invoke the thereto.[7]
rule that statutes in pari materia are to be read Statutes in pari materia should be construed together
together.[5] They posit that CA No. 473 and LOI No. to attain the purpose of an expressed national policy,
270 should be harmonized and reconciled since all thus:
statutes relating to the same subject, or having the On the presumption that whenever the legislature
same general purpose, should be read in connection enacts a provision it has in mind the previous statutes
with it, and should be construed together as they relating to the same subject matter, it is held that in
constitute one law.[6] the absence of any express repeal or amendment
The petitioners maintain that the letter and spirit of therein, the new provision was enacted in accord with
LOI No. 270 was to grant the privilege of Philippine the legislative policy embodied in those prior statutes,
citizenship not only to qualified aliens but also to their and they all should be construed together. Provisions
minor children who were born in the country. They in an act which are omitted in another act relating to
assert that this is apparent from paragraph 4-A the same subject matter will be applied in a
thereof, which extends the option to adopt Filipino proceeding under the other act, when not inconsistent
names not only to qualified applicants for with its purpose. Prior statutes relating to the same
naturalization but also to their wives and minor subject matter are to be compared with the new
children. They submit that when then President provisions; and if possible by reasonable
Ferdinand E. Marcos enacted LOI No. 270, he must construction, both are to be construed that effect is
be presumed to have been acquainted with the given to every provision of each. Statutes
provisions of CA No. 473 and did not intend to in pari materia, although in apparent conflict, are so
abrogate and discontinue the beneficial effects of far as reasonably possible construed to be in harmony
Section 15 thereof; otherwise, Pres. Marcos would with each other.[8]
have expressly repealed Section 15 of CA No. 473 in LOI No. 270 and CA No. 473 are laws governing the
relation to LOI No. 270. Thus, according to the naturalization of qualified aliens residing in the
petitioners, the naturalization of their father during Philippines. While they provide for different
their minority is an act or event affecting their civil procedures, CA No. 473 governs naturalization by
judicial decree while LOI No. 270 governs
Page 49 of 70
naturalization by presidential decree; both statutes New Civil Code include even those that occur after
have the same purpose and objective: to enable aliens the birth of the petitioner. However, in such cases, the
permanently residing in the Philippines, who, having entries in the certificates of birth will not be corrected
demonstrated and developed love for and loyalty to or changed. The decision of the court granting the
the Philippines, as well as affinity to the culture, petition shall be annotated in the certificates of birth
tradition and ideals of the Filipino people, and and shall form part of the civil register in the Office
contributed to the economic, social and cultural of the Local Civil Registrar.[12]
development of our country, to be integrated into the To correct simply means to make or set aright; to
national fabric by being granted Filipino remove the faults or error from. To change means to
citizenship. Under the LOI, the procedure for the replace something with something else of the same
acquisition of citizenship by naturalization is more kind or with something that serves as a
expeditious, less cumbersome and less substitute. Article 412 of the New Civil Code does
expensive. The sooner qualified aliens are not qualify as to the kind of entry to be changed or
naturalized, the faster they are able to integrate corrected or distinguished on the basis of the effect
themselves into the national fabric, and are thus able that the correction or change may be.[13] Such entries
to contribute to the cultural, social and political well- include not only those clerical in nature but also
being of the country and its people. substantial errors. After all, the role of the Court
Clearly, LOI No. 270 and CA No. 473 are, as the under Rule 108 of the Rules of Court is to ascertain
petitioners correctly posit, statutes the truths about the facts recorded therein.[14]
in pari materia. Absent any express repeal of Section The proceedings in Rule 108 of the Rules of Court are
15 of CA No. 473 in LOI No. 270, the said provision summary if the entries in the civil register sought to
should be read into the latter law as an integral part be corrected are clerical or innocuous in
thereof, not being inconsistent with its purpose. Thus, nature. However, where such entries sought to be
Section 15 of CA No. 473,[9] which extends the grant corrected or changed are substantial: i.e., the status
of Philippine citizenship to the minor children of and nationality of the petitioners or the citizenship of
those naturalized thereunder, should be similarly their parents,[15] the proceedings are adversarial in
applied to the minor children of those naturalized nature as defined by this Court
under LOI No. 270, like the petitioners in this case. in Republic v. Valencia, thus:
It is not enough that the petitioners adduce in One having opposing parties; contested, as
evidence the certificate of naturalization of their distinguished from an ex parte application, one of
father, Co Boon Peng, and of his oath of allegiance to which the party seeking relief has given legal warning
the Republic of the Philippines, to entitle them to to the other party, and afforded the latter an
Philippine citizenship. They are likewise mandated to opportunity to contest it. Excludes an adoption
prove the following material allegations in their proceeding.[16]
petition: (a) that they are the legitimate children of Co In such a proceeding, the parties to be impleaded as
Boon Peng; (b) that they were born in the Philippines; respective defendants are (a) the local civil registrar;
and, (c) that they were still minors when Co Boon and, (b) all persons who have claims any interest
Peng was naturalized as a Filipino citizen; which would be affected thereby.[17]
The petitioners recourse to Rule 108 of the Rules of In this case, the petitioners alleged in their petition
Court, as amended, is appropriate. Under Article 412 that they are the legitimate children of Co Boon Peng,
of the New Civil Code, no entry in a civil register who was naturalized as a Filipino citizen, but that
shall be changed or corrected without a judicial their certificates of birth still indicate that he is a
order. The law does not provide for a specific Chinese national. In view of their fathers
procedure of law to be followed. But the Court naturalization, they pray that the entries in their
approved Rule 108 of the Rules of Court to provide certificates of birth relating to the citizenship of their
for a procedure to implement the law.[10] The entries father be changed from Chinese to Filipino.
envisaged in Article 412 of the New Civil Code are The petitioners recourse to the procedure in Rule 108
those provided in Articles 407 and 408 of the New of the Rules of Court, as amended, being appropriate,
Civil Code which reads: it behooved the trial court to do its duty under Section
Art. 407. Acts, events and judicial decrees concerning 4, Rule 108 of the Rules of Court, namely:
the civil status of persons shall be recorded in the civil Sec. 4. Notice and Publication. Upon the filing of the
register. petition, the court shall, by an order, fix the time and
Art. 408. The following shall be entered in the civil place for the hearing of the same, and cause
register: reasonable notice thereof to be given to the person
(1) Births; (2) Marriages; (3) deaths; (4) legal named in the petition. The court shall also cause the
separations; (5) annulments of marriage; (6) order to be published once a week for three (3)
judgments declaring marriages void from the consecutive weeks in a newspaper of general
beginning; (7) legitimations; (8) adoptions; (9) circulation in the province.
acknowledgments of natural children; (10) After hearing, the court shall issue an order either
naturalization; (11) loss, or (12) recovery of dismissing the petition or issue an order granting the
citizenship; (13) civil interdiction; (14) judicial same. In either case, a certified copy of the judgment
determination of filiation; (15) voluntary shall be served upon the civil registrar concerned who
emancipation of a minor; and (16) changes of name. shall annotate the same in the certificates of birth of
Specific matters covered by the said provision include the petitioners. The judgment of the court shall form
not only status but also nationality.[11] The acts, part of the records of the local civil register.[18]
events or factual errors envisaged in Article 407 of the
Page 50 of 70
In this case, the trial court dismissed the petition That they personally know him to be morally
outright in violation of Rule 108 of the Rules of irreproachable; 

Court. Patently, then, the trial court erred in so doing. That he has, in their opinion, all the qualifications
IN THE LIGHT OF THE FOREGOING, the necessary to become a citizen of the Philippines; and
petition is GRANTED. The assailed Orders of the 

Regional Trial Court of Manila, Branch 26, are SET That he “is not in any way disqualified under the
ASIDE and REVERSED. The trial court is provisions” of the Naturalization Law. 

DIRECTED to reinstate the petition in Special The records of the case show that the joint affidavits
Proceedings NO. 98-90470 in the court docket, and executed by petitioner’s witnesses did not establish
ORDERED to continue with the proceedings in the their own qualification to stand as such in a
said case under Rule 108 of the Rules of Court, as naturalization proceeding. In turn, petitioner did not
amended. present evidence proving that the persons he
SO ORDERED. presented were credible. In the words of the CA, “he
did not prove that his witnesses had good standing in
the community, known to be honest and upright,
v—Judicial Declaration reputed to be trustworthy and reliable, and that their
word may be taken at face value, as a good warranty
50. G.R. No. 202809, July 02, 2014 - DENNIS L. GO, of the worthiness of petitioner.”
Petitioner, v. REPUBLIC OF THE PHILIPPINES, Furthermore, the background checks done on
Respondent. petitioner yielded negative results due to the
uncooperative behavior of the members of his
Facts: Petitioner filed a petition for naturalization household. In fact, petitioner himself disobliged when
under Commonwealth Act (C.A.) No. 473, the asked for an interview by BOI agents. To the Court,
Revised Naturalization Law. Aside from his this is a display of insincerity to embrace Filipino
presentation of all other requirements, petitioner customs, traditions and ideals.
presented, as witnesses, Dr. Anlacan, Dr. Tordesillas, Finally, it is noteworthy that petitioner’s failure to
Silvino Ong, Teresita Go, and Juan Go. state his former residence in the petition was fatal to
Dr. Anlacan testified that based on the psychiatric his application for naturalization. Indeed, this
examination he conducted on petitioner, he had no omission had deprived the trial court of jurisdiction to
psychiatric abnormality at the time of the test. Dr. hear and decide the case.
Tordesillas, on the other hand, reported that Hence, the petition for naturalization is dismissed
petitioner’s medical examination results were normal. without prejudice.
Ong, a friend of petitioner’s family, said that he had
known petitioner since childhood through his vii- Administrative Philippine Citizenship
association with the family in times of celebration. Supra – Rep vs Batuigas
Teresita described him as a peace-loving person who
participated in activities sponsored by his school and
the barangay. Lastly, Juan, a businessman by viii- OSG Special Committee on Citizenship
profession, also claimed that he knew petitioner 52. EDISON SO, Petitioner, vs. REPUBLIC OF
personally. THE PHILIPPINES, Respondent.
The RTC rendered a decision granting the petition for
naturalization ruling that the petitioner possessed the Facts:
qualifications set forth by law. But the CA reversed He was born on February 17, 1982, in Manila; he is a
and set aside said decision. Hence, this petition. Chinese citizen who has lived in No. 528 Lavezares
Issue: Whether or not Go’s petition for naturalization St., Binondo, Manila, since birth; as an employee, he
should be granted. derives an average annual income of
around P100,000.00 with free board and lodging and
Ruling: No. Jurisprudence dictates that in judicial other benefits; he is single, able to speak and write
naturalization, the application must show substantial English, Chinese and Tagalog; he is exempt from the
and formal compliance with C.A. No. 473. In other filing of Declaration of Intention to become a citizen
words, an applicant must comply with the of the Philippines pursuant to Section 6 of
jurisdictional requirements, establish his or her Commonwealth Act (C.A.) No. 473.
possession of the qualifications and none of the On March 22, 2002, the RTC issued an Order8 setting
disqualifications enumerated under the law, and the petition for hearing at 8:30 a.m. of December 12
present at least two (2) character witnesses to support and 17, 2002 during which all persons concerned
his allegations. were enjoined to show cause, if any, why the petition
In Ong v. Republic of the Philippines, the Court listed should not be granted. The entire petition and its
the requirements for character witnesses, namely: annexes, including the order, were ordered published
That they are citizens of the Philippines; 
 once a week for three consecutive weeks in the
That they are “credible persons”; 
 Official Gazette and also in a newspaper of general
That they personally know the petitioner; 
 circulation in the City of Manila. The RTC likewise
That they personally know him to be a resident of the ordered that copies of the petition and notice be
Philippines for the period of time required by law; 
 posted in public and conspicuous places in the Manila
That they personally know him to be a person of good City Hall Building.9
repute; 
 During the hearing, petitioner presented Atty. Adasa,
Jr. who testified that he came to know petitioner in
Page 51 of 70
1991 as the legal consultant and adviser of the So What must be credible is not the declaration made but
family’s business. He would usually attend parties the person making it. This implies that such person
and other social functions hosted by petitioner’s must have a good standing in the community; that he
family. He knew petitioner to be obedient, is known to be honest and upright; that he is reputed
hardworking, and possessed of good moral character, to be trustworthy and reliable; and that his word may
including all the qualifications mandated by law. be taken on its face value, as a good warranty of the
Another witness for petitioner, Mark Salcedo, applicant’s worthiness.
testified that he has known petitioner for ten (10) e do not agree with petitioner’s argument that
years; they first met at a birthday party in 1991. He respondent is precluded from questioning the RTC
and petitioner were classmates at the University of decision because of its failure to oppose the petition.
Santo Tomas (UST) where they took up Pharmacy. A naturalization proceeding is not a judicial adversary
Petitioner was a member of some school proceeding, and the decision rendered therein does
organizations and mingled well with friends. not constitute res judicata. A certificate of
The RTC granted the petition on June 4, 2003. naturalization may be cancelled if it is subsequently
Respondent Republic of the Philippines, through the discovered that the applicant obtained it by
Office of the Solicitor General (OSG), appealed the misleading the court upon any material fact. Law and
decision to the CA on the following grounds: jurisprudence even authorize the cancellation of a
Issue: certificate of naturalization upon grounds or
W/N Edison So did meet all the qualification needed conditions arising subsequent to the granting of the
to be a naturalized Filipino citizen. certificate.59 If the government can challenge a final
Ruling: grant of citizenship, with more reason can it appeal
The petition is denied for lack of merit. the decision of the RTC within the reglementary
Naturalization signifies the act of formally adopting a period despite its failure to oppose the petition before
foreigner into the political body of a nation by the lower court.
clothing him or her with the privileges of a
citizen.44 Under current and existing laws, there are IN LIGHT OF ALL THE FOREGOING, the petition
three ways by which an alien may become a citizen is DENIED for lack of merit.
by naturalization: (a) administrative naturalization
pursuant to R.A. No. 9139; (b) judicial naturalization c. LOSS OF PHIL CITIZENSHIP AND
pursuant to C.A. No. 473, as amended; and (c) DENATURALIZATION
legislative naturalization in the form of a law enacted 53. LABO VS COMELEC
by Congress bestowing Philippine citizenship to an 176 SCRA 1
alien. – Law on Public Officers – Election Laws –
First. C.A. No. 473 and R.A. No. 9139 are separate Citizenship of a Public Officer – Dual Citizenship –
and distinct laws – the former covers all aliens Labo Doctrine
regardless of class while the latter covers native-born In 1988, Ramon Labo, Jr. was elected as mayor of
aliens who lived here in the Philippines all their lives, Baguio City. His rival, Luis Lardizabal filed a petition
who never saw any other country and all along for quo warranto against Labo as Lardizabal asserts
thought that they were Filipinos; who have that Labo is an Australian citizen hence disqualified;
demonstrated love and loyalty to the Philippines and that he was naturalized as an Australian after he
affinity to the customs and traditions.52 To reiterate, married an Australian. Labo avers that his marriage
the intention of the legislature in enacting R.A. No. with an Australian did not make him an Australian;
9139 was to make the process of acquiring Philippine that at best he has dual citizenship, Australian and
citizenship less tedious, less technical and more Filipino; that even if he indeed became an Australian
encouraging which is administrative rather than when he married an Australian citizen, such
judicial in nature. Thus, although the legislature citizenship was lost when his marriage with the
believes that there is a need to liberalize the Australian was later declared void for being
naturalization law of the Philippines, there is nothing bigamous. Labo further asserts that even if he’s
from which it can be inferred that C.A. No. 473 was considered as an Australian, his lack of citizenship is
intended to be amended or repealed by R.A. No. just a mere technicality which should not frustrate the
9139. What the legislature had in mind was merely to will of the electorate of Baguio who voted for him by
prescribe another mode of acquiring Philippine a vast majority.
citizenship which may be availed of by native born ISSUES:
aliens. The only implication is that, a native born 1. Whether or not Labo can retain his public office.
alien has the choice to apply for judicial or 2. Whether or not Lardizabal, who obtained the
administrative naturalization, subject to the second highest vote in the mayoralty race, can replace
prescribed qualifications and disqualifications. Labo in the event Labo is disqualified.
In naturalization proceedings, it is the burden of the HELD: 1. No. Labo did not question the authenticity
applicant to prove not only his own good moral of evidence presented against him. He was
character but also the good moral character of his/her naturalized as an Australian in 1976. It was not his
witnesses, who must be credible persons.56 Within the marriage to an Australian that made him an
purview of the naturalization law, a "credible person" Australian. It was his act of subsequently swearing by
is not only an individual who has not been previously taking an oath of allegiance to the government of
convicted of a crime; who is not a police character Australia. He did not dispute that he needed an
and has no police record; who has not perjured in the Australian passport to return to the Philippines in
past; or whose affidavit or testimony is not incredible. 1980; and that he was listed as an immigrant here. It
Page 52 of 70
cannot be said also that he is a dual citizen. Dual from promulgation pursuant to Rule 18, Section 13,
allegiance of citizens is inimical to the national Paragraph (b) of the Comelec Rules of Procedure.
interest and shall be dealt with by law. He lost his Accordingly, respondent (Labo) may still continue to
Filipino citizenship when he swore allegiance to be voted upon as candidate for City Mayor of Baguio
Australia. He cannot also claim that when he lost his City on May 11, 1992 subject to the final outcome of
Australian citizenship, he became solely a Filipino. this case in the event the issue is elevated to the
To restore his Filipino citizenship, he must be Supreme Court either on appeal or certiorari.
naturalized or repatriated or be declared as a Filipino On May 13, 1992, respondent Comelec
through an act of Congress – none of this happened. resolved, motu proprio to suspend the proclamation
Labo, being a foreigner, cannot serve public office. of Labo in the event he wins in the elections for the
His claim that his lack of citizenship should not City Mayor of Baguio.
overcome the will of the electorate is not tenable. The On May 15, 1992, petitioner Labo filed the instant
people of Baguio could not have, even unanimously, petition for review with prayer, among others, for the
changed the requirements of the Local Government issuance of a temporary restraining order to set aside
Code and the Constitution simply by electing a the May 9, 1992 resolution of respondent Comelec; to
foreigner (curiously, would Baguio have voted for render judgment declaring him as a Filipino citizen;
Labo had they known he is Australian). The electorate and to direct respondent Comelec to proceed with his
had no power to permit a foreigner owing his total proclamation in the event he wins in the contested
allegiance to the Queen of Australia, or at least a elections.
stateless individual owing no allegiance to the Petitioner Ortega argues that respondent Comelec
Republic of the Philippines, to preside over them as committed grave abuse of discretion when it refused
mayor of their city. Only citizens of the Philippines to implement its May 9, 1992 resolution
have that privilege over their countrymen. notwithstanding the fact that said resolution
2. Lardizabal on the other hand cannot assert, through disqualifying Labo has already become final and
the quo warranto proceeding, that he should be executory.
declared the mayor by reason of Labo’s Petitioner Ortega submits that since this Court did not
disqualification because Lardizabal obtained the issue a temporary restraining order as regards the May
second highest number of vote. It would be extremely 9, 1992 resolution of respondent Comelec cancelling
repugnant to the basic concept of the constitutionally Labo’s certificate of candidacy, said resolution has
guaranteed right to suffrage if a candidate who has not already become final and executory. Ortega further
acquired the majority or plurality of votes is posits the view that as a result of such finality, the
proclaimed a winner and imposed as the candidate receiving the next highest number of votes
representative of a constituency, the majority of should be declared Mayor of Baguio City.
which have positively declared through their ballots Sec. 78 of the Omnibus Election Code provides:
that they do not choose him. Sound policy dictates Petition to deny due course or to cancel a certificate
that public elective offices are filled by those who of candidacy —
have received the highest number of votes cast in the (e) The decision, order, or ruling of the Commission
election for that office, and it is a fundamental idea in shall, after five (5) days from receipt of a copy thereof
all republican forms of government that no one can be by the parties, be final and executory unless stayed by
declared elected and no measure can be declared the Supreme Court.
carried unless he or it receives a majority or plurality Issue:
of the legal votes cast in the election. WON Petitioner Labo who had the highest number of
votes is qualified to assume as Mayor of Baguio City.
Ramon Labo, Jr. vs COMELEC [211 SCRA 297; WON disqualification of petitioner Labo entitles the
GR 105111, July 3, 1992] candidate (Ortega) receiving the next highest number
(Municipal Corporation, Disqualification, of votes to be proclaimed as the winning candidate for
2nd Highest Number of Votes) mayor of Baguio City.
Facts: For the second time around, believing that he Held:
is a Filipino ctizen, Ramon Labo, Jr filed his COC for First Issue:
mayor of Baguio City on March 23, 1992 for the May No. At the time petitioner Labo filed his petition on
11, 1992 elections. Petitioner Roberto Ortega on other May 15, 1992, the May 9, 1992 resolution of
hand, also filed his COC for the same office on March respondent Comelec cancelling his (Labo’s)
25, 1992. certificate of candidacy had already become final and
On March 26, 1992, petitioner Ortega filed a executory a day earlier, or on May 14, 1992, said
disqualification proceeding against Labo before the resolution having been received by petitioner Labo on
COMELEC on the ground that Labo is not a Filipino the same day it was promulgated, i.e., May 9, 1992
citizen. and in the interim no restraining order was issued by
On May 9, 1992, respondent Comelec issued the this Court.
assailed resolution denying Labo’s COC. The resolution cancelling Labo’s certificate of
On May 10, 1992, respondent Comelec issued an candidacy on the ground that he is not a Filipino
Order which reads: Acting on the “Urgent Ex- citizen having acquired finality on May 14, 1992
Parte Motion for Clarification”, filed by respondent constrains the SC to rule against his proclamation as
(Labo) on May 9, 1992, the Commission resolves that Mayor of Baguio City.
the decision promulgated on May 9, 1992 Sec. 39 of the LGC provides that an elective local
disqualifying respondent Ramon L. Labo, Jr., shall official must be a citizen of the Philippines.
become final and executory only after five (5) days Undoubtedly, petitioner Labo, not being a Filipino
Page 53 of 70
citizen, lacks the fundamental qualification for the age of majority” is construed as within 7 years after
contested office. Philippine citizenship is an reaching the age of majority (in his case 21 years old
indispensable requirement for holding an elective because he was born in 1964 while the 1935
office. The fact that he was elected by the majority of Constitution was in place).
the electorate is of no moment. Ching did elect Filipino citizenship but he only did so
Second Issue: when he was preparing for the bar in 1998 or 14 years
No. The disqualification of petitioner Labo does not after reaching the age of majority. Nevertheless, the
necessarily entitle petitioner Ortega as the candidate Solicitor-General recommended that the rule be
with the next highest number of votes to proclamation relaxed due to the special circumstance of Ching.
as the Mayor of Baguio City. ISSUE: Whether or not Ching should be allowed to
While Ortega may have garnered the second highest take the lawyer’s oath.
number of votes for the office of city mayor, the fact HELD: No. Unfortunately, he belatedly elected
remains that he was not the choice of the sovereign Filipino citizenship. The Supreme Court cannot agree
will. Petitioner Labo was overwhelmingly voted by with the recommendation of the Solicitor-General.
the electorate for the office of mayor in the belief that Fourteen years had lapsed and it’s way beyond the
he was then qualified to serve the people of Baguio allowable 7 year period. The Supreme Court even
City and his subsequent disqualification does not noted that the period is originally 3 years but it was
make respondent Ortega the mayor-elect. extended to 7 years. (It seems it can’t be extended any
Petitioner Ortega lost in the election. He was further). Ching’s special circumstances can’t be
repudiated by the electorate. He was obviously not the considered. It is not enough that he considered all his
choice of the people of Baguio City. life that he is a Filipino; that he is a professional and
Thus, while respondent Ortega (GR No. 105111) a public officer (was) serving this country. The rules
originally filed a disqualification case with the for citizenship are in place. Further, Ching didn’t give
Comelec (docketed as SPA-92-029) seeking to deny any explanation why he belatedly chose to elect
due course to petitioner’s (Labo’s) candidacy, the Filipino citizenship (but I guess it’s simply because
same did not deter the people of Baguio City from he never thought he’s Chinese not until he applied to
voting for petitioner Labo, who, by then, was allowed take the bar). The prescribed procedure in electing
by the respondent Comelec to be voted upon, the Philippine citizenship is certainly not a tedious and
resolution for his disqualification having yet to attain painstaking process. All that is required of the elector
the degree of finality (Sec. 78. Omnibus Election is to execute an affidavit of election of Philippine
Code). citizenship and, thereafter, file the same with the
The rule, therefore, is: the ineligibility of a candidate nearest civil registry. Ching’s unreasonable and
receiving majority votes does not entitle the eligible unexplained delay in making his election cannot be
candidate receiving the next highest number of votes simply glossed over.
to be declared elected. A minority or defeated
candidate cannot be deemed elected to the office. i. Procedure for Reacquisition via Repatriation –
RA 8171
56. TABASA vs CA
G.R. No. 125 793, 29 August 2006 [Naturalization;
d. REACQUISITION OF CITIZENSHIP Reacquisition; R.A. No. 8171]
54. TECSON CASE – SUPRA CASE NO.33
FACTS:
55. BAR MATTER No. 914 October 1, 1999 When he was 7 years old, Joevanie A. Tabasa
RE: APPLICATION FOR ADMISSION TO THE acquired American citizenship when his father
PHILIPPINE BAR, became a naturalized citizen of the US. In 1995, he
Legal Profession – Admission to the Bar – Citizenship arrived in the Philippines and was admitted as
Requirement "balikbayan"; thereafter, he was arrested and detained
In 1998, Vicente Ching finished his law degree at the by the agent of BIR. Th Consul General of the US
Saint Louis University in Baguio City. He eventually embassy of Manila filed a request with the BID that
passed the bar but he was advised that he needs to his passport has been revoked and that Tabasa had a
show proof that he is a Filipino citizen before he be standing warrant for several federal charges against
allowed to take his oath. Apparently, Ching’s father him.
was a Chinese citizen but his mother was a Filipino Petitioner alleged that he acquired Filipino
citizen. His parents were married before he was born citizenship by repatriation in accordance with the RA
in 1963. Under the 1935 Constitution, a legitimate No. 8171, and that because he is now a Filipino
child, whose one parent is a foreigner, acquires the citizen, he cannot be deported or detained by the BID.
foreign citizenship of the foreign parent. Ching
maintained that he has always considered himself as ISSUE:
a Filipino; that he is a certified public accountant – a Whether or not he has validly reacquired Philippine
profession reserved for Filipinos; that he even served citizenship under RA 8171 and therefore, is not an
as a councilor in a municipality in La Union. undocumented alien subject to deportation.
The Solicitor-General commented on the case by
saying that as a legitimate child of a Chinese and a RULING:
Filipino, Ching should have elected Filipino No. Petitioner is not qualified to avail himself of
citizenship upon reaching the age of majority; that repatriation under RA 8171. The only person entitled
under prevailing jurisprudence, “upon reaching the to repatriation under RA 8171 is either a Filipino
Page 54 of 70
woman who lost her Philippine citizenship by Private respondent Editha Agbay opposed the
marriage to an alien, or a natural-born Filipino, application on the ground that petitioner, a Canadian
including his minor children who lost Philippine citizen, is disqualified to own land. She also filed a
citizenship on account of political or economic criminal complaint for falsification of public
necessity. documents under Article 172 of the RPC against the
Petitioner was already 35 years old when he filed for petitioner.
repatriation. The act cannot be applied in his case
because he is no longer a minor at the time of his Meanwhile, petitioner re-acquired his Filipino
repatriation in 1996. The privilege under RA 8171 citizenship under the provisions of Republic Act No.
only belongs to children who are of minor age at the 9225.
time of filing of the petition for repatriation.
The CENRO rejected petitioner’s MLA, ruling that
57. ALTAJEROS vs CA petitioner’s subsequent re-acquisition of Philippine
G.R. No. 163256, 10 Nov 2004 [Naturalization; citizenship did not cure the defect in his MLA which
Reacquisition] was void ab initio.

FACTS: An information for Falsification of Public Document


Private respondents filed with the COMELEC to was filed before the MTC and a warrant of arrest was
disqualify and deny due course or cancel the issued against the petitioner.
certificate of candidacy of Ciceron P. Altarejos, on
the ground that he is not a Filipino citizen and that he Since the crime for which petitioner was charged was
made a false representation in his COC that he was alleged and admitted to have been committed before
not a permanent resident of the Municipality of San he had re- acquired his Philippine citizenship, the
Jacinto, Masbate, the town he's running for as mayor MTC concluded that petitioner was at that time still a
in the May 10, 2004 elections. Altarejos answered Canadian citizen.
that he was already issued a Certificate of
Repatriation by the Special Committee on Petitioner elevated the case to the RTC via a petition
Naturalization in December 17, 1997. for certiorari under Rule 65, alleging grave abuse of
discretion on the part of the MTC. The petition was
ISSUE: denied.
Whether or not the registration of petitioner’s
repatriation with the proper civil registry and with the ISSUE:
Bureau of Immigration a prerequisite in effecting Whether or not petitioner may be indicted for
repatriation. falsification for representing himself as a Filipino in
his Public Land Application despite his subsequent
RULING: re-acquisition of Philippine citizenship under the
Yes. The registration of certificate of repatriation provisions of R.A. 9225.
with the proper local civil registry and with the
Bureau of Immigration is a prerequisite in effecting RULING:
repatriation. Petitioner completed all the Considering that petitioner was naturalized as a
requirements of repatriation only after he filed his Canadian citizen prior to the effectivity of R.A. 9225,
certificate of candidacy for a mayoralty position but he belongs to the first category of natural- born
before the elections. Petitioner’s repatriation Filipinos under the first paragraph of Section 3 who
retroacted to the date he filed his application and was, lost Philippine citizenship by naturalization in a
therefore, qualified to run for a mayoralty position in foreign country. As the new law allows dual
the government in the May 10, 2004 elections. citizenship, he was able to re-acquire his Philippine
citizenship by taking the required oath of allegiance.
ii – RA no. 9225 CITIZENSHIP For the purpose of determining the citizenship of
REACQUISITION ACT petitioner at the time of filing his MLA, it is not
58. David vs. Agbay G.R. No. 199113 March 18, necessary to discuss the rulings
2015 Retroactivity of laws, Citizenship, R.A. 9225 in Frivaldo and Altarejos on the retroactivity of such
AUGUST 4, 2018 reacquisition because R.A. 9225 itself treats those of
FACTS: his category as having already lost Philippine
Petitioner migrated to Canada where he became a citizenship, in contradistinction to those natural-born
Canadian citizen by naturalization. Upon retirement, Filipinos who became foreign citizens after R.A.
petitioner and his wife returned to the Philippines and 9225 came into force. In other words, Section 2
purchased a lot along the beach in Oriental Mindoro declaring the policy that considers Filipinos who
where they constructed a residential house. However, became foreign citizens as not to have lost their
the portion where they built their house is public land Philippine citizenship, should be read together with
and part of the salvage zone. Section 3, the second paragraph of which clarifies that
such policy governs all cases after the new law’s
Petitioner filed a Miscellaneous Lease Application effectivity.
(MLA) over the subject land with the DENR. In the Petitioner made the untruthful statement in the MLA,
said application, petitioner indicated that he is a a public document, that he is a Filipino citizen at the
Filipino citizen. time of the filing of said application, when in fact he
was then still a Canadian citizen.
Page 55 of 70
acquitting the defendants with costs against the
Under CA 63, the governing law at the time he was officials.
naturalized as Canadian citizen, naturalization in a
foreign country was among those ways by which a
natural-born citizen loses his Philippine citizenship.
While he re-acquired Philippine citizenship under
R.A. 9225 six months later, the falsification was 60. 100 Phil. 468 – Political Law – Two-fold
already a consummated act, the said law having no Function of the Government – Constituent vs
retroactive effect insofar as his dual citizenship status Ministrant Functions
is concerned. The MTC therefore did not err in Leopoldo Bacani and Mateo Matoto were court
finding probable cause for falsification of public stenographers assigned in a court in Manila. During
document under Article 172, paragraph 1. the pendency of a particular case in said court,
counsel for one of the parties, National Coconut
59. THE UNITED STATES, complainant-appellee Corporation or NACOCO, requested said
Vs stenographers for copies of the transcript of the
FRED L DORR, ET AL., defendants-appellants stenographic notes taken by them during the hearing.
May 19, 1903 Bacani et al complied with the request and sent 714
G.R. No. 1051 pages and thereafter submitted to said counsel their
bills for the payment of their fees. The National
FACTS OF THE CASE Coconut Corporation paid the amount of P564 to
The defendants were charged of scurrilous libel Bacani and P150 to Matoto for said transcripts at the
against the Government of the United States and the rate of P1 per page.
Insular Government of the Philippine Islands However, in January 1953, the Auditor General
because of an editorial it published in the issue of required Bacani et al to reimburse said amounts on
“Manila Freedom”. The defendants were convicted the strength of a circular of the Department of
for said offense basing upon Section 8 of Act. No. Justice. It was expressed that NACOCO, being a
292 of the Commission. Defendants then appealed government entity, was exempt from the payment
for reversal of judgment made by the lower courts. of the fees in question. Bacani et al counter that
ISSUE (ADMINISTRATION) NACOCO is not a government entity within the
Is the editorial published by the defendants directed purview of section 16, Rule 130 of the Rules of
towards the Government of the United States and the Court. NACOCO set up as a defense that the
Insular Government of the Philippine Islands? NACOCO is a government entity within the purview
RULING/HELD of section 2 of the Revised Administrative Code of
No, the editorial was not directed towards the 1917 and, hence, it is exempt from paying the
government itself but towards the aggregate of stenographers’ fees under Rule 130 of the Rules of
individuals who were administering the government Court.
at that time. ISSUE: Whether or not NACOCO is a government
“We understand, in modern political science, . . . by entity.
the term government, that institution or aggregate of HELD: No. Government owned and controlled
institutions by which an independent society makes corporations (GOCCs) do not acquire the status of
and carries out those rules of action which are being part of the government because they do not
unnecessary to enable men to live in a social state, or come under the classification of municipal or public
which are imposed upon the people forming that corporation. Take for instance the NACOCO. While
society by those who possess the power or authority it was organized with the purpose of “adjusting the
of prescribing them. Government is the aggregate of coconut industry to a position independent of trade
authorities which rule a society. By "administration, preferences in the United States” and of providing
again, we understand in modern times, and “Facilities for the better curing of copra products
especially in more or less free countries, the and the proper utilization of coconut by-products“, a
aggregate of those persons in whose hands the reins function which our government has chosen to
of government are for the time being (the chief exercise to promote the coconut industry, it was,
ministers or heads of departments)." (Bouvier, Law however, given a corporate power separate and
Dictionary, 891.) But the writer adds that the terms distinct from our government, for it was made
"government" and "administration" are not always subject to the provisions of our Corporation Law in
used in their strictness, and that "government" is so far as its corporate existence and the powers that
often used for "administration."” it may exercise are concerned (sections 2 and 4,
In this case, the editorial published by defendants Commonwealth Act No. 518 – the law creating
where directed towards the personnel of the NACOCO). It may sue and be sued in the same
Commission whom they described as "notoriously manner as any other private corporations, and in this
corrupt and rascally, and men of no personal sense it is an entity different from our government.
character". This as being ruled out by the Supreme The Supreme Court also noted the constituent
Court was an attack not to the government system functions of the government. Constituent functions
but to the aggregate of individuals by whom the are those which constitute the very bonds of society
government is being administered. and are compulsory in nature. According
NOTES to U.S. President Woodrow Wilson, they are as
The final judgment of the convictions of the follows:
defendants was reversed by the Supreme Court
Page 56 of 70
1. The keeping of order and providing for the Yes. The implementation of the land reform program
protection of persons and property from violence of the government according to Republic Act No.
and robbery. 3844 is most certainly a governmental, not a
2. The fixing of the legal relations between man and proprietary, function; and for that purpose Executive
wife and between parents and children. Order No. 75 has placed the ACA under the Land
3. The regulation of the holding, transmission, and Reform Project Administration.
interchange of property, and the determination of its
liabilities for debt or for crime. The law itself declares that the ACA is a government
4. The determination of contract rights between office, with the formulation of policies, plans and
individuals. programs vested no longer in a Board of Governors,
5. The definition and punishment of crime. as in the case of the ACCFA, but in the National
6. The administration of justice in civil cases. Land Reform Council, itself a government
7. The determination of the political duties, instrumentality; and that its personnel are subject to
privileges, and relations of citizens. Civil Service laws and to rules of standardization
8. Dealings of the state with foreign powers: the with respect to positions and salaries, any vestige of
preservation of the state from external danger or doubt as to the governmental character of its
encroachment and the advancement of its functions disappears.
international interests.
On the other hand, ministrant functions are those 62. Co Kim Cham vs Valdez Tan Keh
that are undertaken only by way of advancing the de facto government
general interests of society, and are merely optional.
The most important of the ministrant functions are:
public works, public education, public charity, CO KIM CHAM VS VALDEZ TAN KEH
health and safety regulations, and regulations of G.R. No. L-5 75 Phil 113, 122 September
trade and industry. The principles to 17, 1945
consider whether or not a government shall exercise CO KIM CHAM (alias CO KIM CHAM), petitioner,
certain of these optional functions are: (1) that a vs.
government should do for the public welfare those EUSEBIO VALDEZ TAN KEH and ARSENIO P.
things which private capital would not naturally DIZON, Judge of First Instance of Manila,
undertake and (2) that a government should do these respondents.
things which by its very nature it is better equipped Facts:
to administer for the public welfare than is any Petitioner Co Kim Cham had a pending Civil Case
private individual or group of individuals. with the Court of First Instance of Manila initiated
during the time of the Japanese occupation.
61. G.R. No. L-21484 November 29, 1969
The respondent judge, Judge Arsenio Dizon, refused
FACTS: to continue hearings on the case which were initiated
ACCFA was a government agency created under RA during the Japanese military occupation on the
No. 821, as amended. Its administrative machinery ground that the proclamation issued by General
was reorganized and its named changed to MacArthur that “all laws, regulations and processes
Agricultural Credit Administration under the Land of any other government in the Philippines than that
Reform Code or RA 3844. ACCFA Supervisors' of the said Commonwealth are null and void and
Association and the ACCFA Workers' Association without legal effect in areas of the Philippines free
were referred to as Union in the ACCFA. of enemy occupation and control” had the effect of
invalidating and nullifying all judicial proceedings
The Unions and ACCFA entered into a collective and judgments of the court of the Philippines during
bargaining agreement effective for a period of one the Japanese military occupation, and that the lower
year. Few months have passed, however,The courts have no jurisdiction to take cognizance of and
Unions, together with the CUGCO, filed a complaint continue judicial proceedings pending in the courts
against the ACCFA for having allegedly committed of the defunct Republic of the Philippines in the
acts of unfair labor practices and non absence of an enabling law granting such authority.
implementation of said agreement. Court of
Industrial Relations ordered ACCFA to cease from Respondent, additionally contends that the
committing further acts tending to discourage the government established during the Japanese
Union members in the exercise of their right to self- occupation were no de facto government.
organizatoin, to comply with and implement the
provisions of the CBA, and to bargain with good
faith with the complainants. ACCFA moved to Issues:
reconsider but it was turned down in a resolution. Whether or not judicial acts and proceedings of the
ACCFA appealed by certiorari. court made during the Japanese occupation were
valid and remained valid even after the liberation or
ISSUE: reoccupation of the Philippines by the United States
Whether or not ACCFA exercised governmental and Filipino forces.
functions. Whether or not the October 23, 1944 proclamation
issued by General MacArthur declaring that “all
RULING: laws, regulations and processes of any other
Page 57 of 70
government in the Philippines than that of the said substantially unimpaired. . . . Such political
Commonwealth are null and void and without legal resurrection is the result of a law analogous to that
effect in areas of the Philippines free of enemy which enables elastic bodies to regain their original
occupation and control” has invalidated all shape upon removal of the external force, — and
judgments and judicial acts and proceedings of the subject to the same exception in case of absolute
courts. crushing of the whole fibre and content.”
Whether or not those courts could continue hearing
the cases pending before them, if the said judicial
acts and proceedings were not invalidated by Rulings:
MacArthur’s proclamation. The judicial acts and proceedings of the court were
good and valid. The governments by the Philippine
Discussions: Executive Commission and the Republic of the
Political and international law recognizes that all Philippines during the Japanese military occupation
acts and proceedings of a de facto government are being de facto governments, it necessarily follows
good and valid. The Philippine Executive that the judicial acts and proceedings of the court of
Commission and the Republic of the Philippines justice of those governments, which are not of a
under the Japanese occupation may be considered de political complexion, were good and valid. Those
facto governments, supported by the military force not only judicial but also legislative acts of de facto
and deriving their authority from the laws of war. government, which are not of a political complexion,
The doctrine upon this subject is thus summed up by remained good and valid after the liberation or
Halleck, in his work on International Law (Vol. 2, p. reoccupation of the Philippines by the American and
444): “The right of one belligerent to occupy and Filipino forces under the leadership of General
govern the territory of the enemy while in its Douglas MacArthur.
military possession, is one of the incidents of war, The phrase “processes of any other government” is
and flows directly from the right to conquer. We, broad and may refer not only to the judicial
therefore, do not look to the Constitution or political processes, but also to administrative or legislative, as
institutions of the conqueror, for authority to well as constitutional, processes of the Republic of
establish a government for the territory of the enemy the Philippines or other governmental agencies
in his possession, during its military occupation, nor established in the Islands during the Japanese
for the rules by which the powers of such occupation. Taking into consideration the fact that,
government are regulated and limited. Such as above indicated, according to the well-known
authority and such rules are derived directly from the principles of international law all judgements and
laws war, as established by the usage of the world, judicial proceedings, which are not of a political
and confirmed by the writings of publicists and complexion, of the de facto governments during the
decisions of courts — in fine, from the law of Japanese military occupation were good and valid
nations. . . . The municipal laws of a conquered before and remained so after the occupied territory
territory, or the laws which regulate private rights, had come again into the power of the titular
continue in force during military occupation, excepts sovereign, it should be presumed that it was not, and
so far as they are suspended or changed by the acts could not have been, the intention of General
of conqueror. . . . He, nevertheless, has all the Douglas MacArthur, in using the phrase “processes
powers of a de facto government, and can at his of any other government” in said proclamation, to
pleasure either change the existing laws or make refer to judicial processes, in violation of said
new ones.” principles of international law.
General MacArthur annulled proceedings of other Although in theory the authority of the local civil
governments in his proclamation October 23, 1944, and judicial administration is suspended as a matter
but this cannot be applied on judicial proceedings of course as soon as military occupation takes place,
because such a construction would violate the law of in practice the invader does not usually take the
nations. administration of justice into his own hands, but
If the proceedings pending in the different courts of continues the ordinary courts or tribunals to
the Islands prior to the Japanese military occupation administer the laws of the country which he is
had been continued during the Japanese military enjoined, unless absolutely prevented, to respect. An
administration, the Philippine Executive Executive Order of President McKinley to the
Commission, and the so-called Republic of the Secretary of War states that “in practice, they (the
Philippines, it stands to reason that the same courts, municipal laws) are not usually abrogated but are
which had become re-established and conceived of allowed to remain in force and to be administered by
as having in continued existence upon the the ordinary tribunals substantially as they were
reoccupation and liberation of the Philippines by before the occupation. This enlightened practice is,
virtue of the principle of postliminy (Hall, so far as possible, to be adhered to on the present
International Law, 7th ed., p. 516), may continue the occasion.” And Taylor in this connection says:
proceedings in cases then pending in said courts, “From a theoretical point of view it may be said that
without necessity of enacting a law conferring the conqueror is armed with the right to substitute
jurisdiction upon them to continue said proceedings. his arbitrary will for all pre-existing forms of
As Taylor graphically points out in speaking of said government, legislative, executive and judicial.
principles “a state or other governmental entity, From the stand-point of actual practice such
upon the removal of a foreign military force, arbitrary will is restrained by the provision of the
resumes its old place with its right and duties law of nations which compels the conqueror to
Page 58 of 70
continue local laws and institution so far as military laid to rest by Congress and the decision that
necessity will permit.” Undoubtedly, this practice respondent Arroyo is the de jure President made by a
has been adopted in order that the ordinary pursuits co-equal branch of government cannot be reviewed
and business of society may not be unnecessarily by this Court.
deranged, inasmuch as belligerent occupation is
essentially provisional, and the government 64. Republic vs. Sandiganbayan (Case Digest)
established by the occupant of transient character. Republic vs. Sandiganbayan, GR No. 152154, July
15, 2003
63. Estrada vs Desierto Corrected Version
353 SCRA 452 – Political Law – Constitutional Law
– De Jure vs De Facto President – Arroyo a de jure Topic: Prohibitions/Inhibitions of President of the
president Philippines
Joseph “Erap” Estrada alleges that he is the
President on leave while Gloria Macapagal-Arroyo FACTS:
claims she is the President. From the beginning of One of the foremost concerns of the Aquino
Erap’s term, he was plagued by problems that slowly Government in February 1986 was the recovery of
but surely eroded his popularity. His sharp descent the unexplained or ill-gotten wealth reputedly
from power started on October 4, 2000. Singson, a amassed by former President and Mrs. Ferdinand E.
longtime friend of Estrada, went on air and accused Marcos, their relatives, friends and business
the Estrada, his family and friends of receiving associates. Thus, the very first Executive Order (EO)
millions of pesos from jueteng lords. The exposé issued by then President Corazon Aquino upon her
immediately ignited reactions of rage. On January assumption to office after the ouster of the Marcoses
19, Estrada fell from power. At 1:20 p.m. of said was EO No. 1, issued on February 28, 1986. It
day, the Erap informed then Executive Secretary created the Presidential Commission on Good
Edgardo Angara that General Angelo Reyes, Chief Government (PCGG) and charged it with the task of
of Staff of the Armed Forces of the Philippines, had assisting the President in the "recovery of all ill-
defected. January 20 turned to be the day of Erap’s gotten wealth accumulated by former President
surrender. On January 22, the Monday after taking Ferdinand E. Marcos, his immediate family,
her oath, Arroyo immediately discharged the powers relatives, subordinates and close associates, whether
and duties of the Presidency. After his fall from the located in the Philippines or abroad, including the
pedestal of power, Erap’s legal problems appeared in takeover or sequestration of all business enterprises
clusters. Several cases previously filed against him and entities owned or controlled by them during his
in the Office of the Ombudsman were set in motion. administration, directly or through nominees, by
ISSUE: Whether or not Arroyo is a legitimate (de taking undue advantage of their public office and/or
jure) president. using their powers, authority, influence, connections
HELD: The SC holds that the resignation of Estrada or relationship."
cannot be doubted. It was confirmed by his leaving In all the alleged ill-gotten wealth cases filed by the
Malacañang. In the press release containing his final PCGG, this Court has seen fit to set aside
statement, (1) he acknowledged the oath-taking of technicalities and formalities that merely serve to
the respondent as President of the Republic albeit delay or impede judicious resolution. This Court
with the reservation about its legality; (2) he prefers to have such cases resolved on the merits at
emphasized he was leaving the Palace, the seat of the Sandiganbayan. But substantial justice to the
the presidency, for the sake of peace and in order to Filipino people and to all parties concerned, not
begin the healing process of our nation. He did not mere legalisms or perfection of form, should now be
say he was leaving the Palace due to any kind of relentlessly and firmly pursued. Almost two decades
inability and that he was going to re-assume the have passed since the government initiated its search
presidency as soon as the disability disappears; (3) for and reversion of such ill-gotten wealth. The
he expressed his gratitude to the people for the definitive resolution of such cases on the merits is
opportunity to serve them. Without doubt, he was thus long overdue. If there is proof of illegal
referring to the past opportunity given him to serve acquisition, accumulation, misappropriation, fraud
the people as President; (4) he assured that he will or illicit conduct, let it be brought out now. Let the
not shirk from any future challenge that may come ownership of these funds and other assets be finally
ahead in the same service of our country. Estrada’s determined and resolved with dispatch, free from all
reference is to a future challenge after occupying the the delaying technicalities and annoying procedural
office of the president which he has given up; and sidetracks.
(5) he called on his supporters to join him in the
promotion of a constructive national spirit of Issue:
reconciliation and solidarity. Certainly, the national Whether or not President Marcos committed
spirit of reconciliation and solidarity could not be prohibited and inhibited acts as a president during
attained if he did not give up the presidency. The his term of office
press release was petitioner’s valedictory, his final
act of farewell. His presidency is now in the past
tense. Even if Erap can prove that he did not resign, Held:
still, he cannot successfully claim that he is a Yes
President on leave on the ground that he is merely
unable to govern temporarily. That claim has been Ratio:
Page 59 of 70
It is settled that judicial admissions may be provisions are mandatory and should thus be
made: (a) in the pleadings filed by the parties; (b) in construed strictly against the petitioner and liberally
the course of the trial either by verbal or written in favor of respondent Marcoses.
manifestations or stipulations; or (c) in other stages We hold that it was not for petitioner to establish the
of judicial proceedings, as in the pre-trial of the Marcoses other lawful income or income from
case.[82] Thus, facts pleaded in the petition and legitimately acquired property for the presumption to
answer, as in the case at bar, are deemed admissions apply because, as between petitioner and
of petitioner and respondents, respectively, who are respondents, the latter were in a better position to
not permitted to contradict them or subsequently know if there were such other sources of lawful
take a position contrary to or inconsistent with such income. And if indeed there was such other lawful
admissions.[83] income, respondents should have specifically stated
The sum of $304,372.43 should be held as the only the same in their answer. Insofar as petitioner
known lawful income of respondents since they did Republic was concerned, it was enough to specify
not file any Statement of Assets and Liabilities the known lawful income of respondents.
(SAL), as required by law, from which their net
worth could be determined. Besides, under the 1935 Section 9 of the PCGG Rules and Regulations
Constitution, Ferdinand E. Marcos as President provides that, in determining prima facie evidence of
could not receive any other emolument from the ill-gotten wealth, the value of the accumulated
Government or any of its subdivisions and assets, properties and other material possessions of
instrumentalities.[84] Likewise, under the 1973 those covered by Executive Order Nos. 1 and 2
Constitution, Ferdinand E. Marcos as President must be out of proportion to the known lawful incom
could not receive during his tenure any other e of such persons. The respondent Marcos couple
emolument from the Government or any other did not file any Statement of Assets and Liabilities
source.[85] In fact, his management of businesses, (SAL) from which their net worth could be
like the administration of foundations to accumulate determined. Their failure to file their SAL was in
funds, was expressly prohibited under the 1973 itself a violation of law and to allow them to
Constitution: successfully assail the Republic for not presenting
Article VII, Sec. 4(2) The President and the Vice- their SAL would reward them for their violation of
President shall not, during their tenure, hold any the law.
other office except when otherwise provided in this
Constitution, nor may they practice any profession, 65. Gov vs. Monte de Peidad 1916
participate directly or indirectly in the management 35 Phil. 728 – Political Law – Parens Patriae
of any business, or be financially interested directly In June 1863 a devastating earthquake occurred in
or indirectly in any contract with, or in any franchise the Philippines. The Spanish Government then
or special privilege granted by the Government or provided $400,000.00 as aid for the victims and it
any other subdivision, agency, or instrumentality was received by the Philippine Treasury. Out of the
thereof, including any government owned or said amount, $80,000.00 was left untouched; it was
controlled corporation. then invested in the Monte de Piedad Bank which in
Article VII, Sec. 11 No Member of the National turn invested the amount in jewelries. But when the
Assembly shall appear as counsel before any court Philippine government later tried to withdraw the
inferior to a court with appellate jurisdiction, x x said amount, the bank cannot provide for the
x. Neither shall he, directly or indirectly, be amount. The government then filed a complaint. The
interested financially in any contract with, or in any bank argued that the Philippine government is not an
franchise or special privilege granted by the affected party hence has no right to institute a
Government, or any subdivision, agency, or complaint. The bank argues that the government was
instrumentality thereof including any government not the intended beneficiary of the said amount.
owned or controlled corporation during his term of ISSUE: Whether or not the Philippine government is
office. He shall not intervene in any matter before competent to file a complaint against the respondent
any office of the government for his pecuniary bank.
benefit. HELD: Yes. The Philippine government is
Article IX, Sec. 7 The Prime Minister and Members competent to institute action against Monte de
of the Cabinet shall be subject to the provision of Piedad, this is in accordance with the doctrine
Section 11, Article VIII hereof and may not appear of Parens Patriae. The government being the
as counsel before any court or administrative body, protector of the rights of the people has the inherent
or manage any business, or practice any profession, supreme power to enforce such laws that will
and shall also be subject to such other promote the public interest. No other party has been
disqualification as may be provided by law. entrusted with such right hence as “parents” of the
Their only known lawful income of $304,372.43 can people the government has the right to take back the
therefore legally and fairly serve as basis for money intended for the people.
determining the existence of a prima facie case of
forfeiture of the Swiss funds. 65 A. JENIE SAN JUAN DELA CRUZ V.
Respondents argue that petitioner was not able to RONALD PAUL S. GRACIA
establish a prima facie case for the forfeiture of the PHILIPPINES
Swiss funds since it failed to prove the essential Title:
elements under Section 3, paragraphs (c), (d) and (e) Jenie San Juan Dela Cruz, et al., etc., v. Ronald Paul
of RA 1379. As the Act is a penal statute, its S. Gracia, etc.
Page 60 of 70
Court: stating that a written instrument acknowledging
Supreme Court of the Philippines paternity needs to be signed, the requirement for due
Date: execution is implied. In the present case, however,
31 July 2009 special circumstances exist to hold that Dominique's
CRC Provisions: acknowledgement of paternity though unsigned by
Article 3: Best interests of the child him, substantially satisfies the requirement of the
Domestic Provisions: law.
Rule 7 of Administrative Order No. 1, Series of Since Dominique died two months prior to the
2004 - Requirements for the Child to Use the child's birth and is prevented from acknowledging
Surname of the Father: The illegitimate child shall paternity in person. And also, the written
use the surname of the father if a public document is acknowledgement of paternity was handwritten and
executed by the father. If admission of paternity is its authenticity was supported by the facts and
made through a private handwritten instrument, the evidence that Jenie presented. For example,
child shall use the surname of the father, provided Dominique's brother's corroboration and
the registration is supported by certain documents. confirmation of the authenticity of the written
Article 176 of the Family Code: Illegitimate children acknowledgement despite such testimony being
shall use the surname and shall be under the parental prejudicial to his own rights to inheritance.
authority of their mother. Illegitimate children may Furthermore, the fact that Jenie and Dominique lived
use the surname of their father if their filiation has together and Jenie continues to live with
been expressly recognised by the father through the Dominique's parents provide further factual
record of birth appearing in the civil register, or verification. There is therefore no doubt or dispute
when an admission in a public document or private that Dominique's written acknowledgement is
handwritten instrument is made by the father. The authentic and that paternity is established.
father has the right to institute an action before the The law states that the welfare of the child shall be
courts to prove non-filiation during his lifetime. the "paramount consideration". It is in Christian's
Case Summary: best interests to have legal certainty of his paternity
Background: and to allow him to bear the surname of his now
The petitioner Jenie San Juan Dela Cruz (Jenie) deceased father and have a definitive name reflected
cohabited with her partner Christian Dominique in his birth certificate and avoid the stigma of
Tomas Aquino (Dominique) for several months illegitimacy.
leading up to Dominique's death. They were not Excerpts citing CRC and other relevant human
legally married. Two months after Dominique's rights instruments:
death, Jenie gave birth to their child, Christian. Our laws instruct that the welfare of the child shall
Jenie applied to register Christian with Dominique's be the "paramount consideration" in resolving
surname and presented written evidence that questions affecting him. Article 3(1) of the United
Dominique had acknowledged that Christian was his Nations Convention on the Rights of a Child of
child before his death. The application was denied which the Philippines is a signatory is similarly
on the ground that the child was considered to be emphatic:
born out of wedlock and Dominique has no capacity Article 3(1): In all actions concerning children,
to acknowledge paternity of Christian or provide the whether undertaken by public or private social
documents required following his death. welfare institutions, courts of law, administrative
Jenie filed a complaint against the decision and authorities or legislative bodies, the best interests of
argued that Dominique's written acknowledgement the child shall be a primary consideration.
of paternity prior to his death, was also supported It is thus "the policy of the Family Code to liberalise
and verified by his brother, falls within the the rule on the investigation of the paternity and
documents required under the law. The trial court filiation of children, especially of illegitimate
dismissed Jenie's complaint as Dominique's written children." Too, "the State as parens patriae affords
acknowledgement was not signed and held that even special protection to children from abuse,
if it was signed, did not contain an express exploitation and other conditions prejudicial to their
acknowledgement and therefore did not meet the development."
legal requirements. Jenie appealed and argued that In the eyes of society, a child with an unknown
the law did not require an acknowledgement to be father bears the stigma of dishonor. It is to petitioner
signed and that the trial court had erred in not minor child's best interests to allow him to bear the
recognising a clear and unmistakable surname of the now deceased Dominique and enter it
acknowledgement of paternity. in his birth certificate.
Issue and resolution: CRIN Comments:
Paternity of child born out of wedlock. Whether or CRIN believes this decision is consistent with the
not the unsigned handwritten statement of the CRC. The best interests of the child must be a
deceased father can be considered a recognition of primary consideration when determining all matters
paternity in a "private handwritten instrument" affecting the child, including the determination of
within the meaning of Article 176 of the Family the child’s paternity.
Code. The Court held that it did and ordered
Christian to be registered with Dominique's 66. Rubi vs Provincial Board 1919
surname. 39 Phil. 660 – Political Law – Delegation of Powers
Court reasoning: – Liberty and due process
Although there is no express requirement in the law
Page 61 of 70
Rubi and various other Manguianes (Mangyans) in the degree of civilization of the Manguianes is
the province of Mindoro were ordered by the considered. They are restrained for their own good
provincial governor of Mindoro to remove their and the general good of the Philippines. Nor can one
residence from their native habitat and to established say that due process of law has not been followed.
themselves on a reservation in Tigbao, still in the To go back to our definition of due process of law
province of Mindoro, and to remain there, or be and equal protection of the laws, there exists a law;
punished by imprisonment if they escaped. the law seems to be reasonable; it is enforced
Manguianes had been ordered to live in a reservation according to the regular methods of procedure
made to that end and for purposes of cultivation prescribed; and it applies alike to all of a class.”
under certain plans. The Manguianes are a Non-
Christian tribe who were considered to be of “very 67. Agustin vs Edu 88 s 772 (2001)
low culture”. Generally Accepted Principles of International Law
One of the Manguianes, a certain Dabalos, escaped – Police Power
from the reservation but was later caught and Agustin is the owner of a Volkswagen Beetle Car.
was placed in prison at Calapan, solely because he He is assailing the validity of Letter of Instruction
escaped from the reservation. An application for No 229 which requires all motor vehicles to have
habeas corpus was made on behalf by Rubi and early warning devices particularly to equip them
other Manguianes of the province, alleging that by with a pair of “reflectorized triangular early warning
virtue of the resolution of the provincial board of devices”. Agustin is arguing that this order is
Mindoro creating the reservation, they had been unconstitutional, harsh, cruel and unconscionable to
illegally deprived of their liberty. In this case, the the motoring public. Cars are already equipped with
validity of Section 2145 of the Administrative Code, blinking lights which is already enough to provide
which provides: warning to other motorists. And that the mandate to
With the prior approval of the Department Head, the compel motorists to buy a set of reflectorized early
provincial governor of any province in which non- warning devices is redundant and would only make
Christian inhabitants are found is authorized, when manufacturers and dealers instant millionaires.
such a course is deemed necessary in the interest of ISSUE: Whether or not the said is EO is valid.
law and order, to direct such inhabitants to take up HELD: Such early warning device requirement is
their habitation on sites on unoccupied public lands not an expensive redundancy, nor oppressive, for car
to be selected by him and approved by the provincial owners whose cars are already equipped with 1)
board. ‘blinking-lights in the fore and aft of said motor
was challenged. vehicles,’ 2) ‘battery-powered blinking lights inside
ISSUE: Whether or not Section 2145 of the motor vehicles,’ 3) ‘built-in reflectorized tapes on
Administrative Code constitutes undue delegation. front and rear bumpers of motor vehicles,’ or 4)
Whether or not the Manguianes are being deprived ‘well-lighted two (2) petroleum lamps (the Kinke) . .
of their liberty. . because: Being universal among the signatory
HELD: countries to the said 1968 Vienna Conventions, and
I. No. By a vote of five to four, the Supreme Court visible even under adverse conditions at a distance
sustained the constitutionality of this section of the of at least 400 meters, any motorist from this
Administrative Code. Under the doctrine of country or from any part of the world, who sees a
necessity, who else was in a better position to reflectorized rectangular early warning device
determine whether or not to execute the law but the installed on the roads, highways or expressways,
provincial governor. It is optional for the provincial will conclude, without thinking, that somewhere
governor to execute the law as circumstances may along the travelled portion of that road, highway, or
arise. It is necessary to give discretion to the expressway, there is a motor vehicle which is
provincial governor. The Legislature may make stationary, stalled or disabled which obstructs or
decisions of executive departments of subordinate endangers passing traffic. On the other hand, a
official thereof, to whom it has committed the motorist who sees any of the aforementioned other
execution of certain acts, final on questions of fact. built-in warning devices or the petroleum lamps will
II. No. Among other things, the term “non- not immediately get adequate advance warning
Christian” should not be given a literal meaning or a because he will still think what that blinking light is
religious signification, but that it was intended to all about. Is it an emergency vehicle? Is it a law
relate to degrees of civilization. The term “non- enforcement car? Is it an ambulance? Such
Christian” it was said, refers not to religious belief, confusion or uncertainty in the mind of the motorist
but in a way to geographical area, and more directly will thus increase, rather than decrease, the danger of
to natives of the Philippine Islands of a low grade of collision.
civilization. In this case, the Manguianes were being On Police Power
reconcentrated in the reservation to promote peace The Letter of Instruction in question was issued in
and to arrest their seminomadic lifestyle. This will the exercise of the police power. That is conceded by
ultimately settle them down where they can adapt to petitioner and is the main reliance of respondents. It
the changing times. is the submission of the former, however, that while
The Supreme Court held that the resolution of the embraced in such a category, it has offended against
provincial board of Mindoro was neither the due process and equal protection safeguards of
discriminatory nor class legislation, and stated the Constitution, although the latter point was
among other things: “. . . one cannot hold that the mentioned only in passing. The broad and expansive
liberty of the citizen is unduly interfered with when scope of the police power which was originally
Page 62 of 70
identified by Chief Justice Taney of the American petitioner failing in his quest, was likewise prompted
Supreme Court in an 1847 decision, as “nothing by the imperative demands of public safety.
more or less than the powers of government inherent
in every sovereignty” was stressed in the 68. Product v. Fertiphil Corp.
aforementioned case of Edu v. Ericta thus: “Justice G.R. No. 166006 March 14, 2008
Laurel, in the first leading decision after the REYES, R.T., J.
Constitution came into force, Calalang v. Williams,
identified police power with state authority to enact Lessons Applicable: Bet. private and public suit,
legislation that may interfere with personal liberty or easier to file public suit, Apply real party in interest
property in order to promote the general welfare. test for private suit and direct injury test for public
Persons and property could thus ‘be subjected to all suit, Validity test varies depending on which
kinds of restraints and burdens in order to secure the inherent power
general comfort, health and prosperity of the state.
Shortly after independence in 1948, Primicias v. Laws Applicable:
Fugoso reiterated the doctrine, such a competence
being referred to as ‘the power to prescribe FACTS:
regulations to promote the health, morals, peace,
education, good order or safety, and general welfare President Ferdinand Marcos, exercising his
of the people.’ The concept was set forth in negative legislative powers, issued LOI No. 1465 which
terms by Justice Malcolm in a pre-Commonwealth provided, among others, for the imposition of a
decision as ‘that inherent and plenary power in the capital recovery component (CRC) on the domestic
State which enables it to prohibit all things hurtful to sale of all grades of fertilizers which resulted in
the comfort, safety and welfare of society.’ In that having Fertiphil paying P 10/bag sold to the
sense it could be hardly distinguishable as noted by Fertilizer and Perticide Authority (FPA).
this Court in Morfe v. Mutuc with the totality of FPA remits its collection to Far East Bank and Trust
legislative power. It is in the above sense the greatest Company who applies to the payment of corporate
and most powerful attribute of government. It is, to debts of Planters Products Inc. (PPI)
quote Justice Malcolm anew, ‘the most essential, After the Edsa Revolution, FPA voluntarily stopped
insistent, and at least illimitable powers,’ extending the imposition of the P10 levy. Upon return of
as Justice Holmes aptly pointed out ‘to all the great democracy, Fertiphil demanded a refund but PPI
public needs.’ Its scope, ever expanding to meet the refused. Fertiphil filed a complaint for collection
exigencies of the times, even to anticipate the future and damages against FPA and PPI with the RTC on
where it could be done, provides enough room for an the ground that LOI No. 1465 is unjust, unreaonable
efficient and flexible response to conditions and oppressive, invalid and unlawful resulting to denial
circumstances thus assuring the greatest benefits. In of due process of law.
the language of Justice Cardozo: ‘Needs that were FPA answered that it is a valid exercise of the police
narrow or parochial in the past may be interwoven in power of the state in ensuring the stability of the
the present with the well-being of the nation. What is fertilizing industry in the country and that Fertiphil
critical or urgent changes with the time.’ The police did NOT sustain damages since the burden imposed
power is thus a dynamic agency, suitably vague and fell on the ultimate consumers.
far from precisely defined, rooted in the conception RTC and CA favored Fertiphil holding that it is an
that men in organizing the state and imposing upon exercise of the power of taxation ad is as such
its government limitations to safeguard because it is NOT for public purpose as PPI is a
constitutional rights did not intend thereby to enable private corporation.
an individual citizen or a group of citizens to ISSUE:
obstruct unreasonably the enactment of such salutary 1. W/N Fertiphil has locus standi
measures calculated to insure communal peace, 2. W/N LOI No. 1465 is an invalid exercise of the
safety, good order, and welfare.” power of taxation rather the police power
It was thus a heavy burden to be shouldered by
Agustin, compounded by the fact that the particular Held:
police power measure challenged was clearly 1. Yes. In private suits, locus standi requires a
intended to promote public safety. It would be a rare litigant to be a "real party in interest" or party who
occurrence indeed for this Court to invalidate a stands to be benefited or injured by the judgment in
legislative or executive act of that character. None the suit. In public suits, there is the right of the
has been called to our attention, an indication of its ordinary citizen to petition the courts to be freed
being non-existent. The latest decision in point, Edu from unlawful government intrusion and illegal
v. Ericta, sustained the validity of the Reflector official action subject to the direct injury test or
Law, an enactment conceived with the same end in where there must be personal and substantial interest
view. Calalang v. Williams found nothing in the case such that he has sustained or will sustain
objectionable in a statute, the purpose of which was: direct injury as a result. Being a mere procedural
“To promote safe transit upon, and avoid obstruction technicality, it has also been held that locus standi
on roads and streets designated as national roads . . may be waived in the public interest such as cases of
.” As a matter of fact, the first law sought to be transcendental importance or with far-reaching
nullified after the effectivity of the 1935 implications whether private or public suit, Fertiphil
Constitution, the National Defense Act, with has locus standi.

Page 63 of 70
2. As a seller, it bore the ultimate burden of paying cause not previously recognized. It merely gives a
the levy which made its products more expensive remedy to enforce a preexisting liability and submits
and harm its business. It is also of paramount public itself to the jurisdiction of the court, subject to its
importance since it involves the constitutionality of a right to interpose any lawful defense. It follows
tax law and use of taxes for public purpose. therefrom that the state, by virtue of such provisions
of law, is not responsible for the damages suffered
3. Yes. Police power and the power of taxation are by private individuals in consequence of acts
inherent powers of the state but distinct and have performed by its employees in the discharge of the
different tests for validity. Police power is the functions pertaining to their office, because neither
power of the state to enact the legislation that may fault nor even negligence can be presumed on the
interfere with personal liberty on property in order to part of the state in the organization of branches of
promote general welfare. While, the power of public service and in the appointment of its agents.
taxation is the power to levy taxes as to be used for The State can only be liable if it acts through a
public purpose. The main purpose of police power special agent (and a special agent, in the sense in
is the regulation of a behavior or conduct, while which these words are employed, is one who
taxation is revenue generation. The lawful subjects receives a definite and fixed order or commission,
and lawful means tests are used to determine the foreign to the exercise of the duties of his office if he
validity of a law enacted under the police is a special official) so that in representation of the
power. The power of taxation, on the other hand, is state and being bound to act as an agent thereof, he
circumscribed by inherent and constitutional executes the trust confided to him.
limitations. In the case at bar, the ambulance driver was not a
special agent nor was a government officer acting as
In this case, it is for purpose of revenue. But it is a a special agent hence, there can be no liability from
robbery for the State to tax the citizen and use the the government. “The Government does not
funds generation for a private purpose. Public undertake to guarantee to any person the fidelity of
purpose does NOT only pertain to those purpose the officers or agents whom it employs, since that
which are traditionally viewed as essentially would involve it in all its operations in endless
governmental function such as building roads and embarrassments, difficulties and losses, which
delivery of basic services, but also includes those would be subversive of the public interest.”
purposes designed to promote social justice. Thus,
public money may now be used for the relocation of 70. Vigilar vs. Aquino, G.R. No. 180388, Jan. 18,
illegal settlers, low-cost housing and urban or 2011 Immunity from Suit
agrarian reform. DECEMBER 4, 2017
STATE IMMUNITY FROM SUIT FACTS:
Aquino was invited by DPWH to a bidding for the
69. Merritt vs Gov construction of a dike by bulldozing a part of the
34 Phil 311 – Civil Law – Torts and Damages – Porac River at Barangay Ascomo-Pulungmasle,
Liability of the State for acts of special agents Guagua, Pampanga. Aquino was subsequently
Political Law – Non-Suability of the State – Waiver awarded the “Contract of Agreement” by the said
of Non-Suability is Not Admission of Liability government agency.
The facts of the case took place in the 1910’s. E. By 9 July 1992, the project was duly completed by
Merritt was a constructor who was excellent at his respondent, who was then issued a Certificate of
work. One day, while he was riding his motorcycle Project Completion dated 16 July 1992. However,
along Calle Padre Faura, he was bumped by a claimed that PhP1,262,696.20 was still due him, but
government ambulance. The driver of the ambulance petitioners refused to pay the amount. He thus filed a
was proven to have been negligent. Because of the Complaint for the collection of sum of money with
incident, Merritt was hospitalized and he was damages before the Regional Trial Court of Guagua,
severely injured beyond rehabilitation so much so Pampanga. Petitioners, for their part, set up the
that he could never perform his job the way he used defense that the Complaint was a suit against the
to and that he cannot even earn at least half of what state; that respondent failed to exhaust
he used to earn. administrative remedies; and that the “Contract of
In order for Merritt to recover damages, he sought to Agreement” covering the project was void for
sue the government which later authorized Merritt to violating Presidential Decree No. 1445, absent the
sue the government by virtue of Act 2457 enacted by proper appropriation and the Certificate of
the legislature (An Act authorizing E. Merritt to Availability of Funds.
bring suit against the Government of the Philippine The trial court ruled in favor of the respondent.
Islands and authorizing the Attorney-General of said Petitioners (DPWH) then appealed the case before
Islands to appear in said suit). The lower court then the CA which ruled in their favor, declaring the
determined the amount of damages and ordered the contract null and void ab initio but ordered
government to pay the same. compensation to Aquino for worked delivered,
ISSUE: Whether or not the government is liable for subject to Commission on Audit rules. Unsatisfied
the negligent act of the driver of the ambulance. with the CA’s decision, Petitioners then raised the
HELD: No. By consenting to be sued a state simply issue before the Supreme Court seeking complete
waives its immunity from suit. It does not thereby dismissal of the case without paying Aquino any
concede its liability to plaintiff, or create any cause money.
of action in his favor, or extend its liability to any
Page 64 of 70
ISSUE: 400,000,000 in favor of the Philippine government
Was the doctrine of sovereign immunity properly in order to finance the construction of Phase I of the
invoked? Northrail Project.
ISSUE:
RULING: Whether the Contract Agreement is an executive
No. The Supreme Court said that the doctrine of agreement.
governmental immunity from suit cannot serve as an RULING:
instrument for perpetrating an injustice to a citizen. Under the Vienna Convention on the Law of
Citing their decision in EPG Construction (G.R. No. Treaties defines a treaty as, “An international
131544, March 16, 2001, 354 SCRA 566), the court agreement concluded between States in written form
said that it would be the apex of injustice and highly and governed by international law, whether
inequitable if the respondent is not duly embodied in a single instrument or in two or more
compensated for actual work performed and services related instruments and whatever its particular
rendered, where both the government and the public designation.” The Court held that an executive
have received benefits from the project and reaped agreement is similar to a treaty, except that the
the fruits of respondent’s honest toil and labor. The former (a) does not require legislative concurrence;
Court further said that the no government agency or (b) is usually less formal; and (c) deals with a
agent can conveniently hide under the State’s cloak narrower range of subject matters.
of invincibility against suit, because this principle Despite these differences, to be considered an
has limitations especially when that the ends of executive agreement, the following three requisites
justice would be subverted if we were to uphold, in provided under the Vienna Convention must
this particular instance, the State’s immunity from nevertheless concur: (a) the agreement must be
suit. The Court finally said that in this case, it can’t between states; (b) it must be written; and (c) it must
be an instrument of injustice by upholding the governed by international law. The first and the third
immunity from suit principle and affirmed the requisites do not obtain in the case at bar.
decision of the Court of Appeals. The Contract Agreement was not concluded between
73. CHINA NATIONAL MACHINERY vs. the Philippines and China, but between Northrail
SANTAMARIA and CNMEG. By the terms of the Contract
G.R. No. 185572; February 7, 2012 Agreement, Northrail is a government-owned or -
FACTS: controlled corporation, while CNMEG is a
On 14 September 2002, petitioner China National corporation duly organized and created under the
Machinery & Equipment Corp. (CNMEG), laws of the People’s Republic of China. Thus, both
represented by its chairperson, Ren Hongbin, entered Northrail and CNMEG entered into the Contract
into a Memorandum of Understanding with the Agreement as entities with personalities distinct and
North Luzon Railways Corporation for the conduct separate from the Philippine and Chinese
of a feasibility study on a possible railway line from governments, respectively.
Manila to San Fernando, La Union. Neither can it be said that CNMEG acted as agent of
On 30 August 2003, the Export Import Bank of the Chinese government. As previously discussed,
China (EXIM Bank) and the Department of Finance the fact that Amb. Wang, in his letter dated 1
of the Philippines (DOF) entered into a October 2003, described CNMEG as a "state
Memorandum of Understanding, wherein China corporation" and declared its designation as the
agreed to extend Preferential Buyer’s Credit to the Primary Contractor in the Northrail Project did not
Philippine government to finance the Northrail mean it was to perform sovereign functions on
Project. The Chinese government designated EXIM behalf of China. That label was only descriptive of
Bank as the lender, while the Philippine government its nature as a state-owned corporation, and did not
named the DOF as the borrower. Under the Aug 30 preclude it from engaging in purely commercial or
MOU, EXIM Bank agreed to extend an amount not proprietary ventures.
exceeding USD 400,000,000 in favor of the DOF, It is therefore clear from the foregoing reasons that
payable in 20 years, with a 5-year grace period, and the Contract Agreement does not partake of the
at the rate of 3% per annum. nature of an executive agreement. It is merely an
On 1 October 2003, the Chinese Ambassador to the ordinary commercial contract that can be questioned
Philippines, Wang Chungui, wrote a letter to DOF before the local courts.
Secretary Jose Isidro Camacho informing him of 75. DOH v Phil Pharmawealth GR No. 182358
CNMEG’s designation as the Prime Contractor for Department of Health, Secretary Alfredo
the Northrail Project. On 30 December 2003, Romualdez, USec. Margarita Galon, petitioner;
Northrail and CNMEG executed a Contract Philippine Pharmawealth, Inc, respondent;
Agreement for the construction of Section I, Phase I
of the North Luzon Railway System from Caloocan February 20, 2013
to Malolos on a turnkey basis (the Contract Second Division
Agreement). The contract price for the Northrail Del Castillo, J.
Project was pegged at USD 421,050,000.
On 26 February 2004, the Philippine government FACTS:
and EXIM Bank entered into a counterpart financial On December 22, 1998, Administrative Order (AO)
agreement, the Buyer Credit Loan Agreement. In the No. 27 series of 1995 was issued by then
Loan Agreement, EXIM Bank agreed to extend Department of Health Secretary Alfredo G.
Preferential Buyer’s Credit in the amount of USD Romualdez. AO 27 sets the guidelines and procedure
Page 65 of 70
for accreditation of government suppliers of The Regional Trial Court dismissed the case,
pharmaceutical products for sale or distribution to holding that the suit is against the State, thus the
the public, such accreditation to be valid for three principle of immunity form suit is applicable.
years but subject to annual review. On appeal to the CA, however, the latter reversed
On January 25, 2000, Secretary Romualdez issued and set aside the RTC decision. According to the
AO 10 series of 2006 which amended AO 27. Under CA, it was premature for the RTC to have dismissed
Sec 7 of AO 10, accreditation period for government the case, as the cause of actions were sufficiently
suppliers of pharmaceutical products was reduced to alleged in the complaint. Further, by filing a
2 years. Also, accreditation of Pharmaceutical complaint, the DOH officials hypothetically
companies may be recalled, suspended or revoked admitted the allegations in the complaint-that they
after due deliberation and proper notice by the DOH were being sued in their official and private
Accreditation Committee, through its Chairman. capacities. Thus the DOH officials, herein
Sec 7 of AO 10 was later amended AO 66 series of petitioners, elevated the case to the Supreme Court,
2008 which stated that the 2 year accreditation may arguing that PPI’s prayer for damages should be
be recalled, suspended or revoked only after due considered a suit against the State for it would
deliberation, hearing and notice by the DOH require the needed appropriation to satisfy PPI’s
Accreditation Committee, through its Chairman. claim for damages should it win. In issuing the
On August 28, 2000, the DOH issued Memorandum assailed DOH issuances, they acted within the scope
No. 171-C9 which provided for a list and category of of their authority, hence should not be made to
sanctions to be imposed on accredited government account individually. Petition was granted.
suppliers. In line with Memorandum No. 171-C, the ISSUE:
DOH, through former Undersecretary Ma. Margarita Whether or not DOH, in this circumstance, is under
M. Galon, issued Memorandum No. 209 series of the mantle of state immunity.
2000 inviting representatives of 24 accredited drug HELD:
companies, including herein respondent Phil As a general rule, a state may not be sued. However,
Pharmawealth, Inc. (PPI) to a meeting on October if it consents, either expressly or impliedly, then it
27, 2000. may be the subject of a suit. There is express consent
During the meeting, Undersecretary Galon handed when a law, either special or general, so provides.
them copies of a document entitled “Report on On the other hand, there is implied consent when the
Violative Products” issued by the Bureau of Food state “enters into a contract or it itself commences
and Drugs (BFAD), which detailed violations or litigation.” However, it must be clarified that when a
adverse findings relative to these accredited drug state enters into a contract, it does not automatically
companies’ products. PPI’s products were included mean that it has waived its nonsuability. The State
as BFAD found that PPI’s products sold to the “will be deemed to have impliedly waived its non-
public were unfit for human consumption. suability [only] if it has entered into a contract in its
The companies were directed to submit their proprietary or private capacity. [However,] when the
respective explanations on the findings within 10 contract involves its sovereign or governmental
days. PPI did not submit its reply on time. Instead, it capacity, x x x no such waiver may be implied.”
submitted a letter stating that it is referring the “Statutory provisions waiving state immunity are
matter to its lawyers for preparation of a reply but construed in strictissimi juris. For, waiver of
with no indicated date of compliance, which DOH immunity is in derogation of sovereignty.”
Usec Galon found untenable, thus she informed PPI
thru letter that its accreditation had been suspended 77. Landbank vs Rivera 2010
for two years in accordance with AO 10 and SUMMARY: LBP, performing govt function is not
Memorandum No. 171-C. PPI thru letter, demanded liable for the cost of the suit.
that Usec Galon cease and desist from enforcing the
suspension under pain of legal redress. PETITION: Petition for review on certiorari by
PPI then filed a complaint to declare certain DOH Petitioner Land Bank of the Philippines (LBP)
issuances (Memorandum No. 171-C, AO 10, Series assailing the Decision of the Court of Appeals
2000, Usec Galon’s suspension order; and AO 14, ordering the payment by LBP of just compensation
Series 2001) null and void for being in violation of and interest in favor of respondents Esther Anson
Section 26, Republic Act 3720, with prayer for Rivera et al
injunction and damages against Usec Galon and later
DOH Secretary Dayrit. It claimed that its FACTS:
accreditation was suspended without due notice and Rivera are the co-owners of a parcel of agricultural
hearing. It prayed that it be awarded moral damages, land and later transferred in their names under
attorneys fees and costs of suit. Transfer Certificate of Title that was placed under
The respondent DOH officials filed a motion to the coverage of Operation Land Transfer pursuant to
dismiss, alleging that it gave PPI the opportunity to Presidential Decree No. 27.
explain but it did not do so in a timely manner. The After the Department of Agrarian Reform (DAR)
suspension was necessary to stop the distribution directed payment, LBP approved the payment of
and sale of substandard products. In a Manifestation P265,494.20, exclusive of the advance payments
and Motion, the DOH officials further moved to made in the form of lease rental amounting to
dismiss the case as it was a suit against the State; the P75,415.88 but inclusive of 6% increment of
complaint was improperly verified; and the P191,876.99 pursuant to DAR Administrative Order
corporate officer lacked the authority to file the suit. No. 13, series of 1994.
Page 66 of 70
Rivera instituted Civil Case No. 94-03 FOR THE COURT OF APPEALS’ RESOLUTION
DETERMINATION AND PAYMENT OF JUST DATED 18 MARCH 2008.
COMPENSATION BEFORE THE REGIONAL Thus this petition.
TRIAL COURT (RTC), claiming that the
landholding involved was irrigated with two ISSUES:
cropping seasons a year with an average gross I. Is it valid or lawful to award 12% rate of interest
production per season of 100 cavans of 50 per annum in favor of respondents notwithstanding
kilos/hectare, equivalent of 200 cavans/year/hectare; the 6% rate of interest per annum compounded
and that the fair market value of the property was not annually prescribed under DAR A.O. No. 13, series
less that P130,000.00/hectare, or P2,668,302.00 for of 1994, DAR A.O. No. 02, series of 2004, and
the entire landholding of 20.5254 hectares. DAR A.O. No. 06, series of 2008, "xxx from
LBP filed its answer, stating that rice and corn lands November 1994 up to the time of actual payment?
placed under the coverage of Presidential Decree (RELEVANT TO THE CASE)
No. 27 were governed and valued in accordance II. IS IT VALID OR LAWFUL TO ADJUDGE
with the provisions of Executive Order No. 228 etc. PETITIONER LBP, WHICH IS PERFORMING A
The RTC rendered its decision, holding: THE JUST GOVERNMENTAL FUNCTION, LIABLE FOR
COMPENSATION OF THE LAND PARTLY COSTS OF SUIT?
COVERED BY TCT NO. T-95690 IS FIXED AT
PHP1,297,710.63. HELD: the petition is GRANTED. (LBP performing
Land Bank of the Philippines is hereby ordered to GOV’T function IS EXEMPTED from cost of suit)
pay Esther Anson, Cesar Anson and Antonio Anson The decision of the Court of Appeals is
the aforesaid value of the land, plus interest of 12% AFFIRMED with the MODIFICATION that LBP is
per annum or Php194.36 per day effective October hereby held exempted from the payment of costs of
7, 2004, until the value is fully paid, in cash or in suit. In all other respects, the Decision of the Court
bond or in any other mode of payment at the option of Appeals is AFFIRMED. No costs.
of the landowners in accordance with Sec. 18, RA
6657. RATIO:
LBP filed MR – DENIED FIRST ISSUE
LBP next filed a petition for Review to the Court of At the outset, the Court notes that the parcels of land
Appeals then the DECISION DATED OCTOBER 6, subject matter of this case were acquired under In
2004 is MODIFIED, ordering petitioner LAND the instant case, while the subject lands were
BANK OF THE PHILIPPINES to pay to the acquired under Presidential Decree No. 27, the
respondents just compensation (inclusive of interests complaint for just compensation was only lodged
as of October 6, 2004) in the amount of before the court on 23 November 2000 or long after
P823,957.23, plus interest of 12% per annum on the the passage of Republic Act No. 6657 in 1998.
amount of P515,777.57, or P61,893.30 per annum, Therefore, Section 17 of Republic Act No. 6657
beginning October 7, 2004 until the just should be the principal basis of the computation for
compensation is fully paid in accordance with this just compensation.
decision. Conformably with the foregoing resolution, this
The Court of Appeals pointed out that: Court rules that a 12% interest per annum on just
Pursuant to AO 13, considering that the landholding compensation, due to the respondents, from the
involved herein was tenanted prior to October 21, finality of this decision until its satisfaction, is
1972, the rate of 6% per annum is imposed, proper.
compounded annually from October 21, 1972 until
October 21, 1994, the date of the effectivity of AO SECOND ISSUE:IMPORTANT!
13. WHETHER OR NOT THE COURT OF APPEALS
Beyond October 21, 1994, only the simple rate of CORRECTLY ADJUDGED LBP LIABLE TO PAY
6% per annum interest is imposable until October 6, THE COST OF SUIT.
2004 (the date of the rendition of the decision of the
RTC) on the total value (that is, P164,059.26 plus LBP’s argument
the compounded increments up to October 21, 1994) According to LBP, it performs a governmental
but minus the lease rentals of P75,415.88. function when it disburses the Agrarian Reform
Only the simple rate of 6% is applicable up to then Fund to satisfy awards of just compensation. Hence,
because the obligation to pay was not founded on a it cannot be made to pay costs in eminent domain
written agreement that stipulated a different rate of proceedings.
interest. From October 7, 2004 until the full NHA’s argument
payment, the simple interest rate is raised to 12% the NHA contends that it is exempt from paying all
per annum. kinds of fees and charges, because it performs
The reason is that the amount thus determined had governmental functions.
by then acquired the character of a forbearance in COURT’s DECISION
money. COURT AGREES – DO NOT NEED TO PAY
LBP DISAGREED WITH THE IMPOSITION OF COST – PERFORMING GOVT FUNCTIONS
12% INTEREST AND ITS LIABILITY TO PAY While it has not always been easy to distinguish
THE COSTS OF SUIT. IT FILED A MOTION FOR governmental from proprietary functions, the Court's
RECONSIDERATION WHICH WAS DENIED IN declaration in the Decision quoted above is not
without basis. Indeed, the characterization of
Page 67 of 70
governmental functions has veered away from the 78. Lockheed vs. UP G.R. No. 18591 AprIL 18,
traditional constituent-ministrant classification that 2012 Immunity from Suit
has become unrealistic, if not obsolete. DECEMBER 4, 2017
Justice Isagani A. Cruz avers: "[I]t is now obligatory FACTS:
upon the State itself to promote social justice, to Petitioner Lockheed Detective and Watchman
provide adequate social services to promote a rising Agency, Inc. (Lockheed) entered into a contract for
standard of living, to afford protection to labor to security services with respondent UP. In 1998,
formulate and implement urban and agrarian reform several security guards assigned to UP filed separate
programs, and to adopt other measures intended to complaints against Lockheed and UP for payment of
ensure the dignity, welfare and security of its underpaid wages, 25% overtime pay, premium pay
citizens.....These functions, while traditionally for rest days and special holidays, holiday pay,
regarded as merely ministrant and optional, have service incentive leave pay, night shift differentials,
been made compulsory by the Constitution." 13th month pay, refund of cash bond, refund of
The relevant provision of the Rules of Court states: deductions for the Mutual Benefits Aids System
RULE 142, “No costs shall be allowed against the (MBAS), unpaid wages from December 16-31,
Republic of the Philippines unless otherwise 1998, and attorney’s fees.
provided by law.”
Based from jurisprudence, LBP is an agency created ISSUE:
primarily to provide financial support in all phases Having a charter with which it can sue and be sued,
of agrarian reform pursuant to Section 74 of can UP funds be garnished?
Republic Act (RA) No. 3844 and Section 64 of RA
No. 6657. RULING:
It is vested with the primary responsibility and We agree with UP that there was no point for
authority in the valuation and compensation of Lockheed in discussing the doctrine of state
covered landholdings to carry out the full immunity from suit as this was never an issue in this
implementation of the Agrarian Reform Program. case. Clearly, UP consented to be sued when it
It is evident from the afore-quoted jurisprudence that participated in the proceedings below. What UP
the role of LBP in the CARP is more than just the questions is the hasty garnishment of its funds in its
ministerial duty of keeping and disbursing the PNB account. This Court finds that the CA correctly
Agrarian Reform Funds. applied the NEA case. Like NEA, UP is a juridical
As the Court had previously declared, the LBP is personality separate and distinct from the
primarily responsible for the valuation and government and has the capacity to sue and be
determination of compensation for all private lands. sued. Thus, also like NEA, it cannot evade
It has the discretion to approve or reject the land execution, and its funds may be subject to
valuation and just compensation for a private garnishment or levy. However, before execution
agricultural land placed under the CARP. In case the may be had, a claim for payment of the judgment
LBP disagrees with the valuation of land and award must first be filed with the COA.
determination of just compensation by a party, the 79. UP vs Dizon 2012
DAR, or even the courts, the LBP not only has the CASE DIGEST: UNIVERSITY OF THE
right, but the duty, to challenge the same, by appeal PHILIPPINES, et al. v. HON. AGUSTIN S.
to the Court of Appeals or to this Court, if DIZON, et al.
appropriate.
IT IS CLEAR FROM THE ABOVE FACTS: University of the Philippines (UP) entered
DISCUSSIONS THAT SINCE LBP IS into a General Construction Agreement with
PERFORMING A GOVERNMENTAL respondent Stern Builders Corporation (Stern
FUNCTION IN AGRARIAN REFORM Builders) for the construction and renovation of the
PROCEEDING, IT IS EXEMPT FROM THE buildings in the campus of the UP in Los Bas. UP
PAYMENT OF COSTS OF SUIT AS PROVIDED was able to pay its first and second billing. However,
UNDER RULE 142, SECTION 1 OF THE RULES the third billing worth P273,729.47 was not paid due
OF COURT. to its disallowance by the Commission on Audit
Thus, since LBP is performing a gov’t function, it is (COA). Thus, Stern Builders sued the UP to collect
thus exempted from paying the cost of suit. the unpaid balance.
In case the LBP disagrees with the valuation of land
and determination of just compensation by a party, On November 28, 2001, the RTC rendered its
the DAR, or even the courts, the LBP not only has decision ordering UP to pay Stern Builders. Then on
the right, but the duty, to challenge the same, by January 16, 2002, the UP filed its motion for
appeal to the Court of Appeals or to this Court, if reconsideration. The RTC denied the motion. The
appropriate. denial of the said motion was served upon
It is clear from the above discussions that since LBP Atty. Felimon Nolasco (Atty.Nolasco) of the UPLB
is performing a governmental function in agrarian Legal Office on May 17, 2002. Notably,
reform proceeding, it is exempt from the payment of Atty. Nolasco was not the counsel of record of the
costs of suit as provided under Rule 142, Section 1 UP but the OLS inDiliman, Quezon City.
of the Rules of Court.
Thereafter, the UP filed a notice of appeal on June 3,
2002. However, the RTC denied due course to the
notice of appeal for having been filed out of time.
Page 68 of 70
On October 4, 2002, upon motion of Stern Builders, giving the plaintiff the chance to prove, if it can, that
the RTC issued the writ of execution. the defendant is liable.

On appeal, both the CA and the High Court denied The Constitution strictly mandated that "no money
UPs petition. The denial became final and executory. shall be paid out of the Treasury except in pursuance
Hence, Stern Builders filed in the RTC its motion of an appropriation made by law." The execution of
for execution despite their previous motion having the monetary judgment against the UP was within
already been granted and despite the writ of the primary jurisdiction of the COA. It was of no
execution having already issued. On June 11, 2003, moment that a final and executory decision already
the RTC granted another motion for execution filed validated the claim against the UP.
on May 9, 2003 (although the RTC had already
issued the writ of execution on October 4, 2002). HELD: The period of appeal did not start without
Consequently, the sheriff served notices of effective service of decision upon counsel of record.
garnishment to the UPs depositary banks and the (The doctrine of immutability of a final judgment;
RTC ordered the release of the funds. service of judgments; fresh-period rule; computation
of time)
Aggrieved, UP elevated the matter to the CA. The
CA sustained the RTC. Hence, this petition. At stake in the UPs plea for equity was the return of
the amount of P16,370,191.74 illegally garnished
ISSUES: from its trust funds. Obstructing the plea is the
finality of the judgment based on the supposed
I. Was UP's funds validly garnished? tardiness of UPs appeal, which the RTC declared on
II. Has the UP's appeal dated June 3, 2002 been filed September 26, 2002. It is true that a decision that has
out of time? attained finality becomes immutable and unalterable,
and cannot be modified in any respect, even if the
HELD: UP's funds, being government funds, are not modification is meant to correct erroneous
subject to garnishment. (Garnishment of public conclusions of fact and law, and whether the
funds; suability vs. liability of the State) modification is made by the court that rendered it or
by this Court as the highest court of the land. But the
Despite its establishment as a body corporate, the doctrine of immutability of a final judgment has not
UP remains to be a "chartered institution" been absolute, and has admitted several exceptions,
performing a legitimate government function. among them: (a) the correction of clerical errors; (b)
Irrefragably, the UP is a government instrumentality, the so-called nunc pro tunc entries that cause no
performing the States constitutional mandate of prejudice to any party; (c) void judgments; and (d)
promoting quality and accessible education. As a whenever circumstances transpire after the finality
government instrumentality, the UP administers of the decision that render its execution unjust and
special funds sourced from the fees and income inequitable. We rule that the UPs plea for equity
enumerated under Act No. 1870 and Section 1 of warrants the Courts exercise of the exceptional
Executive Order No. 714, and from the yearly power to disregard the declaration of finality of the
appropriations, to achieve the purposes laid down by judgment of the RTC for being in clear violation of
Section 2 of Act 1870, as expanded in Republic Act the UPs right to due process.
No. 9500. All the funds going into the possession of
the UP, including any interest accruing from the Firstly, the service of the denial of the motion for
deposit of such funds in any banking institution, reconsideration upon Atty. Nolasco of the UPLB
constitute a "special trust fund," the disbursement of Legal Office was invalid and ineffectual because he
which should always be aligned with the UPs was admittedly not the counsel of record of the UP.
mission and purpose, and should always be subject Verily, the service of the denial of the motion for
to auditing by the COA. The funds of the UP are reconsideration could only be validly made upon the
government funds that are public in character. They OLS in Diliman, and no other. It is settled that
include the income accruing from the use of real where a party has appeared by counsel, service must
property ceded to the UP that may be spent only for be made upon such counsel. This is clear enough
the attainment of its institutional objectives. from Section 2, second paragraph, of Rule 13, Rules
of Court, which explicitly states that: "If any party
A marked distinction exists between suability of the has appeared by counsel, service upon him shall be
State and its liability. As the Court succinctly stated made upon his counsel or one of them, unless
in Municipality of San Fernando, La Union service upon the party himself is ordered by the
v. Firme: A distinction should first be made between court. Where one counsel appears for several parties,
suability and liability. "Suability depends on the he shall only be entitled to one copy of any paper
consent of the state to be sued, liability on the served upon him by the opposite side."
applicable law and the established facts. The
circumstance that a state is suable does not Secondly, even assuming that the service upon
necessarily mean that it is liable; on the other hand, Atty. Nolasco was valid and effective, such that the
it can never be held liable if it does not first consent remaining period for the UP to take a timely appeal
to be sued. Liability is not conceded by the mere fact would end by May 23, 2002, it would still not be
that the state has allowed itself to be sued. When the correct to find that the judgment of the RTC became
state does waive its sovereign immunity, it is only final and immutable thereafter due to the notice of
Page 69 of 70
appeal being filed too late on June 3, 2002. In so
declaring the judgment of the RTC as final against
the UP, the CA and the RTC applied the rule
contained in the second paragraph of Section 3, Rule
41 of the Rules of Court to the effect that the filing
of a motion for reconsideration interrupted the
running of the period for filing the appeal; and that
the period resumed upon notice of the denial of the
motion for reconsideration. For that reason, the CA
and the RTC might not be taken to task for strictly
adhering to the rule then prevailing.

However, equity calls for the retroactive application


in the UPs favor of the fresh-period rule that the
Court first announced in mid-September of 2005
through its ruling in Neypes v. Court of
Appeals, viz: "to standardize the appeal periods
provided in the Rules and to afford litigants fair
opportunity to appeal their cases, the Court deems it
practical to allow a fresh period of 15 days within
which to file the notice of appeal in the Regional
Trial Court, counted from receipt of the order
dismissing a motion for a new trial or motion for
reconsideration." The retroactive application of the
fresh-period rule, a procedural law that aims "to
regiment or make the appeal period uniform, to be
counted from receipt of the order denying the motion
for new trial, motion for reconsideration (whether
full or partial) or any final order or resolution," is
impervious to any serious challenge. This is because
there are no vested rights in rules of procedure.

Consequently, even if the reckoning started from


May 17, 2002, when Atty. Nolasco received the
denial, the UPs filing on June 3, 2002 of the notice
of appeal was not tardy within the context of the
fresh-period rule. For the UP, the fresh period of 15-
days counted from service of the denial of the
motion for reconsideration would end on June 1,
2002, which was a Saturday. Hence, the UP had
until the next working day, or June 3, 2002, a
Monday, within which to appeal, conformably with
Section 1 of Rule 22, Rules of Court, which holds
that: "If the last day of the period, as thus computed,
falls on a Saturday, a Sunday, or a legal holiday in
the place where the court sits, the time shall not run
until the next working day.

GRANTED

80.

Page 70 of 70

You might also like